You are on page 1of 101

Tailieumontoan.

com

Nguyễn Công Lợi

CHUYÊN ĐỀ CÁC BÀI TOÁN


VỀ TỔ HỢP VÀ SUY LUẬN

Nghệ An, tháng 9 năm 2019


1
Website:tailieumontoan.com

CHUYÊN ĐỀ TỔ HỢP VÀ SUY LUẬN

LỜI NÓI ĐẦU

Nhằm đáp ứng nhu cầu về của giáo viên toán THCS và học sinh về các chuyên đề toán

THCS, website tailieumontoan.com giới thiệu đến thầy cô và các em chuyên đề về các bài toán tổ

hợp và suy luận. Chúng tôi đã kham khảo qua nhiều tài liệu để viết chuyên đề này nhằm đáp ứng

nhu cầu về tài liệu hay và cập nhật được các dạng toán mới về toán tổ hợp và suy luận thường

được ra trong các kì thi gần đây. Chuyên đề này gôm các chủ đề toán sau:

- Chủ đề 1: Các dạng toán về nguyên lý Dirichlet

- Chủ đề 2: Các bài toán về ứng dụng nguyên lý cực hạn

- Chủ đề 3: Các bài toán về đại lượng bất biến và ứng dụng

- Chủ đề 4: Một số bài toán tổ hợp suy luận tổng hợp

Các vị phụ huynh và các thầy cô dạy toán có thể dùng có thể dùng chuyên đề toán tổ hợp và
suy luận để giúp con em mình học tập. Hy vọng chuyên đề này sẽ có thể giúp ích nhiều cho học
sinh phát huy nội lực giải toán nói riêng và học toán nói chung.
Mặc dù đã có sự đầu tư lớn về thời gian, trí tuệ song không thể tránh khỏi những hạn chế,
sai sót. Mong được sự góp ý của các thầy, cô giáo và các em học!
Chúc các thầy, cô giáo và các em học sinh thu được kết quả cao nhất từ chuyên đề này!

Tác giả: Nguyễn Công Lợi TÀI LIỆU TOÁN HỌC


2
Website:tailieumontoan.com
MỘT SỐ BÀI TOÁN TỔ HỢP

Chủ đề 1

CÁC BÀI TOÁN ỨNG DỤNG NGUYÊN LÝ DIRICHLET

I. Nguyên lí Dirichlet.

Nguyên lí Dirichlet - còn gọi là nguyên lí chim bồ câu (The Pigeonhole Principle) hoặc

nguyên lý những cái lồng nhốt thỏ hoặc nguyên lí sắp xếp đồ vật v|o ngăn kéo (The Drawer

Principle) - đưa ra một nguyên tắc về phân chia phần tử các lớp.

 Nguyên lý Dirichlet cơ bản: Nếu nhốt n  1 con thỏ vào n cái chuồng thì bao giờ cũng có
một chuồng chứa ít nhất hai con thỏ.

 Nguyên lý Dirichlet tổng quát: Nếu có N đồ vật được đặt vào trong k hộp thì sẽ tồn tại một hộp

N
chứa ít nhất   đồ vật. (Ở đ}y  x  là số nguyên nhỏ nhất có giá trị nhỏ hơn hoặc bằng x)
k

 Nguyên lí Dirichlet mở rộng: Nếu nhốt n con thỏ vào m  2 cái chuồng thì tồn tại một

 n  m  1
chuồng có ít nhất   con thỏ.
 m 

 Nguyên lí Dirichlet dạng tập hợp: Cho A và B là hai tập hợp khác rỗng có số phần tử

hữu hạn, mà số lượng phần tử của A lớn hơn số lượng phần tử của B. Nếu với một quy tắc

n|o đó, mỗi phần tử của A cho tương ứng với một phần tử của B, thì tồn tại ít nhất hai

phần tử khác nhau của A m| chúng tương ứng với một phần tử của B.

II. Phương pháp ứng dụng.

Nguyên lí Dirichlet tưởng chừng như đơn giản như vậy, nhưng nó l| một công cụ

hết sức có hiệu quả dùng để chứng mình nhiều kết quả hết sức sâu sắc của toán học.

Nguyên lí Dirichlet cũng được áp dụng cho các bài toán của hình học, điều đó được thể

hiện qua hệ thống bài tập sau:

Để sử dụng nguyên lý Dirichlet ta phải làm xuất hiện tình huống nhốt ‚thỏ‛ v|o

‚chuồng‛ v| thoả mãn c{c điều kiện:

+ Số ‘thỏ‛ phải nhiều hơn số chuồng.


Tác giả: Nguyễn Công Lợi TÀI LIỆU TOÁN HỌC
3
Website:tailieumontoan.com
+ ‚Thỏ‛ phải được nhốt hết vào các ‚chuồng‛, nhưng không bắt buộc chuồng nào

cũng phải có thỏ.

Thường thì phương ph{p Dirichlet được áp dụng kèm theo phương ph{p phản

chứng. Ngoài ra nó còn có thể áp dụng với các nguyên lý khác.

III. Một số ví dụ minh họa.

Ví dụ 1. Cho bảng ô vuông kích thước 10.10 gồm 100 ô vuông đơn vị. Điền v|o mỗi ô

vuông của bảng n|y một số nguyên dương không vượt qu{ 10 sao cho hai số ở hai ô

vuông chung cạnh hoặc chung đỉnh nguyên tố cùng nhau. Chứng minh rằng trong bảng ô

vuông đã cho có một số xuất hiện ít nhất 17 lần.

Lời giải

Xét hình vuông cạnh 2x2 , do hình vuông n|y có mỗi hình vuông nhỏ luôn chung

cạnh hoặc chung đỉnh nên tồn tại nhiều nhất 1 số chẵn, nhiều nhất 1 số chia hết cho 3 do

đó có ít nhất 2 số lẻ không chia hết cho 3. Bảng 10x10 được chia th|nh 25 hình vuông có

cạnh 2x2 nên có ít nhất 50 số lẻ không chia hết cho 3. Từ 1 đến 0 có 3 số lẻ không chia hết

cho 3 là 1, 5, 7. Áp dụng nguyên lí Dirichlet ta được một trong ba số trên xuất hiện ít

 50 
nhất    1  17 lần
3

Ví dụ 2. Giả sử 1 bàn cờ hình chữ nhật có 3x7 ô vuông được sơn đen hoặc trắng. Chứng minh rằng

với c{ch sơn m|u bất kì thì trong bàn cờ luôn tồn tại hình chữ nhật gồm các ô ở 4 góc là các ô cùng

màu.

Lời giải

Mẫu sơn m|u có thể xảy ra với bàn cờ này có dạng từ 1 đến 8. Giả sử một trong số các cột

thuộc dạng 1. Bài toán sẽ được chứng minh nếu tất cả các cột còn lại thuộc dạng 1, 2, 3 hoặc 4. Giả

sử tất cả các cột còn lại thuộc dạng 5, 6, 7, 8 khi đó theo nguyên lí Dirichlet thì hai trong số sau cột

có 2 cột cùng 1 dạng v| như vậy b|i to{n cũng được chứng minh

Chứng minh ho|n to|n tương tự nếu 1 cột có dang 8. Giả sử không có cột nào trong các cột

1, 8 thì theo nguyên lí Dirichlet cũng có 2 cột cùng dạng v| b|i to{n cũng đựoc chứng minh

Tác giả: Nguyễn Công Lợi TÀI LIỆU TOÁN HỌC


4
Website:tailieumontoan.com

Ví dụ 3. Trong hình chữ nhật kích thước 1.2 ta lấy 6n 2  1 điểm với n là số nguyên dương. Chứng

1
minh rằng tồn tại 1 hình tròn có bán kính chứa không ít hơn 4 trong số c{c điểm đã cho.
n

Lời giải

Chia các cạnh của hình chữ nhật th|nh n đoạn v| 2n đoạn bằng nhau ,mỗi đoạn có độ dài

1
. Nối c{c điểm chia bằng c{c đường thẳng song songvới các cạnh của hình chữ nhật ta được
n
1
n.2n  2n 2 hình vuông nhỏ với cạnh là . Nếu mỗi hình vuông chứa không qu{ 3 điểm thì tổng
n
số điểm đã cho không qu{ 3.2n2  6n 2 (trái với giả thiết). Do đó phải tồn tại 1 hình vuông chứa

1 2
không ít hơn 4 điểm. Rõ ràng hình vuông cạnh nội tiếp đường tròn bán kính là v| đường
n 2n
1
tròn n|y được chứa trong đường tròn đồng tâm bán kính .
n

Ví dụ 4. Cho bảng vuông gồm n.n ô vuông. Mỗi ô vuông ghi một trong các số 1; 0; 2. Chứng minh

rằng không tìm được bảng vuông nào mà tổng các số trên cột, trên h|ng, trên đường chéo là các

số khác nhau.

Lời giải
Do trong các ô có thể nhận một trong ba số 0; 1; 2 nên có thể có trường hợp tất cả các ô của

một hàng hoặc một cột hoặc một đường chéo nhận giá trị 0 hoặc nhận giá trị 2.

Do đó tổng các số trên cột hoặc trên hàng hoặc trên đường chéo có giá trị nhỏ nhất là

0.n  0 và giá trị lớn nhất là 2.n  2n . Như vậy các tổng các số trên mỗi hàng, mỗi cột, mỗi
đường chéo có thể nhận 2n  1 giá trị là 0;1; 2;...; 2n

Do bảng ô vuông n.n nên sẽ có n hàng, n cột v| hai đường chéo. Do đó sẽ có 2n  2 tổng

nhận một trong 2n  1 giá trị số nguyên từ 0 đến 2n. Theo nguyên tắc Dirichlet phải có ít nhất 2

tổng có giá trị bằng nhau. Điều n|y có nghĩa l| không tìm được bảng vuông nào mà tổng các số

trên cột, trên h|ng, trên đường chéo là các số khác nhau.

Ví dụ 5. Ở vòng chung kết cờ vua có 8 bạn tham gia. Hai bạn bất kỳ đều phải đấu với nhau một

trận v| người n|o cũng phải gặp đủ 7 đấu thủ của mình. Chứng minh rằng trong mọi thời điểm

của cuộc đấu, bao giờ cũng có hai đấu thủ đã đấu một số trận như nhau.

Tác giả: Nguyễn Công Lợi TÀI LIỆU TOÁN HỌC


5
Website:tailieumontoan.com
Lời giải

Giả sử số trận thi đấu của các bạn tham gia thi đấu cờ vua là a1 ; a 2 ;...; a 8 . Do hai bạn thi đấu

với nhau một trận nên ta có 0  a i  7, 1  i  8 . Xét c{c trường hợp sau:

 Tính đến thời điểm đó có một bạn chưa đấu trận nào suy ra không có bạn n|o đấu đủ 7 trận.

Khi đó 0  a i  6, 1  i  8 do đó tồn tại a k  a m có nghĩa l| có hai đấu thủ đã đấu một số trận

như nhau.

 Tính đến thời điểm đang xét, mỗi bạn đều đã đấu ít nhất một ván.

Khi đó ta có 0  a i  7, 1  i  8 , do đó tồn tại a k  a m có nghĩa l| có hai đấu thủ đã đấu một số

trận như nhau.

Vậy b|i to{n được chứng minh.

Ví dụ 6. Cho 40 số nguyên dương a1 ,a 2 ,...,a19 và b1 , b2 ,..., b21 thoả mãn hai điều kiện:

1  a1  a 2  ...  a19  200 và 1  b1  b2  ...  b21  200

Chứng minh rằng tồn tại bốn số a i ;a j ;bk ;bp với 1  i, j  19;1  k,p  21 thỏa mãn


a i  a j ; bk  bp

a j  a i  bp  bk

Lời giải

 
Xét các tổng có dạng a m  bn với a m  a1 ; a 2 ;...; a19 và bn  b1 ; b2 ;...; b21  .


Do tập hợp a1 ;a 2 ;...;a 19  có 19 phần tử và tập hợp b1 ; b2 ;...; b 21  có 21 phần tử nên, nên ta có
tất cả 19.21  399 tổng dạng a m  bn như thế.

Chú ý rằng 1  a1  a 2  ...  a19  200 và 1  b1  b2  ...  b21  200 nên 2  a m  bn  400 .

Nên các tổng a m  bn nhận các giá trị nguyên dương từ 2 đến 400. Đến đ}y ta xét c{c trường hợp

sau:

 Nếu các tổng trên nhận đủ 399 giá trị từ 2 đến 400. Khi đó từ giả thiết cảu b|i to{n ta được

Tác giả: Nguyễn Công Lợi TÀI LIỆU TOÁN HỌC


6
Website:tailieumontoan.com
a1  b1  2 a  b1  1
  1
a19  b21  400 a19  b21  200

a1  a19 ; b1  b21


Từ đó ta suy ra được 
a19  a1  b21  b1  199

 Nếu các tổng trên không nhận đủ 399 giá trị từ 2 đến 400. Khi đó với 399 tổng thì theo nguyên lí
Dirichlet sẽ tồn tại hai tổng có giá trị bằng nhau.


a i  a j ; bk  bp
Không mất tính tổng quát ta giả sử hai tổng đó l| 
a j  bk  bp  a i


a i  a j ; bk  bp
Từ đó suy ra 
a j  a i  bp  bk

Vậy b|i to{n được chứng minh.

Ví dụ 7. Trong một cuộc tranh giải vô địch quốc gia về bóng đ{ có 20 đội tham gia. Số nhỏ nhất các

trận đấu l| bao nhiêu để trong 3 đội bất kỳ luôn tìm được 2 đội đã chơi với nhau.

Lời giải

Ta chia 20 đội thành 2 nhóm, mỗi nhóm 10 đội và chỉ c{c đội trong cùng1 nhóm mới thi

đấu với nhau. Rõ ràng cách sắp xếp này thoả mãn c{c điều kiện của bài toán và tất cả có 90 trận

đấu. Ta chứng minh rằng nếu c{c điều kiện của bài toán thoả mãn thì số trận đấu sẽ lớn hơn hoặc

bằng 90.

Giả sử ngược lại ta tìm đội một A đấu số trận k  8 . Ta ký hiệu c{c đội đã đấu với A là X.

C{c đội không đấu với A l| Y, khi đó X  k; Y  19  k . Dĩ nhiên c{c đội trong Y sẽ đấu với nhau

nếu không hai đội thuộc Y và A sẽ l| 3 đội m| không có đội n|o chơi với nhau. Giả sử trong X có P

cặp không chơi với nhau. Do đó mỗi đội Y phải đấu với mỗi đội trong P cặp đó của X và mỗi đội

trong X có mặt không quá k  1 cặp trong số P cặp (X có tất cả k đội). Vì vậy giữa c{c đội của X và

19  k 19  k
Y đấu số trận bé hơn hoặc bằng . Mặt khác do k  8 nên .P  P .
k 1 k 1

Như vậy nếu thay các trận của c{c đội trong X đấu với c{c đội trong Y bởi các trận đấu cần

thiết sẽ giảm đi. Như vậy số trận đấu cần phải tiến hành là:

Tác giả: Nguyễn Công Lợi TÀI LIỆU TOÁN HỌC


7
Website:tailieumontoan.com
k  k  1 19  k 18  k   k  18k  9.19   k  9   90  90
2
 2

2 2

Vậy số các trận đấu ít nhất cần phải tiến hành là 90

Ví dụ 8. Chứng minh rằng trong 39 số tự nhiên liên tiếp bất kỳ luôn tồn tại ít nhất một số có tổng

các chữ số chia hết cho 11.

Lời giải

Xét tập hợp 39 số tự nhiên liên tiếp S  a1 ;a 2 ;...;a 39  ,  a i 1  a i  1, 1  i  38 

Trong tập a1 ;a 2 ;...;a 20  luôn tồn tại hai số có tận cùng l| 0 v| hơn kém nhau 10. Do đó trong hai

số này tồn tại ít nhất một số có chữ số hàng chục nhỏ hơn 9, kí hiệu số đó l|

A  Bc0  0  c  8, c  , B  
Xét 11 số A; A  1; A  2;...; A  9; A  19 . Nhận xét rằng:

+ 11 số trên thuộc tập S.

+ 11 số đó có tổng các chữ số là 11 số tự nhiên liên tiếp vì các tổng đó l|:

s  A  ; s  A   1; s  A   2;...; s  A   9; s  A   10 ,với s  A  là tổng các chữ số của A.

Trong 11 số tự nhiên liên tiếp luôn tồn tại một số chia hết cho 11.

Do vậy, ta có điều phải chứng minh.

Ví dụ 9. Cho tập A  1; 2; 3;...;16 . Hãy tìm số nguyên dương k nhỏ nhất sao cho trong mỗi tập

con gồm k phần tử của A đều tồn tại hai số phân biệt a, b mà a 2  b2 là một số nguyên tố.

Lời giải

Nếu a, b chẵn thì a 2  b2 là hợp số. Do đó nếu tập con X của A có hai phần tử phân biệt

a, b mà a 2  b2 là một số nguyên tố thì X không thể chỉ chứa các số chẵn. Suy ra k  9 . Ta chứng

tỏ k  9 là giá trị nhỏ nhất cần tìm. Điều đó có ý nghĩa l| với mọi tập con X gồm 9 phần tử bất kỳ

của A luôn tồn tại hai phần tử phân biệt a, b mà a 2  b2 là một số nguyên tố.

Tác giả: Nguyễn Công Lợi TÀI LIỆU TOÁN HỌC


8
Website:tailieumontoan.com
Để chứng minh khẳng định trên ta chia tập A thành các cặp hai phần tử phân biệt a, b mà

a 2  b2 là một số nguyên tố, ta có tất cả 8 cặp

1; 4  , 2; 3 , 5; 8 , 6;11 , 7;10 , 9;16 , 12;13 , 14;15


  . Theo nguyên lý Dirichlet thì 9 phần
tử của X có hai phần tử cùng thuộc một cặp v| ta có điều phải chứng minh.

Ví dụ 10. Cho 2014 số tự nhiên bất kỳ. Chứng minh rằng trong số các số đó có một số chia hết cho

2014 hoặc có một số số mà tổng của các số ấy chia hết cho 2014.

Lời giải

Gọi 2014 số tự nhiên đã cho l| a1 ,a 2 , ,a 2014 .

Xét dãy S1  a1 ; S2  a1  a 2 ; ...; S2014  a1  a 2   a 2014

Chia tất cả các số hạng của dãy cho 2014 ta có c{c trường hợp sau:

 Trường hợp 1: Nếu có một số hạng nào của dãy chia hết cho 2014 thì b|i to{n được chứng minh.

 Trường hợp 2: Nếu không có số hạng nào của dãy chia hết cho 2014 thì vì có tất cả 2014 phép
chia mà số dư chỉ gồm 1, 2, ..., 2013 do đó theo nguyên lý Dirichle có ít nhất hai số hạng của dãy

có cùng số dư khi chia cho 2014. Gọi hai số hạng đó l| S i và S j .

Không mất tính tổng quát, giả sử 1  i  j  2014 thì

S i  a1  a 2   ai và S j  a1  a 2   ai   aj

Lúc đó S j  Si 2014  a i 1   a j 2014 . Từ đó ta có điều phải chứng minh.

Ví dụ 11. Chứng minh rằng từ 53 số tự nhiên bất kì luôn chọn được 27 số mà tổng của chúng chia

hết cho 27.

Lời giải

Ta chứng minh từ 5 số tự nhiên bất kì luôn tìm được 3 số mà tổng của chúng chia hết cho 3. Thật

vậy, mỗi số tự nhiên khi chia cho 3 thì có phần dư l| 0, 1 hoặc 2

Nếu trong 5 số dư có một số bằng 0, một số bằng 1, một số bằng 2 thì tổng của ba số tự nhiên

tương ứng với ba số dư n|y l| chia hết cho 3

Tác giả: Nguyễn Công Lợi TÀI LIỆU TOÁN HỌC


9
Website:tailieumontoan.com
Nếu 5 số dư chỉ nhận không quá 2 trong 3 số 0, 1, 2 thì theo nguyên tắc Dirichlet thì tồn tại 3

số dư nhận cùng một giá trị và tổng của ba số tự nhiên tương ứng là chia hết cho 3

Từ 53 số tự nhiên đã cho chọn được 3 số mà tổng của chúng là a1 chia hết cho 3. Xét 50 số còn

lại chọn được 3 số mà tổng là a2 chia hết cho 3. Lặp lại lập luận này từ 53 số ta chọn được 17 bộ,

mỗi bộ gồm 3 số có tổng lần lượt là a1, a2, < a17 sao cho mỗi tổng đều chia hết cho 3.

Chứng minh tương tự nhận thấy từ 5 số tự nhiên bất kì mà mỗi số đều chia hết cho 3 ta chọn

được 3 số có tổng chia hết cho 9. Vậy từ 17 số ta chọn được 5 bộ mỗi bộ gồm 3 số có tổng lần lượt

là b1 , b2 , , b5 sao cho bi 9 với i  1; 2; 3; 4; 5

Từ 5 số chia hết cho 9 là b1 , b2 , , b5 chọn được 3 số mà tổng của chúng là chia hết cho 27.

Tổng của 3 số này chính là tổng của 27 số ban đầu. Vậy từ 53 số tự nhiên bất kì luôn chọn được 27

số mà tổng của chúng chia hết cho 27.

Ví dụ 12. Trong một giải bóng đ{ có 12 đội tham dự, thi đấu vòng tròn một lượt(hai đội bất kì thi

đấu với nhau đúng một trận).

a) Chứng minh rằng sau bốn vòng đấu(mỗi đội thi đấu đúng 4 trận) luôn tìm được ba đội đôi

một chưa thi đấu với nhau

b) Khẳng định còn đúng không nếu mỗi đội thi đấu đúng 5 trận.

Lời giải

a) Có 12 đội mà mỗi đội thí đấu đúng 4 trận nên luôn tìm được hai đội chưa thi đấu với nhau. Gọi

hai đội đó l| A v| B. Vì A v| B thi đấu đúng 4 trận nên trong 10 đội còn lại luôn tìm được ít nhất

hai đội chưa thi đấu với cả A và B. Gọi một trong hai đội đó l| C. Ba đội A, B, C chưa thi đấu với

nhau một trận n|o nên ba đội A, B, C l| ba đội cần tìm.

b) Ta chia 12 đội bóng trên thành hai nhóm, mỗi nhóm 6 đội. Trong mỗi nhóm đôi một thi đấu với

nhau. Như vậy trong 12 đội này, mỗi đội thi đấu đúng 5 trận. Xét ba đội tùy ý, theo nguyên lí

Dirichlet luôn tìm được hai đội cùng nhóm. Như vậy trong ba đội bóng bất kì luôn tìm được hai

đội thí đấu với nhau. Do đó khẳng định trên không còn đúng nếu mỗi đội thi đấu đúng 5 trận.

Tác giả: Nguyễn Công Lợi TÀI LIỆU TOÁN HỌC


10
Website:tailieumontoan.com
Ví dụ 13. Cho X là một tập hợp gồm 700 số nguyên dương đôi một khác nhau, mỗi số không lớn

hơn 2006. Chứng minh rằng trong tập hợp X luôn tìm được hai phần tử x, y sao cho x – y thuộc tập


hợp E  3; 6; 9 .
Lời giải

 700 
Theo nguyên lý Dirichlet thì trong 700 số có ít nhất    1  234 số có cùng số dư khi chia cho
 3 
3. Gọi 234 số đó l| 1  a1  a 2  ...  a 234  2006 . Giả sử không tồn tại hai số a i ; a j nào thỏa mãn

 
ai  a j 3; 6; 9 do đó a i  a j  12 (vì a i  a j 3 và a i  a j ). Trong 234 số trên, hai số kề nhau

hơn kém nhau ít nhất 12 đơn vị nên a 234  a1  233.12  2796  2006 , điều này vô lý.

Như vậy ta có điều phải chứng minh.

Chú ý:

+ Ta có thể làm chặt bài toán bằng cách giảm số các số cho ban đầu hoặc tăng gi{ trị cho các số có

thể nhận.

Ta có thể làm chặt bài toán bằng cách thay 700 số thành 504 số. Gọi 504 số nguyên dương đôi một

kh{c nhau đã cho l| 1  a1  a 2  ...  a 504  2006 . Xét 504  4  2016 số nguyên dương như sau:

a1 a2 < a 503 a 504

a1  3 a2  3 < a 503  3 a 504  3

a1  6 a2  6 < a 503  6 a 504  6

a1  9 a2  9 < a 503  9 a 504  9

Vì các số trong bảng trên nhận các giá trị nguyên từ 1 đến 2006  9  2015 nên theo nguyên lí

 2016 
Dirichlet, có ít nhất    1  2 số nhận cùng một giá trị hay có hai số bằng nhau, suy ra điều
 2015 
phải chứng minh.

Tác giả: Nguyễn Công Lợi TÀI LIỆU TOÁN HỌC


11
Website:tailieumontoan.com
Ví dụ 14. Cho năm số nguyên dương đôi một phân biệt sao cho mỗi số trong chúng không có ước

số nguyên tố nào khác 2 và 3. Chứng minh rằng trong năm số đó tồn tại hai số mà tích của chúng

là một số chính phương.

Lời giải

Gọi các số đã cho l| a1 ;a 2 ;a 3 ;a 4 ;a 5 vì các số này không có ứơc số nguyên tố nào khác 2 và

3 nên các số n|y đều có dạng a i  2 i .3


x yi
với xi ; y i là các số tự nhiên.

Xét 5 cặp số  x1 ; y1  ,  x2 ; y2  ,  x3 ; y3  ,  x4 ; y 4  ,  x5 ; y 5  mỗi cặp số này nhận giá trị một

trong bốn trường hợp sau (số chẵn; số chẵn), (số chẵn; số lẻ), (số lẻ; số lẻ) và (số lẻ; số chẵn) nên

theo nguyên lí Dirichlet thì có ít nhất 2 cặp số trên nhận cùng một dạng giá trị. Không mất tính

tổng quát khi giả sử  x1 ; y1  ,  x2 ; y 2  cùng nhận giá trị dạng( số chẵn; số lẻ). Khi đó x1  x2 và

y1  y 2 đều là số chẵn nên a1 .a 2  2x1 .3y1 .2x2 .3y2  2x1 x2 .3y 1  y2 là một chính phương. Do đó ta có

điều phải chứng minh.

Ví dụ 15. Cho lưới ô vuông kích thước 5x5 . Người ta điền vào mỗi ô của lưới một trong các số

1; 0; 1 . Xét tổng của các số được tính theo từng cột, theo từng hàng và theo từng đường chéo.
Chứng minh rằng trong tất cả các tổng đó luôn tồn tại hai tổng có giá trị bằng nhau.

Lời giải

Có tất cả 12 tổng gồm 5 tổng theo cột, 5 tổng theo hàng và 2 tổng theo đường chéo. Mỗi

tổng gồm năm số hạng mà mỗi số hạng nhận một trong ba số là 1 hoặc 0 hoặc 1 . Do mỗi tổng là

một số nguyên.

Gọi các tổng đó l| S i với i  1; 2; 3;...;12 thỏa mãn 5  Si  5 .

Vậy S i có thể nhận trong mười một giá trị 5; 4; 3;...; 0;1;...;5 .

Mà ta lại có 12 tổng S i nên theo nguyên lí Dirichlet thì có ít nhất hai tổng nhận cùng một giá trị.

Vậy ta có điều phải chứng minh.

Ví dụ 16. Cho n  3 số nguyên dương a1 ;a 2 ;a 3 ;...;a n đôi một khác nhau. Tìm giá trị lớn nhất của

n sao cho tổng của ba số bất kỳ trong n số đó luôn l| một số nguyên tố.

Tác giả: Nguyễn Công Lợi TÀI LIỆU TOÁN HỌC


12
Website:tailieumontoan.com
Lời giải

Dễ thấy với n  3 ta luôn tìm được các số nguyên dương thỏa mãn yêu cầu bài toán.

Với n  4 ta xét c{c trường hợp sau:

 Trường hợp 1: Với n  4 , ta tìm được bốn số nguyên dương 1, 3, 7, 9 thỏa mãn yêu cầu bài toán.

 Trường hợp 2: Với n  5 , ta sẽ chứng minh luôn tìm được ba số có tổng lớn hơn 3 v| chia hết
cho 3.

Thật vậy, một số nguyên khi chia cho 3 có thể có số dư l| hoặc 0 hoặc 1 hoặc 2. Theo nguyên lý

Dirichlet thì trong 5 số nguyên dương bất kỳ có ít nhất hai số có cùng số dư khi chia cho 3.

+ Nếu có nhiều hơn 2 số có cùng số dư khi chí cho 3 thì có ít nhất 3 số có cùng số dư khi chia cho 3.

Chọn 3 số này thì tổng của chúng chia hết cho 3.

+ Nếu có đúng 2 số có số dư r với r  0;1; 2 thì loại hai số n|y, khi đó ta còn lại 3 số có số dư

khác r. Theo nguyên lý Dirichlet thì có ít nhất 2 số có cùng số dư kh{c r v| một số còn lại có số dư

khác số dư của hai số n|y. Như vậy trong 5 số đó luôn tồn tại 3 số có 3 số dư kh{c nhau khi chia

cho 3. Chọn 3 số này thì tổng của chúng chia hết cho 3.

Do đó trong 5 số nguyên dương ta luôn chọn được 3 số có tổng chia hết cho 3 và tổng này lớn hơn

3 nên nó không phải là số nguyên tố. Từ đó suy ra n  5 thì không thỏa mãn yêu cầu bài toán.

Vậy giá trị lớn nhất thỏa mãn yêu cầu bài toán là n  4 .

Ví dụ 17. Mỗi đỉnh của hình lập phương được điền một trong các số 1; 2; 3; 4; 5; 6;7; 8 . Hai đỉnh

kh{c nhau điền hai số kh{c nhau. Người ta tính tổng hai số ở hai đỉnh kề nhau. Chứng minh rằng

trong các tổng tính được có ít nhất hai tổng bằng nhau.

Lời giải

Do mỗi đỉnh của hình lập phương nhận các giá trị khác nhau từ các số 1; 2; 3; 4; 5; 6;7; 8 . Do

ta tính tổng hai số ở hai đỉnh kề nhau nên ta có 12 tổng. Khi đó mỗi tổng là một số nguyên dương

nhận các giá trị thuộc tập 3; 4; 5;...;13;14;15 . Ta sẽ chứng minh trong 12 tổng này không thể

đồng thời nhận các giá trị 3, 4, 5 ,6 và không thể đồng thời nhận các giá trị 12, 13, 14, 15.

Thật vậy, giả sử có các tổng nhận các giá trị 3, 4, 5, 6. Ta kí hiệu đỉnh K l| đỉnh được điền số K.

Tác giả: Nguyễn Công Lợi TÀI LIỆU TOÁN HỌC


13
Website:tailieumontoan.com
Ta có 3  1  2; 4  1  3 nên đỉnh 1 v| đỉnh 2 kề nhau, đỉnh 1 kề với đỉnh 3. Do đó đỉnh 2 v| đỉnh 3

không kề nhau. Vì 5  1  4 hoặc 5  2  3 , nhưng đỉnh 2 v| đỉnh 3 không kề nhau nên đỉnh 1 và

đỉnh 4 kề nhau. Do đó đỉnh 1 lần lượt kề với đỉnh 2, đỉnh 3, đỉnh 4, suy ra đỉnh 1 không kề với

đỉnh 5, đỉnh 2 không kề với đỉnh 4. Vì vậy không xuất hiện tổng có giá trị bằng 6.

Với các tổng nhận các giá trị 12, 13, 14, 15 ta chứng minh ho|n to|n tương tự.

Từ đó suy ra 12 tổng nhận không quá 11 giá trị nên theo nguyên lí Dirichslet có ít nhất hai tổng

bằng nhau.

 
Ví dụ 18. Cho tập hợp X  1; 2; 3;...; 2024 . Chứng minh rằng trong 90 số khác nhau bất kỳ

1
được lấy ra từ tập X luôn tồn tại hai số x và y sao cho x  y  .
2

Lời giải

Chia 2012 số 1; 2; 3;...; 2012 th|nh 44 đoạn gồm  1; 3  ,  4; 8  ,...,  1936; 2012  .
     
 
 k  1
2
C{c đoạn trên có dạng tổng quát là  k 2 ;  1 .
 

Như vậy 90 số thuộc tập hợp X nằm trong 44 đoạn trên. Theo nguyên lí Dirichlet thì tồn tại ba số

trong 90 số trên nằm trong cùng một đoạn. Không mất tính tổng quát ta giả sử hai số đó l| x, y, z

 
 k  1
2
và chúng nằm trong đoạn  k 2 ;  1 .
 

   
 k  1  1  th|nh hai đoạn là  k 2 ; k 2  k  và  k 2  k;  k  1
2 2
Chia đoạn  k 2 ;  1
     

Khi đó theo nguyên lí Dirichlet tồn tại hai số nằm trên cùng một đoạn. Giả sử hai số đó l| x v| y.

Khi đó

 Nếu x và y nằm trên đoạn  k 2 ; k 2  k  thì ta được


 

1 1 1
x  y  k2  k  k2  k2  k   k2  k   k 
4 2 2

 
 k  1
2
 Nếu x và y nằm trên đoạn  k 2  k;  1  thì ta được
 

Tác giả: Nguyễn Công Lợi TÀI LIỆU TOÁN HỌC


14
Website:tailieumontoan.com

 k  1
2
 1  k2  k
 k  1 k 1
2
xy  1  k  k 
2
  .
 k  1 2k 2
2
 1  k2  k

Từ đó ta có điều phải chứng minh.

Ví dụ 19. Cho A là tập hợp gồm 6 phần tử bất kỳ của tập hợp 0;1; 2;...;14 . Chứng minh rằng tồn

tại hai tập hợp con B1 , B2 của tập hợp A( B1 , B2 khác nhau và khác rỗng ) sao cho tổng tất cả các

phần tử của tập hợp B1 bằng tổng tất cả các phần tử của tập hợp B 2 .

Lời giải

Do A là tập hợp có 6 phần tử nên số tập hợp con khác rỗng và khác A của tập hợp A là 26  2  62

Xét tập hợp X là tập hợp con bất kì trong 62 tập hợp con trên và S  X  là tổng các phần tưt của X.

Tập hợp X có nhiều nhất 5 phần tử thuộc tập hợp 0;1; 2;...;14 nên ta có

0  S  X   10  11  12  13  14  60 .

Như vậy với 62 tập hợp con của A như trên thì tồng tại 62 tổng không vượt quả 60.

Theo nguyên lý Dirichlet thì tồn tại hai tổng có giả trị bằng nhau. Điều đó chứng tỏ tồn tại hai tập

hợp con

B1 , B2 của tập hợp A có tổng các phần tử của chúng bằng nhau.

Ví dụ 20. Trong hình vuông cạnh bằng 1 ta đặt 51 điểm phân biệt bất kì. Chứng minh rằng

1
có ít nhất 3 trong số 51 điểm đó nằm trong một hình tròn bán kính .
7

Lời giải

1
Chia hình vuông đã cho th|nh 25 hình vuông con bằng nhau có cạnh bằng . Theo nguyên lý
5
Dirichlet thì tồn tại ít nhất một hình vuông nhỏ chứa ít nhất ba điểm trong số 51 điểm đó. Ta kí

2
hiệu hình vuông đó l| C. Khi đó hình vuông nhỏ C có đường chéo là . Đường tròn ngoại tiếp
5

Tác giả: Nguyễn Công Lợi TÀI LIỆU TOÁN HỌC


15
Website:tailieumontoan.com

1 2 1 1
 
hình vuông nhỏ C có bán kính .
2 5
  . Vậy ba điểm nói trên nằm trong hình tròn
5 2 7
1
đồng tâm với đường tròn ngoại tiếp hình vuông nhỏ đó có b{n kính .
7

Tổng quát hóa bài toán: Dựa vào bài giải bài toán trên ta có thể tổng quát hóa bài toán trên với a là

kích thước của cạnh hình vuông, m là số điểm đặt bất kì, phân biệt. Chứng minh rằng có ít nhất n

a2
trong số m điểm đó nằm trong một hình trong bán kính . (Trong đó kí hiệu [a] là phần
 m 
2.  
 n  1
nguyên của a).

Lời giải

 m  a2
Chia hình vuông đã cho th|nh   hình vuông con bằng nhau có cạnh bằng . Theo
 n  1  m 
 n  1
 
nguyên lí Dirichlet thì tồn tại ít nhất một hình vuông nhỏ có chứa ít nhất n điểm trong số m điểm

 
đó. Ta kí hiệu đó l| hình vuông C. Đường tròn ngoại tiếp C có bán kính

a2 a2
 . Vậy n điểm trên nằm trong hình tròn đồng tâm với đường tròn C có  
 m   m 
2.   2.  
 n  1  n  1

a2
bán kính .
 m 
2.  
 n  1

Ví dụ 21. Trong hình tròn đường kính bằng 5 có 10 điểm. Chứng minh rằng tồn tại ít nhất hai

điểm mà khoảng cách giữa chúng bé hơn hoặc bằng 2.

Lời giải

Tác giả: Nguyễn Công Lợi TÀI LIỆU TOÁN HỌC


16
Website:tailieumontoan.com
Thật vậy, trong đường tròn t}m O đường kính
D
5, vẽ đường tròn đồng t}m v| đường kính 2. IX II

Chia hình tròn đã cho th|nh 9 phần(xem hình C


III
vẽ) đường tròn đường kính 2 và 8 phần bằng VIII

O B
1 A
nhau II, III, <, IX m| mỗi phần là hình VII I IV
8
v|nh khăn. Rõ ràng phần I có đường kình VI V
bằng 2. Xét chẳng hạn hình III, ta kí hiệu là

1
ABCD (có là hình v|nh khăn). Ta hãy tính
8
đường kính của nó. Có thể thấy ngay đường

kính của III là d  AC  BD .

Vì DOA  45o , nên d2  DA2  DO2  AO2  2DO.OA.cos450

2
5 5 2 24 5 2 29 5
Hay ta được d2     12  2 .1.   1 . Tù đó suy ra d2   .1, 4 nên d  2 .
2 2 2 4 2 4 2

Theo nguyên lí Dirichlet tồn tại ít nhất hai điểm rơi v|o một trong các miền I, II, III, < , IX có

đường kính bằng 2, còn các miền II, <, IX có đường kính bằng nhau và bằng d  2 , từ đó suy ra

tồn tại hai trong số 10 điểm đã cho m| khoảng cách giữa chúng nhỏ hơn hoặc bằng 2. Đó chính l|

điều cần chứng minh.

Ví dụ 22. Trên mặt phẳng cho 25 điểm. Biết rằng trong ba điểm bất kì trong số đó luôn luôn tồn tại

hai điểm cách nhau nhỏ hơn 1. Chứng minh rằng tồn tại hình tròn bán kính 1 chứa không ít hơn 13

điểm đã cho.

Lời giải

Lấy A là một trong số 25 điểm đã cho. Xét hình tròn O1  A;1 có tâm A bán kính 1. Chỉ có hai khả

năng sau có thể xảy ra như sau:

+ Thứ nhất: Nếu tất cả c{c điểm đã cho nằm trong O1  A;1 thì kết luận của bài toán hiển nhiên

đúng.

Tác giả: Nguyễn Công Lợi TÀI LIỆU TOÁN HỌC


17
Website:tailieumontoan.com
+ Thứ hai: Tồn tại điểm B kh{c điểm A và B thuộc trong số 25 điểm đã cho, sao cho B không nằm

trong đường tròn O1  A;1 , khi đó ta có AB  1 .

Xét hình tròn O2  B;1 có tâm B bán kính 1. Lấy C l| điểm bất kì trong số 25 điểm đã cho sao cho

C khác A và khác B. Theo giả thiết và AB  1 ta có Min CA; CB  1 .

Vì thế C thuộc đường tròn O1  A;1 hoặc C thuộc đường tròn O2 B;1 .  

 
Điều này chứng tỏ rằng các hình tròn O1  A;1 và O2 B;1 chứa tất cả 25 điểm đã cho. Vì thế

theo nguyên lí Dirichlet thì ít nhất 1 trong hai hình tròn trên chứa 13 điểm đã cho. Đó l| điều phải

chứng minh.

Bài toán tổng quát: Cho 2n  1 điểm trên mặt phẳng với n  3 . Biết rằng trong ba điểm bất kì

trong số đó luôn luôn tồn tại hai điểm cách nhau nhỏ hơn 1. Khi đó tồn tại hình tròn bán kính 1

chứa không ít hơn n  1 điểm đã cho.

Ví dụ 23. Tìm hình vuông có kích thước bé nhất, để trong hình vuông đó có thể sắp xếp năm hình

tròn bán kính bằng 1 sao cho không có hai hình tròn n|o trong chúng có điểm chung.

Lời giải

Giả sử hình vuông ABCD có tâm O A B

và cạnh a, chứa năm hình tròn không cắt


O' A' B'
O''
nhau v| đều có bán kính bằng 1. Vì cả năm

hình tròn n|y đểu nằm trọn trong hình


O
O'
vuông, nên các tâm của chúng nằm trong
D' C'
hình vuông A’B’C’D’ có tâm O và cạnh
O''
C
a  2 , ở đ}y A’B’//AB. C{c đường thẳng D

nối c{c trung điểm cùa các cạnh đối diện

của hình vuông A’B’C’D’ chưa A’B’C’D’

thành 4 hình vuông nhỏ.

Theo nguyên lí Dirichlet tồn tại một trong 4 hình vuông nhỏ mà trong hình vuông này chứa ít nhất

hai trong số 5 tâm hình tròn nói trên(không mất tính tổng quát ta giả sử l| O’ v| O‛).

Để ý rằng vì không có hai hình tròn nào(trong số năm hình tròn) cắt nhau nên O'O''  2

Tác giả: Nguyễn Công Lợi TÀI LIỆU TOÁN HỌC


18
Website:tailieumontoan.com
Mặt kh{c do O’ v| O‛ cùng nằm trong một hình vuông nhỏ(cạnh của hình vuông nhỏ đó bằng

a2 a2 a2


) nên ta lại có O' O''  . 2 . Từ đó ta suy ra được . 2  2 a  2 2 2.
2 2 2

Vậy mọi hình vuông cạnh a thỏa mãn yêu cầu đề b|i, ta đều có a  2 2  2 . Bây giờ xét hình

vuông ABCD có a  2 2  2 . Xét năm hình tròn có t}m l| O, A’, B’, C’, D’ (xem hình vẽ) , thì mọi

yêu cầu của đề bài thỏa mãn. Tóm lại, hình vuông có kích thước bé nhất cần tìm là hình vuông với

cạnh a  2 2  2 .

Ví dụ 24. Chứng minh rằng trong một hình tròn bán kính 1, không thể chọn được qu{ 5 điểm mà

khoảng cách giữa hai điểm tùy ý trong chúng đều lớn hơn 1.

Lời giải

Chia hình tròn thành 6 hình quạt bằng nhau (tâm

các hình quạt đều tại t}m O đã cho). Ta biết rằng


A2
khoảng cách giữa hai điểm bất kì trong một hình
A1 A3
quạt nhỏ hơn hoặc bằng 1, vì thế từ giả thiết suy ra
O
tại mỗi hình quạt có không qu{ 1 điểm rơi v|o. Giả
A6 A4
thiết phản chứng chọn được qu{ năm điểm thỏa
A5
mãn yêu cầu đề bài. Vì lí do trên nên số điểm không

thể quá 7(vì nếu số điểm chọn được mà lớn hơn

hoặc bằng 7 thì theo nguyên lí Dirichlet có ít nhất

hai điểm được chọn nằm trong một cung hình quạt,

m| điều này mâu thuẫn với nhận xét trên.).

Vậy từ giả thiết phản chứng suy ra tồn tại s{u điểm A1 ,A2 ,A3 ,A4 ,A5 ,A6 và mỗi điểm nằm

trong một hình quạt sao cho khoảng cách giữa hai điểm tùy ý trong chúng đều lớn hơn 1.

Do A1OA2  A2 OA3  A3OA4  A4OA5  A5OA6  A1OA6  360 0 .

3600
Khi đó suy ra MinAi OAi 1   600 ,i  1,6 (ở đ}y đặt A7  A1 ).
6

Xét tam giác Ak OAk1 với k  1; 2; 3; 4; 5; 6 và A7  A1 , sao cho MinAi OAi 1  Ak OAk 1

Tác giả: Nguyễn Công Lợi TÀI LIỆU TOÁN HỌC


19
Website:tailieumontoan.com

Khi đó Ak OAk 1  600 .

Vì OAk  1; OAk1  1; Ak OAk 1  600 nên từ đó suy ra Ak OAk1  Max Ak Ak1O; OAk Ak 1 .  
Từ đó theo mối liên hệ giữa cạnh và góc trong tam giác Ak OAk1 , thì

Ak Ak1  max OAk ,OAk1   1 .

Điều này mâu thuẫn với Ak Ak1  1 (vì hệ s{u điểm A1 ,A2 ,A 3 ,A 4 ,A 5 ,A 6 thỏa mãn yêu cầu đề

bài). Từ đó ta thấy giả thiết phản chứng l| sai. Điều đó có nghĩa l| không thể chọn qu{ 5 điểm thỏa

mãn yêu cầu để bài.

Ví dụ 25. Cho 1000 điểm M1 ,M2 ,...,M 1000 trên mặt phẳng. Vẽ một đường tròn bán kính bằng 1

tùy ý. Chứng minh rằng tồn tại điểm S trên đường tròn sao cho SM1  SM2  ...  SM1000  1000 .

Lời giải

Xét một đường kính S1S 2 tùy ý của đường tròn bán
M2
M1
kính bằng 1, ở đ}y S1 ,S 2 l| hai đầu của đường kính.

Khi đó ta có S1S 2  2 , nên ta có


S1 S2
O
 S 1 M 1  S 2 M 1  S 1S 2  2

 S 1M 2  S 2 M 2  S 1S 2  2

 ...
S1M1000  S 2 M1000  S1S 2  2 M1000

Cộng từng vế của 1000 bất đẳng thức trên ta có:

S M
1 1
 S1M2  ...  S1M1000    S2M1  S2M2  ...  S 2M1000   2000

Từ đó theo nguyên lí Dirichlet suy ra trong hai tổng

S M 1 1
 S1M2  ...  S1M1000  ;  S 2M1  S 2M2  ...  S 2M1000 

có ít nhất một tổng lớn hơn hoặc bẳng 1000.

Giả sử S1M1  S1M2  ...  S1M1000  1000. khi đó lấy S  S1 . Đó l| điều phải chứng minh.

Tác giả: Nguyễn Công Lợi TÀI LIỆU TOÁN HỌC


20
Website:tailieumontoan.com
Bài toán tổng quát: Cho n điểm M1 ,M2 ,...,Mn trên mặt phẳng. Vẽ một đường tròn bán kính bằng

1 tùy ý. Chứng minh rằng tồn tại điểm S trên đường tròn sao cho SM1  SM2  ...  SMn  n .

Ví dụ 26. Cho chín đường thẳng cùng có tính chất là mỗi đường thẳng chia hình vuông thành hai

2
tứ giác có tỉ số diện tích bằng . Chứng minh rằng có ít nhất ba đường thẳng trong số đó cùng đi
3
qua một điểm.

Lời giải

B M C B P C

J3

E F E F
J J1 J2
J4

A N D A Q D

C{c đường thẳng đã cho không thể cắt các cạnh kề nhau của hình vuông, bởi vì nếu thế

chúng chia hình vuông thành một tam gi{c v| ngũ gi{c ( chứ không phải là chia hình vuông thành

hai tứ giác). Vì lẽ đó, mọi đường thẳng ( trong số chín đường thẳng) đều cắt hai cạnh đối của hình

vuông v| dĩ nhiên không đi qua một đỉnh nào của hình vuông cả.

Giả sử một đường thẳng cắt hai cạnh đối BC và AD tại c{c điểm M và N.

.AB.  BM  AN 
1
S ABMN 2 2 EJ 2
Ta có   2    , ở đ}y E v| F l| c{c trung điểm của AB và
.CD.  MC  ND 
S MCDN 3 1 3 JF 3
2
CD tương ứng.

Gọi E, F, P, Q tương ứng l| c{c trung điểm của AB, CD, BC, AD. Gọi J1 , J 2 , J 3 , J 4 l| c{c điểm sao

EJ1 FJ 2 PJ 3 QJ 4 2
cho J1 , J 2 nằm trên EF và J 3 , J 4 nằm trên PQ và thỏa mãn     .
J 1F J 2 F J 3 Q J 4 P 3

Khi đó từ đó lập luận trên ta suy ra mỗi đường thẳng có tính chất thỏa mãn yêu cầu của đề bài

phải đi qua một trong 4 điểm J1 , J 2 , J 3 , J 4 nói trên. Vì có chín đường thẳng, nên theo nguyên lí

Tác giả: Nguyễn Công Lợi TÀI LIỆU TOÁN HỌC


21
Website:tailieumontoan.com
dirichlet phải tồn tại ít nhất một trong 4 điểm J1 , J 2 , J 3 , J 4 sao cho nó có ít nhất ba trong 9 đường

thẳng đã cho đi qua. Vậy có ít nhất 3 đường thẳng trong 9 đường thẳng đã cho đi qua một điểm.

Bài toán tổng quát 1: Cho 4n  1  n  2  đường thẳng cùng có tính chất là mỗi đường thẳng chia

2
hình vuông thành hai tứ giác có tỉ số diện tích bằng . Chứng minh rằng có ít nhất n  1 đường
3
thẳng trong số đó cùng đi qua một điểm.

Bài toán tổng quát 2: Cho 4n  r  n  2,r  1 đường thẳng cùng có tính chất là mỗi đường thẳng

2
chia hình Chữ nhật thành hai tứ giác có tỉ số diện tích bằng . Chứng minh rằng có ít nhất n  1
3
đường thẳng trong số đó cùng đi qua một điểm.

Ví dụ 27. Giả sử mỗi điểm trong mặt phẳng được tô bằng một trong 2 m|u đen v| trắng. Chứng

minh tồn tại một hình chữ nhật có c{c đỉnh cùng màu.

Lời giải

Giả sử ta có một lưới ô vuông tạo bởi 3 đường nằm ngang v| 9 đường thẳng đứng, mỗi nút lưới

được tô bởi một màu trắng hoặc đen.

A X

B
Y

C Z

Xét 3 nút lưới của một đường dọc, mỗi nút có hai cách tô màu nên mỗi bộ ba nút trên đường dọc

ấy có 2.2.2  8 c{ch tô m|u. Có 9 đường dọc, mỗi đường có 8 cách tô màu nên theo nguyên lý

Dirichlet tồn tại hai đường có c{ch tô m|u như nhau. Chẳng hạn hai bộ ba điểm đó l| A, B, C và X,

Y, Z.

Vì 3 điểm A, B, C chỉ được tô bởi hai màu nên tồn tại hai điểm cùng màu, chẳng hạn B v| C khi đó

hình chữ nhật BYZC có 4 đỉnh cùng một màu.

Ví dụ 28. Trong mặt phẳng cho 6 điểm, trong đó không có ba điểm nào thẳng hàng. Mỗi đoạn

thẳng nối từng cặp điểm được tô m|u đỏ hoặc xanh. Chứng minh rằng tồn tại ba điểm trong số

Tác giả: Nguyễn Công Lợi TÀI LIỆU TOÁN HỌC


22
Website:tailieumontoan.com
s{u điểm đã cho, sao cho chúng l| c{c đỉnh của một tam giác mà các cạnh của nó được tô cùng một

màu.

Lời giải

Xét A là một trong số 6 điểm đã cho. Khi đó xét B


B'
năm đoạn thẳng(mỗi đoạn thẳng nối điểm A với năm

điểm còn lại). Vì mỗi đoạn thẳng được tô chỉ m|u đỏ

hoặc màu xanh, nên theo nguyên lí Dirichlet có ít nhất

ba trong năm đoạn nói trên cùng màu. Giả sử đó là


A B''
c{c đoạn AB, AB’ v| AB‛ v| có thể cho rằng chúng

cùng màu xanh. Chỉ có hai trường hợp sau xảy ra:

 Trường hợp 1: Nếu ít nhất một trong ba đoạn BB’, B’B‛, B‛B m|u xanh, thì tồn tại một tam giác
với ba cạnh xanh và kết luận của b|i to{n đúng trong trường hợp này.

 Trường hợp 2: Nếu không phải như vậy, tức l| BB’, B’B‛, B‛B m|u đỏ , thì ba điểm phải tìm là B,
B’,B‛ vì BB’B‛ l| tam gi{c có ba cạnh m|u đỏ. Đpcm.

Ví dụ 29. Trên mặt phẳng cho 18 điểm, sao cho không có ba điểm nào thẳng hàng. Nối từng cặp

điểm với nhau và tô màu cho mọi đoạn thẳng thu được một trong hai m|u xanh v| đỏ. Chứng

minh rằng luôn tìm được một tứ gi{c m| c{c đỉnh của nó nằm trong tập điểm đã cho sao cho cạnh

v| đường chéo của nó cùng màu.

Lời giải

 
Giả sử Ai i  1,18 l| 18 điểm đã cho. Xuất A2 A3


phát từ A1 có 17 đoạn thẳng A1Ai i  2,18 .  A4

Mười bảy đoạn thẳng đó chỉ có hai màu xanh


A5
hoặc đỏ, nên theo nguyên lí Dirichlet tồn tại ít
A6
nhất chín đoạn thằng cùng màu.
A1
A7
A8
Không giảm tính tổng quát giả sử đó l| c{c

đoạn thẳng A1A2 ,A1A3 ,...,A1A10 và chúng cùng A9

m|u đỏ. Xét chín điểm A2 ,A2 ,...,A10 chỉ có thể

Tác giả: Nguyễn Công Lợi TÀI LIỆU TOÁN HỌC


23
Website:tailieumontoan.com
xảy ra hai trường hợp sau:

 Trường hợp 1: Hoặc là tồn tại điểm A j  2  j  10  sao cho trong t{m đoạn thẳng

A jAk  2  k  10,k  j có ít nhất bốn đoạn m|u đỏ. Không mất tính tổng quát có thể cho là

A2 A3 ,A2 A4 ,A2 A5 ,...,A2 A6 m|u đỏ. Đến đ}y lại chỉ còn hai khả năng:

+ Hoặc là mọi đoạn thẳng A3 A4 ,A3 A5 ,A3 A6 ,A4 A5 ,A4 A6 ,A5 A6 đều m|u xanh. Khi đó

A3 A4 A5 A6 là tứ giác xanh thỏa mãn yêu cầu.

 
+ Tồn tại một đoạn thẳng Ai , A j  3  i  j  6  m|u đỏ. Khi đó A1A2 Ai A j 3  i  j  6 là tứ giác

đỏ thỏa mãn yêu cầu bài toán.

 Trường hợp 2: Hoặc là với mọi điểm A j  2  j  10  , thì trong t{m đoạn thẳng

A jAk  2  k  10,k  j có tối đa ba đoạn m|u đỏ m| thôi. Khi đó phải tồn tại một điểm (chẳng

hạn A 2 ) m| trong c{c đoạn A2 Ak  3  k  10,k  j có tối đa hai đoạn m|u đỏ thôi (thật vậy, nếu

với mọi A j  2  j  10  m| có đúng ba đoạn A jAk  2  k  10,k  j m|u đỏ, thì số đoạn thẳng

là số nguyên. Vô lí. Vì A2 Ak  3  k  10,k  j có tối đa


9.3
m|u đỏ nối trong nội bộ 9 điểm đó l|
2
hai đoạn m|u đỏ mà thôi, nên trong số c{c đoạn A2 A3 , A2 A4 , A2 A5 ,..., A2 A10 có ít nhất s{u đoạn

màu xanh. Không mất tính tổng quát ta cho A2 A5 ,A2 A6 ,...,A2 A10 màu xanh.

Xét s{u điểm A5 ,A6 ,A7 ,A8 ,A9 ,A10 . Đó l| s{u điểm m| trong đó không có ba điểm nào thẳng

hàng, và mỗi đoạn thẳng nối hai điểm chỉ có hai màu xanh hoặc đỏ. Theo bài 19 thì luôn luôn tồn

tại ít nhất một tam gi{c m| ba đỉnh chọn trong A5 ,A6 ,A7 ,A8 ,A9 ,A10  sao cho ba cạnh cùng

màu. Lại có hai khả năng:

+ Giả sử tồn tại tam giác Ai ,A j ,Ak  5  i  j  k  10  m|u xanh. Khi đó tứ giác A2 Ai A j Ak với

5  i  j  k  10 là tứ giác xanh thỏa mãn yêu cầu đề bài.

+ Nếu tồn tại tam giác Ai ,A j ,Ak  5  i  j  k  10  m|u đỏ, thì A1Ai A j Ak là tứ giác cần tìm.

Như vậy ta luôn chứng mình được tồn tại một tứ gi{c m| c{c đỉnh của nó nằm trong t}m điểm đã

cho sao cho cạnh v| đường chéo cùng màu.

Tác giả: Nguyễn Công Lợi TÀI LIỆU TOÁN HỌC


24
Website:tailieumontoan.com
Ví dụ 30. Cho 6 điểm trong mặt phẳng sao cho bất kì ba điểm n|o cũng l| đỉnh của một tam giác

có các cạnh chiều dài khác nhau. Chứng minh rằng tồn tại một cạnh là cạnh nhỏ nhất của một tam

giác vừa là cạnh lớn nhất của một tam giác khác.

Lời giải

Trong mỗi ta gi{c ta tô m|u đỏ cạnh nhỏ nhất của tam giác và tô màu xanh hai cạnh kia. Ta

cần chứng minh tồn tại một tam giác có các cạnh cùng m|u đỏ. Gọi s{u điểm đã cho l| A, B, C, D,

E, F.

Từ điểm A trong s{u điểm đã cho ta nối với năm điểm còn lại, khi đó ta được cạnh cạnh.

Theo nguyên lí Dirichlet thì trong năm cạnh này ít nhất có cạnh cạnh cùng màu. Không mất tính

tổng quát ta giả sử ba cạnh đó l| AB, AC, AD. Khi đó ta có c{c trường hợp sau:

 Nếu AB, AC, AD cùng m|u đỏ. Khi đó nếu tam giác BCD có các cạnh cùng m|u đỏ, giả sử đó l|
BC thì ta có tam giác ABC có các cạnh cùng m|u đỏ.

 Nếu AB, AC, AD cùng m|u xanh. Khi đó nếu tam giác BCD có các cạnh cùng m|u đỏ thì cạnh
lớn nhất của tam giác này là cạnh cần tìm.

Ví dụ 31. Trong một cuộc họp có 6 đại biểu. Người ta nhận thấy cứ ba người bất kỳ thì có hai

người quen nhau. Chứng minh rằng có ba người đôi một quen nhau.

Lời giải

C{c đại biểu tương ứng với 5 điểm A, B, C, D, E, F. Hai đại biểu X v| Y n|o đó m| quen

nhau thì ta tô đoạn thẳng XY bằng màu xanh còn nếu X v{ Y không quen nhau thì tô đoạn XY màu

đỏ. Xét 5 đoạn thẳng AB, AC, AD, AE, AF. Theo nguyên lí Dirichlet thì tồn tại ba đoạn cùng màu.

 Nếu AB, AC, AD m|u xanh. Xét ba điểm B, C, D. Vì 3 đại biểu n|o cũng có hai người quen nhau
suy ra một trong ba đoạn BC, CD, DB màu xanh. Giả sử BC m|u xanh, khi đó ta có ba đoạn thẳng

AB, BC, CA có m|u xanh, do đó A, B, C đôi một quen nhau.

 Còn nếu AB, AC, AD m|u đỏ, khi đó với ba điểm A, B, C thì ta có đoạn BC màu xanh, với ba
điểm A, C, D thì ta có đoạn CD màu xanh và với ba điểm A, B, D thì ta có đoạn BD m|u xanh. Như

vậy ba đoạn thẳng BC, CD, CB có m|u xanh nên B, C, D đôi một quen nhau.

Vậy b|i to{n được chứng minh.

Tác giả: Nguyễn Công Lợi TÀI LIỆU TOÁN HỌC


25
Website:tailieumontoan.com
Ví dụ 32. Chứng minh rằng từ sáu số vô tỉ tùy ý có thể chọn ra được ba số (ta gọi ba số đó l| a, b,c)

sao cho a  b, b  c,c  a cũng l| số vô tỉ.

Lời giải

Xét trên mặt phẳng s{u điểm sao cho không có ba điểm nào thẳng hàng. Với mỗi điểm ta sẽ

gắn cho nó một số vô tỉ. Như vậy s{u điểm được gắn sáu số vô tỉ đã cho. Hai điểm mang số a và b

sẽ được nối với nhau bằng một đoạn thẳng m|u đỏ nếu a  b là số vô tỉ, còn sẽ có màu xanh khi

a  b là số hữu tỉ.

Theo đề bài tồn tại ít nhất một tam giác cùng màu. Giả sử tam gi{c đó có ba đỉnh được gắn số là a,

b, c.

Chỉ có hai khả năng xảy ra như sau:

+ Nếu tam gi{c đó là tam giác xanh. Khi ấy a  b, b  c,c  a là 3 số hữu tỉ.

Lúc này  a  b    b  c    c  a   2b cũng l| một số hữu tỉ. Điều này vô lí vì b là số vô tỉ.

+ Nếu tam gi{c đỏ l| tam gi{c đỏ. Khi ấy a  b, b  c,c  a là 3 số vô tỉ. Khi đó ta có điều phải

chứng minh.

Ví dụ 33. Cho mỗi điểm trên mặt phẳng được tô bằng một trong hai m|u xanh, đỏ. Chứng minh

rằng tồn tại một tam gi{c m| ba đỉnh và trọng tâm cùng màu.

Lời giải

Lấy năm điểm tùy ý sao cho không có A'

ba điểm nào thẳng hàng trên mặt phẳng. Khi

đó vì chỉ dùng có hai m|u để tô c{c đỉnh, mà


A
theo nguyên lí Dirichlet phải tồn tại ba điểm
P N

trong số đó cùng m|u. Giả sử đó l| ba điểm G


B M C
A, B, C có m|u đỏ. Như vậy ta có tam giác
B' C'
ABC với ba đỉnh m|u đỏ. Gọi G là trọng tâm

tam giác ABC. Chỉ có hai khả năng xảy ra:

+ Nếu G có m|u đỏ. Khi đó A, B, C, G cùng

đỏ v| b|i to{n đã được giải.

Tác giả: Nguyễn Công Lợi TÀI LIỆU TOÁN HỌC


26
Website:tailieumontoan.com
+ Nếu G có m|u xanh. Kéo d|i GA, GB, GC c{c đoạn AA’  3GA, BB’  3GB, CC’  3GC . Khi

đó gọi M, N, P tương ứng l| c{c trung điểm của BC, CA, AB thì

A’A  3AG  6GM  A’A  2AM.

Tương tự B’B  2BN, CC’  2CP . Do đó c{c tam gi{c A’BC, B’AC, C’AB tương ứng nhận A, B, C

là trọng tâm. Mặt kh{c, ta cũng có c{c tam gi{c ABC v| A’B’C’ có cùng trọng t}m G. Có hai trường

hợp sau có thể xảy ra:

 Nếu A’, B’, C’ cùng xanh. Khi đó tam gi{c A’B’C’ v| trọng tâm G có cùng màu xanh.

 Nếu ít nhất một trong c{c điểm A’, B’, C’ có m|u đỏ. Không mất tính tổng quát giả sử A’ đỏ. Khi
đo tam gi{c A’BC v| trọng t}m A m|u đỏ.

Vậy trong mọi khả năng luôn tồn tại một tam gi{c m| ba đỉnh và trọng tâm cùng màu.

Ví dụ 34. Để khuyến khích phong tr|o học tập , một trường THCS đã tổ chức 8 đợt thi cho

c{c học sinh. Ở mỗi đợt thi , có đúng 3 học sinh được chọn để trao giải . Sau khi tổ chức

xong 8 đợt thi , người ta nhận thấy rằng với 2 đợt thi bất kỳ luôn có đúng 1 học sinh được

trao giải ở cả 2 đợt thi đó. Chứng minh rằng:

a) Có ít nhất 1 học sinh được trao giải ít nhất 4 lần .

b) Có đúng một học sinh được trao giải ở tất cả 8 đợt thi.

Lời giải

Ta biểu diễn mỗi học sinh bằng một điểm trên mặt phẳng sao cho không có ba điểm nào

thẳng hàng. Ở mỗi đợt thi có đúng ba học sinh được trao giải nên ta nối ba điểm biểu thị ba học

sinh bằng một tam giác(không nối hai điểm bất kì), có 8 đợt trao giải nên ta có 8 tam gi{c. Hai đợt

thi bất kì luôn có đúng một học sinh được trao giải ở cả hai đợt tương ứng với hai tam giác bất kì

luôn có một điểm chung.

a) Xét tam giác ABC bất kì trong 8 tam giác trên, vì 7 tam giác mỗi tam gi{c đều có một đỉnh chung

với tam gi{c ABC, theo nguyên lí Dirichlet trong ba điểm A, B, C có ít nhất một điểm l| đỉnh

chung của 4 tam gi{c, điều n|y tương ứng với có ít nhất một học sinh được trao giải ít nhất 4 lần.

b) Không mất tính tổng quát ta giả sử A l| đỉnh chung của 4 tam giác, ta chứng minh tất cả các tam

gi{c đều nhận A l|m đỉnh chung.

Tác giả: Nguyễn Công Lợi TÀI LIỆU TOÁN HỌC


27
Website:tailieumontoan.com
Xét tam giác DEF bất kì, nếu tam giác này trùng với A thì tam giác này sẽ có đỉnh chung với bốn

tam gi{c m| đã có đỉnh chung l| A, điều này là vô lí vì hai tam giác bất kì chỉ có một điểm chung.

Vậy cả 8 tam gi{c đều có đỉnh chung l| A, điều n|y tương ứng với có dúng một học sinh được trao

giải trong cả tám lần.

Ví dụ 35. Cho điểm M  x; y  trên mặt phẳng tọa độ được gọi l| điểm nguyên nếu cả x v| y đều là

các số nguyên. Tìm số nguyên dương bé nhất sao cho từ mỗi bộ n điểm nguyên đều tìm được bộ

ba điểm nguyên l| đỉnh của một tam giác có diện tích nguyên (trong trường hợp ba điểm thẳng

hàng thì coi diện tích tam giác bằng 0)

Lời giải

Xét tam giác ABC với tọa độ A  x1 ; y1  , B  x2 ; y 2  , C  x 3 ; y 3 

 x  x1  y 2  y1    x3  x1  y 3  y1 
1
Khi đó ta được S ABC 
2 3

Xét tam giác bất kì có tọa độ c{c đỉnh l| c{c điểm nguyên, khi đó luôn tồn tại có cạnh song song

với các trục tọa độ thỏa mãn một đỉnh của hình chữ nhật tròng với một đỉnh của tam giác và hai

đỉnh còn lại nằm trên hai cạnh của hình chữ nhật hoặc trùng với hai đỉnh của hình chữ nhật

 x1  x 3  x 2
Xét hình bên, khi đó ta được 
 y1  y 2  y 3

Ta có SAPQR   x2  x1  y3  y1  và SABC  SAPQR  SAPC  S BCQ  SABR

 x2  x1  y 3  y1  , S BCQ   x2  x3  y 3  y 2  , S ABR   x 2  x 3  y 2  y 3  


1 1 1
Với S APC 
2 2 2

S ABC  S APQR  S APC  S BCQ  S ABR

  x 2  x1  y 3  y1  
x 2
 x1  y 3  y1    x 2  x 3  y 3  y 2    x 2  x 3  y 2  y 3 
2
  x 2  x1  y 3  y1    x 3  x1  y 2  y1  
1
2

 x  x1  y 3  y1    x3  x1  y 2  y1 
1
Do đó ta được S ABC 
2 2

Với n  2 , không tồn tại tam giác

Tác giả: Nguyễn Công Lợi TÀI LIỆU TOÁN HỌC


28
Website:tailieumontoan.com

Với n  3 , chọn A  1; 0  , B  0;1 , C  0; 0 ta được S ABC 


1
(loại)
2

Với n  4 , chọn A  1; 0  , B  1;1 , C  0;1 , D 0; 0 ta được S 


1
với S là diện tích một tam giác
2
bất kì(loại)

 
Với n  5 , ta có với mỗi điểm M x; y tồn tại một trong bốn dạng x và y cùng chẵn, x và y cùng

le, x lẻ và y chẵn, x chẵn và y lẻ. Với 5 điểm như trên theo nguyên lí Dirichlet luôn tồn tại hai đểm

cùng dạng. Không mất tính tổng quát ta giả sử đó l| hai điểm A và B

 x  x1  y 3  y1    x3  x1  y 2  y1 
1
Khi đó S ABC 
2 2

Vì A và B cùng dạng nên x2  x1 2; y2  y1 2

 x  x1  y 3  y1    x3  x1  y 2  y1  là số nguyên
1
Do đó S ABC 
2 2

Vậy số nguyên dương nhỏ nhất thỏa mãn yêu cầu bài toán là 5.

Ví dụ 36. Cho tam giác nhọn ABC có BAC  60o , BC  2 3 cm. Bên trong tam giác này cho 13

điểm bất kỳ. Chứng minh rằng trong 13 điểm ấy luôn tìm được 2 điểm mà khoảng cách giữa

chúng không lớn hơn 1cm.

Lời giải

Gọi (O) l| đường tròn ngoại tiếp tam giác ABC và M, A

N, P lần lượt l| trung điểm của BC, CA, AB. Do tam giác ABC
E
F I

nhọn nên O nằm trong tam giác ABC. Vì BAC  600 nên N
P
MC H
MOC  600 , suy ra OA  OB  OC   2 . Vì O nằm G

sin 600 O

trong tam giác ABC và OM  BC, ON  AC, OP  AB . C


B M

Suy ra tam gi{c ABC được chia thành 3 tứ giác ANOP, BMOP,

CMON nội tiếp c{c đường tròn có đường kính 2 (đường kính

lần lượt là OA, OB, OC).

Theo nguyên lý Dirichlet, tồn tại ít nhất một trong 3 tứ giác này chứa ít nhất 5 điểm trong 13 điểm

đã cho, giả sử đó l| tứ giác ANOP.


Tác giả: Nguyễn Công Lợi TÀI LIỆU TOÁN HỌC
29
Website:tailieumontoan.com
Gọi E, F, G, H lần lượt l| trung điểm của NA, AP, PO, ON v| I l| trung điểm OA, suy ra

IA  IP  IO  IN  1 .

Khi đó tứ gi{c ANOP được chia thành 4 tứ giác AEIF, FIGP, IGOH, IHNE nội tiếp các

đường tròn có đường kính 1.

Theo nguyên lý Đirichlê, tồn tại ít nhất một trong 4 tứ giác này chứa ít nhất 2 điểm trong 5

điểm đã cho, giả sử đó l| tứ giác AEIF chứa 2 điểm X, Y trong số 13 điểm đã cho.

Vì X, Y nằm trong tứ giác AEIF nên X, Y nằm trong đường tròn ngoại tiếp tứ giác này, do

đó XY không lớn hơn đường kính đường tròn n|y, nghĩa l| khoảng cách giữa X, Y không vượt quá

1.

Ví dụ 36. Cho hình chữ nhật ABCD có AB  3, BC  4.

a) Chứng minh rằng từ 7 điểm bất kì nằm trong hình chữ nhật luôn tìm được hai điểm mà

khoảng các của chúng không lớn hơn 5.

b) Chứng minh rằng khẳng định ở câu a vẫn đúng nếu có đúng 6 điểm nằm trong hình chữ

nhật ABCD.

Lời giải

a) Chia hình chữ nhật ABCD thành 6 hình chữ nhật có A B

kích thức 1x2. Vì 7 chia 6 dư 1 nên theo nguyên lí

Dirichlet thì tồn tại hai điểm cùng nằm trong một

hình chữ nhật. Gọi hai điểm đó l| A’ v| B’ khi đó ta


D C
được A' B'  12  22  5 . Do đó ta có khoảng cách

giữa hai điểm A v| B không vượt quá 5.


A B

b) Chia hình chữ nhật th|nh 5 hình đa gi{c như hình

vẽ. Vì 6 chia 5 dư 1 nên theo nguyên lí Dirichlet luôn

tồn tại hai điểm nằm trong cùng một đa gi{c. Gọi hai
D C
điểm đó l| M v| N. Ta dễ dàng chứng minh được

MN  5 . Do đó ta có điều phải chứng minh.

Tác giả: Nguyễn Công Lợi TÀI LIỆU TOÁN HỌC


30
Website:tailieumontoan.com
Ví dụ 67. Cho 13 điểm phân biệt nằm trong hay trên cạnh của một tam gi{c đều có cạnh bằng 6cm.

Chứng minh răng luôn tồn tại hai điểm trong số 13 điểm đã cho m| khoảng cách giữa chúng

không vượt quá 3 cm .

Lời giải

Chia tam gi{c đều ABC cạnh 6cm thành bốn tam A
gi{c đều cạnh 3cm.Theo nguyên lí Dirichlet thì có ít nhất
P O M
bốn điểm thuộc cùng một tam gi{c đều canh 3cm. Giả sử

có 4 điểm thuộc tam gi{c đều ADE cạnh 3cm. D E


N
Chia tam gi{c đều ADE cạnh 3cm thành ba phần

như hình vẽ(với M, N, P lần lượt l| trung điểm của AE,


B F C
ED, DA và O là trọng t}m tam gi{c ADE). Khi đó mỗi

phần của tam giác ADE là một tứ giác nội tiếp đường

tròn đường kính 3 cm .

Theo nguyên lí Dirichlet thì có ít nhất hai điểm thuộc cùng một phần ,hai điểm này có

khoảng c{ch không vượt quá 3 cm . Vậy ta có điều phải chứng minh.

Ví dụ 38. Cho tứ giác ABCD nội tiếp đường tròn bán kính 2cm. Chứng minh rằng trong số 17 điểm

A1 ; A2 ;...; A 17 bất kỳ nằm trong tứ giác ABCD luôn có thể tìm được hai điểm mà khoảng cách

giữa hai điểm đó không lớn hơn 1cm.

Lời giải

Gọi O l| t}m đường tròn ngoại tiếp tứ giác ABCD. B


F
Khi đó ta xét c{c trường hợp sau
C
 Trường hợp 1: Xét trường hợp điểm O nằm trong tứ E
L
giác ABCD K
O G
Gọi E, F, G, H lần lượt là hình chiếu vuông góc A I M
N
của O trên AB, BC, CD và DA. H

Khi đó tứ giác AEOH, BEFO, CFOG và DGOH là D

bốn tứ gi{c n|y đều là tứ giác nội tiếp đường tròn bán

kính 1cm.

Theo nguyên lí Dirichlet thì trong bốn tứ giác trên

Tác giả: Nguyễn Công Lợi TÀI LIỆU TOÁN HỌC


31
Website:tailieumontoan.com
Có một tứ giác chứa ít nhất 5 điểm trong 17 điểm A1 ; A2 ;...; A17 không mất tính tổng quát

khi giả sử năm điểm A1 ; A2 ; A3 ; A4 ; A5 nằm trong tứ giác AEOH.

Gọi I l| t}m đường tròn ngoại tiếp tứ gi{c AEOH thì I l| trung điểm của OA. Gọi K, L, M, N

lần lượt là hình chiếu vuông góc của I trên AE, EO, OH v| HA. Khi đó bốn tứ giác AKIN, ELIK,

OLIM, HMIN đều là tứ giác nội tiếp đường tròn đường kính 1cm.

Theo nguyên lí Dirichlet thì có ít nhất hai điểm trong năm điểm A1 ; A2 ; A3 ; A4 ; A5 nằm

trong tứ giác AKIN hoặc tứ giác ELIK hoặc tứ giác OLIM hoặc tứ gi{c HMIN. Hai điểm này có

khoảng cách không lớn hơn 1cm. Ta có điều phải chứng minh.

 Trường hợp 2: Xét trường hợp điểm O nằm trên cạch hoặc bên ngoài tứ giác ABCD. Khi đó
chứng minh ho|n to|n tương tự như trên ta cũng được điều phải chứng minh.

Ví dụ 39. Trên đường tròn cho 16 điểm được tô bởi một trong ba màu xanh hoặc đỏ hoặc vàng

(mỗi điểm một màu). Mỗi đoạn thẳng nối hai điểm trong 16 điểm trên được tô màu tím hoặc nâu

(mỗi đoạn thẳng một màu). Chứng minh rằng với mọi cách tô màu ta luôn chọn được một tam

gi{c có ba đỉnh cùng màu và ba cạnh cùng màu.

Lời giải

Trên đường tròn 16 điểm tô bởi ba màu xanh hoặc đỏ A

hoặc vàng và do 16  3.5  1 nên theo nguyên lý Dirichlet F

ta có ít nhất 6 điểm cùng màu. E

Giả sử 6 điểm đó l| A, B, C, D, E cùng m|u đỏ như hình vẽ.


B
Nối AB, AC, AD, AE, AF ta được 5 đoạn thẳng tô bởi hai D

màu nên theo nguyên lí Dirichlet thì có ít nhất 3 đoạn thẳng C

cùng màu.

Giả sử AB, AC, AD có cùng m|u n}u. Khi đó ta có c{c

trường hợp sau.

 Trường hợp 1: Nếu một trong ba đoạn BC, BD hoặc CD có màu nâu thì ta có một tam giác có ba
đỉnh m|u đỏ và ba cạnh màu nâu.

 Trường hợp 2: Nếu cả ba đoạn BC, BD v| CD được tô m|u tím thì ta được tam giác BCD có ba
đỉnh m|u đỏ và ba cạnh màu tìm.

Vậy ta có điều phải chứng minh.


Tác giả: Nguyễn Công Lợi TÀI LIỆU TOÁN HỌC
32
Website:tailieumontoan.com

Ví dụ 40. Đặt 28 điểm v|o tam gi{c đều cạnh 6 3 cm. Chứng minh rằng tồn tại 2 điểm trong 28

đã cho khoảng c{ch không vượt quá 2cm.

Lời giải

Chia tam gi{c đều ABC cạnh 6 3 cm thành A

chín tam gi{c đều cạnh 2 3 cm. Theo nguyên lí P O M

Dirichlet thì có ít nhất bốn điểm thuộc cùng một D E


N

tam gi{c đều canh 2 3 cm. Giả sử có 4 điểm thuộc

tam gi{c đều ADE cạnh 2 3 cm. F

Chia tam gi{c đều ADE cạnh 2 3 cm thành ba


B C
phần như hình vẽ(với M, N, P lần lượt là trung

điểm của AE, ED, DA và O là trọng tâm tam giác

ADE). Khi đó mỗi phần của tam giác ADE là một tứ

giác nội tiếp đường tròn đường kính 2 cm .

Theo nguyên lí Dirichlet thì có ít nhất hai điểm thuộc cùng một phần, hai điểm này có

khoảng c{ch không vượt quá 2 cm. Vậy ta có điều phải chứng minh.

Ví dụ 41. Cho 33 điểm hình vuông có cạnh bằng 4, trong đó không có ba điểm nào thẳng hàng. Vẽ

c{c đường tròn có bán kính bằng 2 v| t}m l| c{c điểm đã cho. Hỏi có hay không ba điểm trong
c{c điểm trên nằm trong phần chung của ba đường tròn có t}m chính l| ba điểm trên.

Lời giải

Chia hình vuông đã cho th|nh 16 hình vuông, khi đó mỗi hình vuông nhỏ có cạnh bằng 1.

Do 33  2.16  1 nên theo nguyên lí Dirichlet thì luôn tồn tại ba điểm cùng nằm trong một

hình vuông. Giả sử ba điểm đó l| A, B, C cùng nằm trong hình vuông MNPQ. Ta có đường chéo

MP  2 và mọi điểm E thuộc hình vuông MNPQ đều có 2  MP  AE . Khi đó đường tròn

 A; 2  sẽ phủ kín hình vuông MNPQ. Hoàn to|n tương tự thì các hình vuông  B; 2  và
 C; 2  cũng sẽ phủ kín hình vuông MNPQ. Như vậy cả ba đường tròn trên cùng chứa hình
vuông MNPQ. Từ đó ta được hình vuông MNPQ nằm trong phần chung của ba đường tròn trên.

Tác giả: Nguyễn Công Lợi TÀI LIỆU TOÁN HỌC


33
Website:tailieumontoan.com
M| ba điểm A, B, C cùng nằm trong hình vuông MNPQ nên chúng mằn trong phần chung của ba

 
đường tròn A; 2 , B; 2 và C; 2 .   
Ví dụ 42. Trong một bàn cờ 8x8 ta đ{nh giấu tất cả tâm của các ô. Tồn tại hay không 13 đường

thẳng chia bàn cờ thành các phân sao cho mỗi phần chứa không quá một điểm được đ{nh dấu.

Lời giải

Giả sử tồn tại 13 đường thẳng l1 ; l 2 ;...; l13 chia bàn cờ thành các phần sao cho mỗi phần chứa

không quá một điểm được đ{nh dấu. Gọi tâm của 28 ô biên lần lượt là A1 ; A2 ; A3 ;...; A28 . Và xét

28 đoạn thẳng Ai Ai 1 với i  1; 2;...; 28 , ta quy ước A29  A1 .

Dễ thấy mỗi đường thẳng trong 13 đường thẳng l1 ; l 2 ;...; l13 cắt không quá hai trong 28 đoạn

thẳng Ai Ai 1 nói trên. Như vậy luôn tồn tại một đoạn thẳng tròn 28 đoạn thẳng trên không bị cắt

bởi đường thẳng nào từ c{c đường thẳng đã cho. Giả sử đoạn thẳng đó l| Ai A j , khi đó hai điểm

A i và A j nằm trong cùng một phần. Điều này mâu thuẫn với giả sử ban đầu.

Vậy không tồn tại 13 đường thẳng thỏa mãn yêu cầu bài toán.

Ví dụ 43. Trong mặt phẳng cho n đường thẳng sao cho đôi một không song song với nhau. Chứng

180 0
minh rằng tồn tại góc giữa hai đường thẳng n|o đó không lớn hơn .
n

Lời giải

Cho điểm O tùy ý trong mặt phẳng. Qua O vẽ n đường thẳng d1 ;d2 ;d3 ;...;dn lần lượt song

song với n đường thẳng đã cho (ta luôn l|m được điều n|y do n điểm đã cho đôi một không song

song với nhau)

Gọi  i là góc tạo bởi hai đường thẳng d i và di 1 .

Khi đó ta có 2 1   2   3  ...   n   3600 nên ta được 1   2   3  ...   n  180 0 .

1800
Từ đó theo nguyên lí Dirichlet tồn tại một góc  j  . Từ đó suy ra điều phải chứng minh.
n

Tác giả: Nguyễn Công Lợi TÀI LIỆU TOÁN HỌC


34
Website:tailieumontoan.com
Ví dụ 44. Trong hình vuông có cạnh bằng 1 cho 33 điểm bất kì. Chứng minh rằng trong c{c điểm

1
đã cho bao giờ cũng tìm được ba điểm tạo thành một tam giác có diện tích không vượt quá .
32

Lời giải

Chia mỗi cạnh hình vuông thành bốn A B A B

đoạn thẳng bằng nhau, khi đó ta được 16


G
1 G
hình vuông có diện tích bằng . Do có
16
E K
33 điểm nên theo nguyên lý Dirichlet thì I

tồn tại ba điểm cũng nằm trong một hình


D E H F C D F C
1
vuông nhỏ có diện tích bằng .
16
1
Không mất tính tổng quát ta giả sử ba điểm đó l| G, E, F v| hình vuông có diện tích bằng chứa
16
1
ba điểm đó l| ABCD. Từ đó ta được SGEF  S . Ta xét hai trường hợp sau:
2 ABCD

 Trường hợp 1: Tam giác GEF có một cạnh nằm trên một cạnh của hình vuông ABCD.

1
Giả sử EF nằm trên CD. Kẻ GH vuông góc với CD tại H, ta có S GEF  GH.EF .
2

1 1 1
Mà ta có GH  AD và EF  CD nên ta được SGEF  AD.CD  S ABCD  .
2 2 32

 Trường hợp 2: Tam giác GEF không có cạnh nào nằm trên cạnh của hình vuông ABCD.

Qua E kẻ đường thẳng song song với BC cắt GF và BC lần lượt tại I v| K. Ho|n to|n tơng tự như

1 1 1 1
trên ta chứng minh được SEGI  S AEKB ; SEFI  SEDCK , do đó ta được SGEF  S ABCD  .
2 2 2 32

Ví dụ 45. Trong hình vuông cạnh 12 chứa 2014 điểm. Chứng minh rằng luôn tồn tại một tam giác

đều cạnh 11 phủ kín 504 điểm trong 2014 điểm đã cho.

Lời giải

Tác giả: Nguyễn Công Lợi TÀI LIỆU TOÁN HỌC


35
Website:tailieumontoan.com
Lấy J trên OF sao cho EJ  11 . Ta thấy A B
F

6 3 J
sinFEC    FEC  600 G
4 3 2

Trên tia EC lấy K sao cho EK  EJ  11 . Ta có tam giác


O
I
JEK đều cạnh 11. Ta đi chứng minh tam giác JEK phủ
H
kín tứ giác OHCE.
D E C K
Gọi gi{o điểm của JK với BC là I. Suy ra ta được

 
IC  KC 3  3 5  2 3  5 3  6  6  2 3  CH

Do CH  CI nên H nằm giữa C và I.Suy ra tam giác JEK phủ kín hoàn toàn tứ giác OHCE.

Do vai trò của các tứ gi{c OHCE, OEDG, OGAF, OFBH l| như nhau.

 2014 
Áp dụng nguyên lý Dirichlet ta suy ra: luôn tồn tại    1  504 điểm trong 2014 điểm đã
 4 
cho nằm trong một trong các tứ giác OHCE, OEDG, OGAF hoặc OFBH.

Vậy luôn tồn tại một tam gi{c đều cạnh 11 phủ kín 504 điểm trong 2014 điểm đã cho.

Ví dụ 46. Cho tam giác ABC vuông cân tại A có độ dài cạnh huyền bằng 2015. Trong tam

giác ABC lấy ABC lấy 2031121 điểm phân biệt bất kỳ. Chứng minh rằng tồn tại ít nhất hai

điểm có khoảng cách không lớn hơn 1.

Lời giải

Chia cạnh huyền BC th|nh 2015 đoạn thẳng bằng nhau, mỗi đoạn thẳng có dộ d|i

bằng 1. Từ c{c điểm chia đó vẻ c{c đường thẳng song song với hai cạnh AB v| AC ta được

2015 tam gi{c vuông c}n có cạnh huyền bằng 1 v|  2014  2013  1 hình vuông có

đường chéo bằng 1.


1
Do đó trong tam gi{c ABC có tất cả 2015  2014.2015  2031120 hình (vừa hình vuông có
2
đường chéo bằng 1 vừa tam gi{c vuông c}n có cạnh huyền bằng 1).

Như v}y theo nguyên lý Dirchhlet thì trong 2031121 điểm sẽ tồn tại ít nhất hai điểm nằm

trong một hình n|o đó.

Với hai điểm đó thì khoảng c{ch của nó không lớn hơn 1

Tác giả: Nguyễn Công Lợi TÀI LIỆU TOÁN HỌC


36
Website:tailieumontoan.com
Ví dụ 47. Trong mặt phẳng cho 9 điểm có tọa độ nguyên, trong đó không có 3 điểm nào thẳng

hàng. Hỏi trong số c{c tam gi{c được tạo thành từ 3 trong 9 điểm đó có ít nhất bao nhiêu tam giác

có diện tích nguyên?

Lời giải

Với tam giác ABC có tọa độ đỉnh A  xA ; yA  , B  xB ; yB  , C xC ; yC  thì

S ABC 
1
 x  xA  yB  yA    xB  xA  yC  yA  (1)
2 C

 Xét 9 điểm A, B, C, D, F, G, H, I có tọa độ nguyên thì tọa độ của mỗi điểm sẽ thuộc một trong các
dạng sau: (chẵn, chẵn), (lẻ, lẻ), (lẻ, chẵn), (chẵn, lẻ). Do đó theo nguyên lí Dirichlets tồn tại ít nhất

9
 4   1  3 điểm thuộc cùng một dạng, tức là tọa độ cùng tính chẵn lẻ, giả sử đó l| A, B, C.
 

 Với hai điểm A, B có tọa độ cùng tính chẵn lẻ thì y B  yA và x B  xA đều là số chẵn nên diện

tích tam giác có cạnh AB đều nguyên (do(1)). Tương tự diện tích các tam giác có cạnh là AC, BC

đều nguyên.

 Với mỗi 2 trong 3 điểm A, B, C kết hợp với 6 điểm còn lại thì được 6 tam giác có diện tích
nguyên. Vậy có ít nhất 3.6  1  19 tam giác có diện tích nguyên.

Ví dụ 48. Cho 19 điểm trong đó không có ba điểm nào thẳng hàng, nằm trong một hình lục giác

đều có cạnh bằng 1. Chứng minh rằng luôn tồn tại một tam gi{c m| đỉnh l| ba trong 19 điểm trên

3
có ít nhất một góc không lớn hơn 450 và nằm trong đường tròn bán kính nhỏ hơn .
5

Lời giải

Vẽ c{c đường chéo của lục gi{c đều. C{c đường chéo này chia lục gi{c đều thành 6 tam giác

bằng nhau mỗi cạnh tam gi{c có độ dài bằng 1. Theo nguyên lí Dirichlet thì trong 19 điểm luôn tồn

tại bốn điểm nằm tròn một tam gi{c đều.

Giả sử bốn điểm cùng nằm trong một tam gi{c đều là A, B, C, D. Ta xét các vị trí của bốn điểm A,

B, C, D theo c{c trường hợp sau:

Tác giả: Nguyễn Công Lợi TÀI LIỆU TOÁN HỌC


37
Website:tailieumontoan.com
 Trường hợp 1: Bốn điểm A, B, C, D tạo thành một tứ giác A

lồi. Khi đó ta có A  B  C  D  3600 .

Như vậy trong bốn góc trên tồn tại một góc nhỏ hơn hoặc
B
bằng 900 , giả sử đó l| góc A. Khi đó ta có
D

DAC  CAB  90 nên một trong hai góc DAC; CAB có


0

một góc không lớn hơn 450 .


C
Như vậy một trong hai tam giác ADC và ABD có một góc

không lớn hơn 450 .

 Trường hợp 2: Trong bốn điểm A, B, C, D có một điểm nằn trong tam gi{c có ba đỉnh l| ba điểm
còn lại. Giả sử điểm D nằm trong tam giác ABC.

+ Nếu BDC  900 thì ta được DBC  DCB  900 nên một A

trong hai góc DBC; DCB không lớn hơn 450 . Suy ra tam

giác BCD thỏa mãn yêu cầu bài toán.

+ Nếu BDC  900 thì ta được BAC  900 , do đó D

CAD  BAD  900

Từ đó ta được một trong hai góc CAD; BAD không lớn hơn B C

450 hay một trong hai tam giác ADC và ADB thỏa mãn yêu
cầu bài toán.

3
Mạt khác ta gi{c đều có cạnh bằng một nên b{n kính đường tròn ngoại tiếp tam gi{c đều là .
3

3 3
Mà  nên ta có điều phải chứng minh.
3 5

BÀI TẬP TỰ LUYỆN

Bài 1. Có 15 đội bóng tham dự giải vô địch quốc gia theo thể thức đấu vòng tròn một lượt. Chứng

minh rằng tại bất kì thời điểm nào của giải ta luôn tìm được 2 đội có cùng số trận đấu bằng nhau

tại thời điểm đó(có thể là 0 trận).

Bài 2. Một bà mẹ chiều con nên ng|y n|o cũng cho con ăn ít nhất một chiếc kẹo. Để hạn chế, mỗi

tuần b| cho con không ăn qu{ 12 chiếc kẹo. Chứng minh rằng trong một số ngày liên tiếp n|o đó

bà mẹ đã cho con tổng số 20 chiếc kẹo.


Tác giả: Nguyễn Công Lợi TÀI LIỆU TOÁN HỌC
38
Website:tailieumontoan.com
Bài 3. Chứng minh rằng trong 2001 người bất kỳ, luôn có ít nhất hai người có số người quen bằng

nhau(số người quen chỉ tính trong nhóm)

Bài 4. Trong một thời gian nọ của một lớp học Toán có một nhóm gồm 5 học sinh mà cứ mỗi người

trong nhóm n|y thì rơi v|o trong trạng thái ngủ gục trong lớp đúng 2 lần. Với mỗi cặp học sinh,

đều có cả hai cùng ngủ gục một lần. Chứng minh rằng tại một thời điểm n|o đó có ba học sinh

trong nhóm đó đồng thời ngủ gục .

Bài 5. Có 5 người đấu cờ với nhau. Hãy x{c định kết quả của tất cả các trận đấu nếu biết rằng mỗi

người chơi một lần với 4 người kia và số điểm của mỗi người nhận được đều khác nhau. Ngoài ra:

a) Người xếp thứ nhất không hoà trận nào.

b) Người xếp thứ nhì không thua trận nào.

c) Người xếp thứ tư không thắng trận nào.

Bài 6. Các học sinh được phát bài kiểm tra với mỗi môn một bài và trong n( n  3 ) môn học. Biết

rằng với một môn học bất kỳ có đúng 3 học sinh đạt điểm tối ưu, còn với hai môn tuỳ ý thì có

đúng 1 học sinh đạt điểm tối ưu cho mỗi môn trong cả hai môn đó. Hãy x{c định số n bé nhất sao

cho từ c{c điều kiện trên có thể suy ra rằng có đúng 1 học sinh đạt điểm tối ưu cho mỗi môn trong

cả n môn học.

 
Bài 7. Cho m máy tính và n máy in m  n mỗi sợi dây cáp chỉ nối được một máy tính và một

máy in. Tại một thời điểm bất kỳ mỗi máy tính chỉ có thể điều khiển được một m{y in v| người lại

mỗi máy in chỉ in được cho một máy tính. Hỏi phải dùng ít nhất là bao nhiêu sợi d}y c{p để n máy

tính bất kỳ có thể đồng thời in được?

Bài 7. Kì thi tuyển sinh v|o trường THPT chuyên Long An năm nay có 529 học sinh đến từ 16 địa

phương kh{c nhau tham dự. Giả sử điểm bài thi môn Toán của mỗi học sinh đều là số nguyên lớn

hơn 4 v| bé hơn hoặc bằng 10. Chứng minh rằng luôn tìm được 6 học sinh có điểm môn Toán

giống nhau v| cùng đến từ một địa phương.

Bài 8. Xét 20 số nguyên dương đầu tiên 1, 2, 3, , 20. Hãy tìm số nguyên dương k nhỏ nhất có

tính chất: Với mỗi cách lấy ra k số phân biệt từ 20 số trên, đều lấy được hai số phân biệt a và b sao

cho a  b là một số nguyên tố.

Tác giả: Nguyễn Công Lợi TÀI LIỆU TOÁN HỌC


39
Website:tailieumontoan.com

 
Bài 9. Cho tập hợp X  1; 2; 3;...; 2024 . Chứng minh rằng trong 45 số khác nhau bất kỳ

được lấy ra từ tập X luôn tồn tại hai số x, y sao cho x  y  1 .

 
Bài 10. Chứng minh rằng tồn tại số nguyên dương n thỏa mãn 13579 n  1 chia hết cho 313579 .

Bài 11. Trong một cái bát hình vuông cạnh 18 cm có 128 hạt vừng. Chứng minh rằng tồn tại hai hạt

vừng có khoảng cách tới nhau nhỏ hơn 2 cm.

Bài 12. Bên trong tam gi{c đều ABC cạnh 1 đặt 5 điểm. Chứng minh rằng tồn tại 2 điểm có khoảng

cách nhỏ hơn 0,5.

Bài 13. Cho hình tròn có bán kính n, ở đ}y n l| số nguyên dương. Trong hình tròn có 4n đoạn

thẳng đều có độ dài bẳng 1. Cho trước một đường thẳng d. Chứng minh rằng tồn tại đường thẳng

d’ hoặc song song với d, hoặc là vuông góc với d sao cho d’ cắt ít nhất hai đoạn thẳng đã cho.

Bài 14. Cho một bảng có kích thước 2n  2n ô vuông. Người ta đ{nh dấu vào 3n ô bất kì của

bảng. Chứng minh rằng có thể chọn ra n hàng và n cột của bảng sao cho c{c ô được đ{nh dấu đều

nằm trên n hàng và n cột này.

Bài 15. Chứng minh rằng trong mọi đa gi{c lồi với số cạnh chẵn, tồn tại đường chéo không song

song với một cạnh nào của đa gi{c.

Bài 16. Một hình lập phương có cạnh bằng 15 chứa 11000 điểm. Chứng minh rằng có một hình cầu

bán kính 1 chứa ít nhất 6 điểm trong số 11000 điểm đã cho.

Bài 17. Giả sử 1 bàn cờ hình chữ nhật có 3x7 ô vuông được sơn đen hoặc trắng. Chứng minh rằng

với c{ch sơn m|u bất kì, trong bàn cờ luôn tồn tại hình chữ nhật gồm các ô ở 4 góc là các ô cùng

màu.

Bài 18. Trong một tờ giấy hình vuông bằng giấy có cạnh bằng 12 cm có 31 lỗ kim châm. Chứng

minh rằng ta vẫn có thể cắt từ tờ giấy này ra một hình tròn có bán kính 1 cm mà không chứa một

lỗ kim châm nào.

Bài 19. Cho hình tròn (C) có diện tích bằng 8, đặt 17 điểm phân biệt bất kì. Chứng minh rằng bao

giờ cũng tìm được ít nhất ba điểm tạo thành một tam giác có diện tích bé hơn 1.

Tác giả: Nguyễn Công Lợi TÀI LIỆU TOÁN HỌC


40
Website:tailieumontoan.com
Bài 20. Trong hình vuông cạnh bằng 15 đặt 20 hình vuông nhỏ cạnh bằng 1 và từng đôi một không

cắt nhau. Chứng minh rằng trong hình vuông lớn có thể đặt một hình tròn bán kính 1 sao cho nó

không cắt hình vuông nào.

Bài 21. Trong mặt phẳng cho tập S gồm 8065 điểm đôi một phân biệt mà diện tích của mỗi tam giác có 3

đỉnh thuộc tập S đều không lớn hơn 1 (quy ước nếu 3 điểm thẳng hàng thì diện tích của tam giác tạo bởi

3 điểm này bằng 0). Chứng minh rằng tồn tại một tam giác T có diện tích không lớn hơn 1 chứa ít nhất

2017 điểm thuộc tập S (mỗi điểm trong số 2017 điểm đó nằm trong hoặc nằm trên cạnh của tam giác T).

Bài 22. Cho tam gi{c đều MNP có cạnh bằng 2 cm. Lấy n điểm thuộc các cạnh hoặc ở phía trong

tam gi{c đều MNP sao cho khoảng cách giửa hai điểm tuỳ ý lớn hơn 1 cm (với n là số nguyên

dương). Tìm n lớn nhất thoả mãn điều kiện đã cho.

Bài 23. Trên mặt phẳng cho 25 điểm phân biệt và trong ba điểm bất kì bao giờ cũng tìm được hai

điểm có khoảng cách giữa chúng nhỏ hơn 1. Chứng minh rằng tồn tại một hình tròn có bán kính

bằng 1 chứa không ít hơn 13 điểm trong c{c điểm trên.

Bài 24. Cho điểm P nằm trong đa gi{c lồi 2n cạnh. Vẽ c{c đường thẳng đi qua P và mỗi đỉnh của

đa gi{c. Chứng minh rằng bao giờ cũng tìm được một cạnh của đa gi{c sao cho không một đường

thẳng n|o trong c{c đường thẳng trên có điểm chung với cạnh đó.

Bài 25. Cho 19 điểm phân biệt nằm trong một tam gi{c đều có cạnh bằng 3, trong đó không có 3

điểm nào thẳng hàng. Chứng minh rằng luôn tìm được một tam gi{c có 3 đỉnh l| 3 trong 19 điểm đã

3
cho mà có diện tích không lớn hơn .
4

Bài 26. Trong hình vuông cạnh bằng 1 cho 5 điểm bất kỳ. Chứng minh rằng, trong các

2
điểm đã cho có thể tìm được 2 điểm sao cho khoảng cách giữa chúng không lớn hơn
2

Bài 27. Cho tam giác nhọn ABC có BAC  600 và BC  2 3cm . Bên trong tam giác này

cho 2017 điểm bất kì. Chứng minh rằng trong 2017 điểm ấy luôn tìm được 169 điểm mà khoảng

cách giữa hai điểm trong chúng không lớn hơn 1cm.

Bài 28. Trên mặt phẳng cho năm điểm phân biệt sao cho không có ba điểm nào thẳng hàng và

không có bốn điểm nào thuộc cùng một đường tròn. Chứng minh rằng tồn tại một đường tròn đi

Tác giả: Nguyễn Công Lợi TÀI LIỆU TOÁN HỌC


41
Website:tailieumontoan.com
qua ba điểm trong năm điểm đã cho v| hai điểm còn lại có đúng một điểm nằm bên trong đường

tròn

Bài 29. Trong hình chữ nhật có chiều dài và rộng lần lượt bằng 4 v| 3 cho 49 điểm, trong đó không

có ba điểm nào thẳng hàng. Chứng minh rằng tồn tại một tam gi{c có c{c đỉnh thuộc 49 điểm trên

1
mà diện tích nhỏ hơn 
2

Bài 30. Trong tam gi{c đều có cạnh bằng 8 đặt 193 điểm ph}n biệt. Chứng minh tồn tại 2

3
điểm trong 193 điểm đã cho có khoảng c{ch không vượt quá .
3
Bài 31. Trên cùng một mặt phẳng cho 4033 điểm, biết rằng 3 điểm bất kì trong 4033 điểm trên luôn

chọn được hai điểm có khoảng cách nhỏ hơn 1. Chứng minh rằng trong c{c điểm nói trên có ít

nhất 2016 điểm nằm trong đường tròn bán kính 1.

Bài 32. Trong mặt phẳng cho 2015 điểm. Mỗi điểm là tâm một đường tròn đi qua một điểm cố định

O. Chứng minh rằng từ những hình tròn tạo ra có thể chọn được 5 hình tròn mà chúng phủ tất cả

2015 điểm.

Bài 33. Có 6 đội bóng thi đấu với nhau(mỗi đội phải đấu 1 trận với 5 đội khác). Chứng minh rằng

vào bất cứ lúc n|o cũng có 3 đội trong đó từng cặp đã đấu với nhau hoặc chưa đấu với nhau trận

nào.

Bài 34. Bên trong hình lục gi{c đều có cạnh bằng 2 cho 81 điểm phân biệt. Chứng minh rằng tồn tại

một hình vuông có cạnh bằng 1 (kể cả biên) chứa ít nhất 6 điểm trong số c{c điểm đã cho.

Tác giả: Nguyễn Công Lợi TÀI LIỆU TOÁN HỌC


42
Website:tailieumontoan.com
HƯỚNG DẪN GIẢI

Bài 1. Số lần gặp nhau mà mỗi đội có, có thể nhận 15 giá trị khác nhau: 0; 1; 2; <; 14.

Trong trường hợp này không thể áp dụng nguyên tắc Dirichlet được vì số đội cũng l| 15.

Hai trường hợp 0 trận và 14 trận không thể xảy ra đồng thời vì nếu có một đội n|o chưa

đấu trận n|o thì đồng thời không thể có một đội n|o đó đã đấu hết 14 trận, ngược lại nếu

có một đội đã đ{ 14 trận thì không thể có 1 đội chưa đ{ một trận nào. Vì vậy số lần gặp

nhau mà mỗi đội đã thực hiện trong thực tế có thể nhận thêm 14 giá trị từ 0 đến 13 hoặc từ

1 đến 14. Khi đó theo nguyên tắc Dirichlet ta luôn có thể tìm được hai đội có cùng một số

trận đấu .
Bài 2. Xét 21 ngày liên tiếp kể từ một ngày thứ hai n|o đó. Gọi S(n) là tổng số kẹo mà bà mẹ đã cho


con tính đến ngày thứ n 1  n  21 . 

Ta có S  m   S  n  , m  n 1  m, n  21 và 1  S  n   3.12  36 .

Vì có 21 ngày và chú ý rằng 0  S  m   S  n   36 nên tồn tại m  1 sao cho

S  m   S  n  mod 20   S  m   S  n  20  S  m   S  n   20

Như vậy từ ngày n  1 đến ngày thứ m, bà mẹ đã cho con tổng cộng đúng 20 chiếc kẹo.

Bài 3. Gọi số người quen của A i là a i , khi đó ta có 0  a i  2001 với 1  i  2001 . Xét c{c trường

hợp:

 Tồn tại một người trong 2001 không quen ai, suy ra không có ai quen cả 2000 người còn lại trong
nhóm.

Khi đó đó ta có 0  a i  1999 với 1  i  2001 từ đó suy ra tồn tại hai số a k  a m với

1  k,m  2001 hay tồn tại hai người có số người quen bằng nhau

 Mỗi người đều quen ít nhất một người suy ra, khi đó ta có 0  ai  2000 với 1  i  2001 từ đó

suy ra tồn tại hai số a k  a m với 1  k,m  2001 hay tồn tại hai người có số người quen bằng

nhau

Vậy b|i to{n được chứng minh.

Tác giả: Nguyễn Công Lợi TÀI LIỆU TOÁN HỌC


43
Website:tailieumontoan.com
Bài 4. Giả sử ngược lại rằng không hề có chuyện 3 học sinh đồng thời ngủ gục. Ta sẽ chứng minh

điều này mâu thuẫn.

Thật vậy, trong khoảng thời gian có hai người đồng thời ngủ gục, 3 người còn lại tỉnh táo.

Theo đề bài, mỗi học sinh trong nhóm đều ngủ gục đúng hai lần nên một trong hai người(đang

ngủ gục) sẽ có lúc lại ngủ gục với một trong 3 người còn lại. Như vậy nhiều nhất sẽ có tất cả là 9

khoảng thời gian diễn ra ngủ gục từng cặp. Nhưng nhóm n|y có 5 học sinh nên số cặp là học sinh

có thể ra là 10, mà chỉ có nhiều lắm là 9 khoảng thời gian. Do vậy sẽ có ít nhất một cặp không đồng

thời ngủ gục. Ta có điều mâu thuẫn.

Bài 5. Theo điều kiện của bài ra ta thấy ngay người xếp thứ nhất thắng người xếp thứ ba, thứ tư,

thứ năm v| được tất cả 3 điễm. Còn người thứ nhì thắng người xếp thứ nhất. Người thứ nhì hòa

trong các trận đấu với người xếp thứ ba, thứ tư, thứ năm v| nhận 2,5 điểm.

Những người còn lại chỉ nhận số điểm lớn nhất lần lượt là 2; 1; 5; 1. Ta chứng minh họ

không thể nhận ít hơn.

Thật vậy, vì có 5 người nên họ chơi tất cả 10 trận và nhận tất cả 10 điểm. Nhưng người xếp

thứ nhất và thứ nhì đã nhận 5,5 điểm nên ba người còn lại nhận 4,5 điểm. Mặt khác

2  1,5  1  4,5 nên họ không thể nhận ít hơn. Như vậy, do người thứ tư không thắng trận nào
nên anh ta hoàvới người xếp thứ ba và thứ năm. Còn lại người thứ ba thắng người thứ năm.

Bài 6. Ta biểu thị mỗi học sinh bằng một điểm trong mặt phẳng sao cho không có ba điểm nào

thẳng hàng. Nếu hai học sinh đạt điểm tối ưu ở một môn n|o đó, ta nối hai điểm tương ứng lại với

nhau. Khi đó, theo đề bài, mỗi môn học sẽ cho tương ứng duy nhất một tam giác vàbất cứ hai tam

gi{c n|o cũng có đúng một đỉnh chung.

Chú ý rằng nếu như bốn tam giác có chung một đỉnh thì tất cả c{c tam gi{c đều có chung

đỉnh đó, bởi vì nếu không thì tam giác thứ năm sẽ có chung đỉnh với mỗi một trong bốn tam giác

đó. Như vậy tam giác thứ năm n|y sẽ có bốm đỉnh, điều này mâu thuẩn.

Bây giờ nếu n  8 thì một tam giác sẽ có chung một đỉnh với mỗi một trong 7 tam giác còn lại.

Theo nguyên lí Dirichlet thì một trong c{c đỉnh của nó sẽ có chung đỉnh với ít nhất ba tam giác

khác, tức là tồn tại 4 tam giác có chung một đỉnh.

Cuối cùng ví dụ sau đ}y chứng tỏ rằng trường hợp n  7 không thỏa mãn đề bài. Trong

bảng dưới đ}y ta dùng dấu chéo () để chỉ học sinh đạt điểm tối ưu ở môn học tương ứng

Tác giả: Nguyễn Công Lợi TÀI LIỆU TOÁN HỌC


44
Website:tailieumontoan.com
Học

sinh 1 2 3 4 5 6 7

Môn học

I x x x

II x x x

III x x x

IV x x x

V x x x

VI x x x

VII x x x

Như vậy giá trị nhỏ nhất của n là 8.

Bài 7. Ta xét một cách nối thoả mãn đề b|i như sau: Với n m{y tính đầu tiên mỗi máy nối với một

máy in, còn với m  n máy tính còn lại, mỗi máy nối với tất cả n máy in.

Khi đó số dây cáp cần dùng trong cách nối này là S  n  n  m – n   n  m  n  1

Ta sẽ chứng minh rằng nếu số dây cáp S  n  m  n  1 thì không thoả mãn điều kiện đầu bài.

Thật vậy, nếu S  n  m  n  1 thì có ít nhất một m{y in x n|o đó được nối với không quá m  n

máy tính. Từ đó suy ra rằng có m máy tín mà trong số đó có m{y n|o nối với m{y in x, điều này có

nghĩa l| m{y tính đó không thể n|o đồng thời in được.

Tóm lại số sợi dây cáp ít nhất cần phải dùng là S  n  m  n  1

Bài 7. Ta có 529 học sinh có điểm bài thi từ 5 điểm đến 10 điểm. Theo nguyên lý Dirichlet ta có 89

học sinh có điểm b|i thi như nhau (từ 5 điểm đến 10 điểm).

Ta có 89 học sinh có điểm b|i thi như nhau v| đến từ 16 địa phương. Theo nguyên lý

Dirichlet tìm được 6 em có cùng điểm thi môn to{n v| đến từ cùng một địa phương.

Tác giả: Nguyễn Công Lợi TÀI LIỆU TOÁN HỌC


45
Website:tailieumontoan.com

Bài 8. Xét tập hợp 2; 4; 6; 8;10;12;14;16;18; 20  , ta thấy tổng của hai phần tử bất kì của tập hợp

n|y đều không phải là số nguyên tố. Do đó k  11 , ta sẽ chứng minh k  11 là số nhỏ nhất thỏa

mãn yêu cầu bài toán. Thật vậy, ta chia tập hợp A  1; 2; 3;...; 20 thành 10 cặp số sau:

1,2  ,  3,16  ,  4,19  ,  5,6  , 7,10  , 8,9  , 11,20  , 12,17  , 13,18  , 14,15
Tổng của hai số trong mỗi cặp số trên là số nguyên tố. Khi đó mỗi tập con của A có 11 phần

tử thì tồn tại ít nhất hai phần tử thuộc cùng vào một trong 10 cặp số trên. Suy ra trong A luôn có

hai phần tử phân biệt có tổng là một số nguyên tố.

Bài 9. Chia 2012 số 1; 2; 3;...; 2024 th|nh 44 đoạn gồm

 1; 3  ,  4; 8  ,...,  1936; 2024  . C{c đoạn trên có dạng tổng quát là  k 2 ;  k  1


2 
 1
       
.

Như vậy 45 số thuộc tập hợp X nằm trong 44 đoạn trên. Theo nguyên lí Dirichlet thì tồn tại hai số

trong 45 số trên nằm trong cùng một đoạn. Không mất tính tổng quát ta giả sử hai số đó l| x, y v|

 
 k  1
2
chúng nằm trong đoạn  k 2 ;  1 .
 

 k  1  k  1
2 2
Khi đó ta có x  y   1  k2   k2  k  1  k  1 .

Từ đó ta có điều phải chứng minh.

Bài 10. Đặt a  313579 , khi đó ta được  a,13579   1 .

Xét a  1 số có dạng 13579;135792 ;135793 ;...;13579a 1 .

Chia a  1 số trên cho a  313579 ta được a  1 số dư . M| trong phép chia cho a thì có a số dư. Như

vậy Trong a  1 số dư như trên tồn tại hai số dư bằng nhau hay tong tại hai số trong dãy số trên có

cùng số dư khi chia cho a. Giả sử hai số đó l| 13579m và 13579 với m  p .


p

 
Khi đó ta được 13579m  13579p a hay 13579p 13579 m p  1 a .  
 
Do  a,13579   1 nên a,13579 p  1 , suy ra 13579m p  1 a  

Tác giả: Nguyễn Công Lợi TÀI LIỆU TOÁN HỌC


46
Website:tailieumontoan.com

 
Điều n|y có nghĩa l| tông tại số có dạng 13579 n  1 chia hết cho 313579 .

Bài 11. Lấy mỗi hạt vừng làm tâm dựng hình tròn bán kính 1 cm. Các hình tròn này nằm hoàn

toàn trong hình vuông có cạnh 20cm thu được từ hình vuông đã cho bằng cách tịnh tiến bốn cạnh

của nó một khoảng 1cm ra phía ngoài. Tổng diện tích của các hình tròn bán kính 1cm này là 128 

> 402,112 > 400. Do đó tổng diện tích các hình tròn này lớn hơn diện tích hình vuông cạnh 20 cm.

Bài 12. C{c đường trung bình của tam giác đều cạnh 1 sẽ chia nó ra l|m 4 tam gi{c đều cạnh 0,5.

Do đó trong một tam giác nhỏ đó có ít nhất 2 điểm đã cho, v| c{c điểm đó không thể rơi v|o c{c

đỉnh của tam giác ABC.Vậy khoảng cách giữa hai điểm đó nhỏ hơn 0,5.

Bài 13. Giả sử AB l| đoạn thẳng có độ dài bằng 1, a v| a’ l| hai đường thẳng bất kì vuông góc với

nhau. Gọi A’B’ v| A‛B‛ l| c{c hình chiếu của AB lên a v| a’. Khi đó ta có: A’B’  A‛B‛  AB hay

A’B’  A‛B‛  1.

Áp dụng vào bài toán ta gọi d‛ l| đường thẳng bất kì vuông góc với d. Chiếu vuông góc tất cả 4n

đoạn thẳng lên d v| d‛. từ (1) suy ra tổng độ dài hình chiếu của tất cả 4n đoạn thẳng không bé hơn

4n.

Vì vậy, theo nguyên lí Dirichlet trong hai đường thẳng d v| d‛ có ít nhất một đường thẳng mà

tổng độ dài của hình chiếu c{c đoạn thằng lên nó không bé hơn 2n. Không mất tính tổng quát ta có

thể giả sử đó l| d.

Mặt khác, mỗi đoạn thẳng đầu nằm trọn trong hình tròn b{n kính n (đường kính 2n), nên hợp các

hình chiếu của chúng trên d có độ d|i không vượt quá 2n.

Vì vậy, theo nguyên lí dirichlet trên d tồn tại ít nhất một điểm M thuộc vào hình chiếu của ít nhất

hai đoạn thẳng trong số 4n đoạn thẳng đã cho. Gọi d’ l| đường thẳng vuông góc với d tại M.

Đường thẳng d’ chính l| đường thẳng cần tìm.

Bài 14. Chọn ra n hàng có chứa số ô được đ{nh dấu nhiều trên c{c h|ng đó nhất. Ta chứng minh

rằng c{c ô được đ{nh dấu còn nhỏ hơn hoặc bằng n.

Tác giả: Nguyễn Công Lợi TÀI LIỆU TOÁN HỌC


47
Website:tailieumontoan.com
Giả sử ngược lại không phải như vậy, tức là

số ô được đ{nh dấu lớn hơn hoặc bằng n  1

. Số các hàng còn lại chưa chọn là n. Vậy theo

nguyên lí Dirichlet sẽ có ít nhất một hàng (

tỏng số n hàng còn lại) chứa ít nhất hai ô đã

đ{nh dấu. Chú ý rằng theo cách chọn thì n

h|ng đã chọn có chứa số ô được đ{nh dấu

nhiều trên c{c h|ng đó nhất. Có một hàng

còn lại chưa chọn có ít nhất hai ô đ{nh dấu,

nên suy ra mọi hàng trong số n h|ng đã chọn

đều có ít nhất hai ô được chọn, tức là trên n

h|ng đã chọn có không ít hơn 2n ô đã được

đ{nh dấu.

Như vậy, số ô được đ{nh dấu lớn hơn hoặc bằng 2n   n  1  3n . Vô lí vì chỉ có 3n ô được đ{nh

dấu. Vậy nhận xét được chứng mình.

Như vậy, sau khi đã chọn ra n hàng(với cách chọn như trên), theo nhận xét còn lại có không quá n

ô được đ{nh dấu. Vì thế cùng lắm là có n cột chứa chúng. Vì lẽ đó sẽ không thấy còn ô đ{nh dấu

nào nằm ngoài các hàng hay cột được chọn.

n  n  3
Bài 15. Ta giả thiết rằng nếu một đa gi{c có n cạnh thì có đường chéo.
2

Xét một đa gi{c lồi bất kì với số cạnh là chẵn (đa gi{c lồi 2k cạnh với k  2 ).

2k  2k  3 
Khi đó số đường chéo của nó là s  .
2


Ta có s  k  2k – 3   2k  k – 2   k nên suy ra s  2k k – 2 . 
Giả sử ngược lại đa gi{c n|y có tính chất mỗi đường chéo của nó đều song song với một cạnh nào

 
đó của đa gi{c. Đa gi{c n|y có 2k cạnh, vì thế từ s  2k k – 2 suy ra tồn tại ít nhất k  1 đường

chéo d1 ,d2 ,d3 ,...,dk1 m| c{c đường chéo này cùng song song với một cạnh a n|o đó của tam giác

Tác giả: Nguyễn Công Lợi TÀI LIỆU TOÁN HỌC


48
Website:tailieumontoan.com
đã cho. Thật vậy, nếu ngược lại mỗi cạnh tối đa l| song song k  2 đường chéo thì tối đa ta chỉ có

k  k  2  đường chéo và s  2k  k – 2  . Điều này mâu thuẫn với (1).

Như thế ta có k đường thẳng song song với nhau a,d1 ,d2 ,d3 ,...,dk1 .

Mặt kh{c đa gi{c đã cho l| đa gi{c lồi nên c{c đường chéo d1 ,d2 ,d3 ,...,dk1 cùng nằm trên một

nửa mặt phẳng bờ x{c định cạnh a. Không mất tính tổng quát có thế cho d1 l| đường chéo xa nhất

đối với a. Ta có tất cả k đoạn thẳng phân biệt, nên mỗi đỉnh của đa gi{c đều l| đầu mút của một

đoạn n|o đó trong k đoạn trên. Từ đó suy ra to|n bộ đa gi{c nằm hẳn về một nửa mặt phẳng xác

định bởi d1 . Do d1 l| đường chéo, nên điều này mâu thuẫn với tính lồi của đa gi{c. Vậy giả thiết

phản chứng là sai.

Từ đó ta có điều phải chứng minh.

Bài 16. Chia mỗi cạnh của hình lập phương th|nh 13 phần bằng nhau. Như thể hình lập phương

đã cho được chia thành 133  2197 hình lập phương nhỏ. Do 11000  5.2197 – 10985 , nên tồn tại

ít nhất 1 hình lập phương nhỏ, mà hình lập phương n|y chứa ít nhất 6 điểm. Như đã biết, nếu gọi

1
cạnh hình l}p phương bằng a, thì hình cầu ngoại tiếp có bán kính R với R  a 3.
2

15
Vì thế hình cầu ngoại tiếp hình lập phương nhỏ(cạnh của nó là ) được x{c định là
13

2
1 15 1  15  1 675 1
R 3 3    4 1
2 13 2  13  2 169 2

Hình cầu b{n kính R n|y dĩ nhiên chứa ít nhất 6 điểm trong số 11000 điểm đã cho.

Bài 17. Mẫu sơn m|u có thể xảy ra với bàn cờ này có dạng từ 1 đến 8. Giả sử một trong số các cột

thuộc dạng 1. Bài toán sẽ được chứng minh nếu tất cả các cột còn lại thuộc dạng 1, 2, 3 hoặc 4. Giả

sử tất cả các cột còn lại thuộc dạng 5, 6, 7, 8. Khi đó theo nguyên lí Dirichlet 2 trong số 6 cột có 2 cột

cùng 1 dạng v| như vậy b|i to{n cũng được chứng minh.

Chứng minh ho|n to|n tương tự nếu 1 cột có dạng 8. Giả sử không có cột nào trong các cột

1, 8 thì theo nguyên lí Dirichlet cũng có 2 cột cùng dạng và b|i to{n cũng đựơc chứng minh.

Bài 18. Lấy mỗi lỗ kim là tâm dựng một hình tròn bán kính 1cm. Tổng diện tích của 31 hình tròn

này sẽ là 31 nhỏ hơn diện tích của hình vuông cạnh 10 cm . Do đó phải có một điếm M trong
Tác giả: Nguyễn Công Lợi TÀI LIỆU TOÁN HỌC
49
Website:tailieumontoan.com
hình vuông cạnh 10 cm(l| hình vuông thu được từ hình vuông cạnh 12 cm đã cho bằng cách thu

hẹp các chiều 1cm) và không nằm trong 31 hình tròn b{n kính được dựng như đã trình b|y ở trên.

Lấy điểm M làm tâm ta cắt một hình tròn bán kính 1cm, thì hình tròn này nằm hoàn toàn trong

hình vuông đã cho có cạnh dài 12 cm và không chứa một lỗ kim châm nào cả.

Bài toán tổng quát có thể được phát biểu như sau: Trong một tờ giấy hình vuông có cạnh

bằng b có a lỗ kim ch}m. (trong đó b  ar 2 ). Chứng minh rằng ta vẫn có thể cắt từ tờ giấy này

ra một hình tròn có bán kính r cm mà không chứa một lỗ kim châm nào.

Bài 19. Chia hình tròn thành (C) thành 8 hình quạt bằng nhau, mỗi hình quat có diện tích bằng 1.

Theo nguyên lý Dirichlet, tồn tại ít nhất một hình quạt (a) chứa 3 điểm trong số 17 điểm đã cho.

Tam gi{c có 3 đỉnh l| 3 điểm đó nằm trọn trong hình quạt nên có diện tích nhỏ hơn diện tích hình

quạt, tức l| bé hơn 1.

Bài 20. Xét hình gồm tất cả c{c điểm cách hình vuông nhỏ cạnh 1 một khoảng không lớn hơn 1. Rõ

ràng hình tròn bán kính 1 có tâm nằm ngo|i hình đó nên không thể cắt hình vuông nhỏ. Diện tích

hình đó bằng 5   . Tâm hình tròn cần tìm cũng cần phải cách các cạnh của hình vuông lớn hơn

 
một khoảng lớn hơn 1, tức là ở bên trong hình vuông cạnh 13. Vì 20 5    132 . Hình tròn có

tâm tại điểm không bị phủ sẽ có tính chất thỏa mãn đề bài.

Bài 21. Gọi d l| khoảng c{ch hai điểm Ai , A j xa nhất trong tất cả c{c điểm thuộc tập hợp

S. Giả sử A k có khoảng c{ch đến đường thẳng Ai A j lớn nhất. Khi đó tam gi{c Ai A jAk có

S Ai AjAk  1 v| lớn nhất. Từ c{c điểm Ai , A j và A k vẽ c{c đường thẳng song song với c{c

cạnh của tam gi{c Ai A jAk thì ta thu được bốn tam gi{c con bằng nhau v| một tam gi{c

lớn. Tam gi{c lớn có diện tích không vượt qu{ 4 đơn vị. Tam gi{c n|y chứa 8065 điểm đã

cho. Vì 8065 : 4  2016 dư 1. Nên theo nguyên lí Dirichlet thì tồn tại một tam gi{c con chứa

2017 điểm thuộc tập hợp S thỏa mãn đề b|i.

Bài 22. Tam gi{c đều có cạnh bằng 2 cm thì diện tích bằng 3 cm2 , tam gi{c đều có cạnh bằng 1

3 3
cm thì diện tích bằng cm2 . Nếu tam gi{c đều có cạnh lớn hơn 1cm thì diện tích lớn hơn
4 4
cm2

Tác giả: Nguyễn Công Lợi TÀI LIỆU TOÁN HỌC


50
Website:tailieumontoan.com
Gọi t là số tam gi{c đều có cạnh lớn hơn hoặc bằng 1cm chứa được trong tam gi{c đều có cạnh 2

cm:

1  t  4 (với t là số nguyên dương). Suy ra t max  3 .

Theo nguyên lý Drichlet sẽ có 1 trong t tam gi{c đều có cạnh lớn hơn 1cm đó chứa tối đa 2

điểm thoả mãn khoảng cách giữa hai điểm bất kỳ luôn lớn hơn 1cm.

Vậy số điểm thoả yêu cầu bài toán là 2  n  4 nên giá trị lớn nhất của n 4

Bài 23. Xét hai điểm A v| B trong 25 điểm đã cho thỏa mãn điều kiện AB có độ dài lớn nhất. Ta xét

c{c trường hợp sau:

 Trường hợp 1: Với AB  1 . Khi đó mọi điểm C bất kì trong 25 điểm đã cho ta đều có
AC  AB  1 . Suy ra toàn bộ 25 điểm trên cùng nằm trong đường tròn tâm A có bán kính bằng 1.

 Trường hợp 2: Với AB  1 . Khi đó xét điểm C trong số c{c điểm còn lại.

Theo giả thiết với ba điểm A, B, C ta luôn có AC  1 hoặc BC  1 .

Như vậy với c{c điểm còn lại có ít nhất 12 đoạn thẳng xuất phát từ A có độ dài nhỏ nhơn 1 hoặc

có ít nhất 12 điểm xuất phát từ B có độ dài nhỏ hơn 1. Vẽ đường tròn tâm A bán kính bằng 1 hoặc

đường tròn tâm B bán kính bằng 1.

Suy ra đường tròn tâm A hoặc đường tròn tâm B chứa ít nhất 13 điểm tròn số c{c điểm đã cho.

Từ đó b|i to{n được chứng minh.

Bài 24. Xét đa gi{c lồi 2n cạnh A1A2 A3 ...A2n . Ta xét c{c trường hợp sau:

 Trường hợp 1: Điểm P nằm trên một đường chéo của đa gi{c A1A2 A3 ...A2n .

Khòn mất tình tổng quát ta giả sử P nằm trên đường chéo Ai A j của đa gi{c. Khi đó hai đường

thẳng PAi và PA j trùng nhau và không cắt phần trong của bất cứ cạnh nào của đa gi{c. Khi đó

2n  2 chỉ cắt tối đa 2n  2 cạnh của đa gi{c. Vậy có ít nhất hai cạnh của đa gi{c thỏa mãn yêu cầu
của bài toán.

 Trường hợp 2: Điểm P không nằm trên bất kì đường chéo nào của đa gi{c A1A2 A3 ...A2n . Vẽ

đường chéo A1An 1 , khi đó mỗi phía của đường chéo chứa n cạnh của đa gi{c. Không mất tính

Tác giả: Nguyễn Công Lợi TÀI LIỆU TOÁN HỌC


51
Website:tailieumontoan.com
tổng quát ta giả sử điểm P nằm trong đa gi{c A1A2 A3 ...An 1 . Khi đó n  1 đường thẳng

PAn1 ; PAn2 ; PAn3 ;...; PA2n không thể cắt các cạnh An1An2 ; An2 An2 ;...; A2n A1 . Còn lại

n  1 đường thẳng cắt phần trong tối đa của n  1 cạnh trong n cạnh n|y. Như vậy tồn tại một
trong n cạnh An1An2 ; An2 An2 ;...; A2n A1 thỏa mãn yêu cầu bài toán.

Bài 25. Giả sử 19 điểm nằm trong tam gi{c đều ABC cạnh bằng 3. Chia tam giác ABC thành 9 tam

3
gi{c đều, có cạnh bằng 1 (gọi là tam giác nhỏ). Mỗi tam giác nhỏ có diện tích là S  . Để ý là
4
19  2.9  1

Vì có 19 điểm nằm trong 9 tam giác nhỏ nên theo nguyên lý Dirichlets thì có ít nhất 3 điểm cùng

thuộc một hình tam giác nhỏ. Giả sử 3 điểm đó l| I1 ,I 2 ,I 3 .

3
Khi đó tam gi{c I1I 2 I 3 nằm trong một tam giác nhỏ nên S I I I  .
1 2 3
4

Bài 26. Chia hình vuông đã th|nh 4 hình vuông con. Dễ d|ng tính được, cạnh của một hình vuông

1 2
con là v| đường chéo là . Gieo 5 điểm đã cho v|o hình vuông ban đầu, 5 điểm đó sẽ nằm
2 2
trong 4 hình vuông con. Theo nguyên tắc Dirichlets thì tồn tại 2 điểm nằm trong cùng một hình

vuông. do đó khoảng cách giữa 2 điểm đó sẽ không lớn hơn đường chéo của hình vuông chứa nó,

2
tức là không lớn hơn .
2

Bài 27. Gọi O l| t}m đường tròn ngoại tiếp ΔABC. Kẻ OH, OK, OG lần lượt vuông góc với

c{c cạnh AB, AC, BC. Dễ d|ng chứng minh được c{c tứ gi{c AHOK, BHOG, KOGC nội

tiếp c{c đường tròn đường kính OA  OB  OC  2 cm . Ta có 2017 điểm nên theo nguyên

lí Diriclet thì sẽ tồn tại một tứ gi{c có chứa ít nhất 673 điểm, giả sử đó l| tứ gi{c OKCG.

Xét tứ gi{c OKCG. Gọi I l| trung điểm OC nên I l| t}m đường tròn ngoại tiếp tứ gi{c

OKCG

Khi đó ta được IO  IK  IC  IG  1 cm . Kẻ IM, IN, IP, IQ lần lượt vuông góc với OK, KC,

CG, GO. Suy ra bốn tứ gi{c OMIQ, MKNI, INCP, PGQI nội tiếp c{c đường tròn đường

kính bằng 1 cm

Tác giả: Nguyễn Công Lợi TÀI LIỆU TOÁN HỌC


52
Website:tailieumontoan.com
m| có 673 điểm. Như vậy theo nguyên lí Diriclet thì sẽ tồn tại một tứ gi{c có chứa ít nhất

169 điểm, giả sử đó l| tứ gi{c MKNI. Khi đó 169 điểm n|y sẽ thuộc đường tròn ngoại tiếp

tứ gi{c MKNI có đường kính IK  1cm . Do đó khoảng c{ch 169 điểm n|y không lớn hơn

1cm

Bài 28. Giả sử 5 điểm đó l| A, B, C, D, E. Vì trong năm điểm n|y không có ba điểm n|o

thẳng h|ng nên ta tồn tại 2 điểm trong năm điểm đã cho sao cho đường thẳng đi

qua 2 điểm n|y chia mặt phẳng th|nh hai nửa mặt phẳng v| 3 điểm còn lại nằm cùng về

một nửa mặt phẳng bờ l| đường thẳng trên. Giả sử đó l| 2 điểm A và B. Xét 3 góc sau

ACB; ADB;AEB . Khi đó do trong 5 điểm không có bốn điểm n|o cùng thuộc một đường

tròn nên ta ho|n to|n có thể sắp thứ tự chúng như sau: ACB  ADB  AEB . Khi đó dễ

thấy điểm E nằm bên trong đường tròn ngoại tiếp tam gi{c ADB còn điểm C nằm bên

ngo|i đường tròn ngoại tiếp tam gi{c ADB.(chứng minh đơn giản dựa theo c{ch tính số đo

của góc có đỉnh nằm trong, nằm ngo|i đường tròn)


1
Bài 29. Chia hình chữ nhật 4  3 thành 24 hình chữ nhật  1 , mỗi hình chữ nhật có diện tích là
2
1 1
 Vì có 49 điểm nằm trong 24 hình chữ nhật nên tồn tại một hình chữ nhật  1 chứa ít nhất 3
2 2
điểm trong 49 điểm đã cho. Tam gi{c có ba đỉnh l| 3 điểm nằm trong hình chữ nhật này có diện tích

1
nhỏ hơn 
2

Bài 30. Chia mỗi cạnh của tam gi{c th|nh 8 đoạn thẳng

bằng nhau. Nối c{c điểm chia đó bằng c{c đoạn thẳng song A

song với các cạnh của tam gi{c. Ta được c{c tam gi{c đều
O
có cạnh bằng 1 K I

Số tam gi{c đều là 1  3  15  82  64 G

Đặt ngẫu nhiên 193 điểm vào 64 tam giác này ( 193 : 64  3 C
H
B

dư 1) Theo nguyên lí dirichlet thì sẽ có ít nhất 1 tam giác

đều có ít nhất 4 điểm. Xét tam gi{c đều này, gọi G là trọng

tâm của tam giác, từ G vẽ c{c đoạn thẳng vuông góc đến

các cạnh, tạo thành 3 tứ giác bằng nha

Tác giả: Nguyễn Công Lợi TÀI LIỆU TOÁN HỌC


53
Website:tailieumontoan.com

Đặt ngẫu nhiên 4 điểm vào tam giác này theo nguyên lí Dirichlet sẽ có một

tứ giác chứa ít nhất 2 điểm. Mà tứ giác này nội tiếp trong đường tròn đường kính

GA nên khoảng cách của chúng d  AG Mà ta có

AB 3 2 2 AB 3 3 3
AH   GA  .AH  .  . Do đó d 
2 3 3 2 3 3
Bài 31. Lấy 1 điểm A bất kì trong 403 điểm đó vẽ đường tròn tâm A bán kính 1.

+ Nếu 4032 điểm còn lại thuộc hình tròn thì b|i to{n được chứng minh.

+ Nếu tất cả 4032 điểm đó nằm ngo|i hinh tròn thì ta lấy 1 điểm B bất kì trong số đó vẽ

đường tròn t}m B b{n kính 1.

+ Nếu 4031 điểm còn lại thuộc hình tròn thì b|i to{n được chứng minh.

+ Nếu trong đó có một điểm C không thuộc cả 2 hình tròn thì AC  1; AB  1; BC  1 điều

n|y tr{i với giả thiết đầu b|i.

Suy ra 4033 điểm đó phải thuộc hình tròn t}m A hoặc t}m B. Như vậy theo nguyên lý

Dirichlet thì tồn tại 1 hình tròn có b{n kính 1 chứa ít nhất 2016 điểm

Bài 32. Gọi 2015 điểm trên mặt phẳng lần lượt là A1 ; A2 ; A3 : ...A 2015 . Vẽ c{c đường tròn

tâm O bán kính lần lượt là OA1 ; OA2 ; OA3 ;...OA2015 . Ta gọi điểm A1 gần

điểm O hơn A j nếu OAi  OA j . Giả sử A1 l| điểm gần O nhất thì ta có nhiều nhất 2015 đường

tròn tâm OO. Vẽ hai đường thẳng vuông góc với nhau tại O thì hai đường thẳng đó sẽ chia mặt

phẳng làm 4 phần. Từ 4 phần trên ở mỗi phần ta chọn điểm xa O nhất ở từng phần rồi từ 4 điểm

ấy ta vẽ đường tròn theo yêu cầu đề bài thì 44 đường tròn ấy sẽ phủ kín toàn bộ 2015 điểm trên

hoặc sẽ phủ hầu hết c{c điểm và vẫn còn một v|i điểm chưa được phủ. Tuy nhiên c{c điểm chưa

được phủ sẽ nằm trong vùng cắt của ba đường tròn. Khi đó từ một trong c{c điểm đó ta có thể vẽ

một đường tròn bao phủ 2015 điểm còn lại.

Bài 33. Giả sử 6 đội bóng đó l| A, B, C, D, E, F. Xét đội A, theo nguyên lý Dirichlê ta suy ra A phải

đấu hoặc không đấu với ít nhất 3 đội khác. Không mất tính tổng quát ta giả sử A đã đấu với B, C,

D.

+ Nếu B,C, D từng cặp chưa đấu với nhau thì b|i to{n được chứng minh.

+ Nếu B, C, D có hai đội đã đấu với nhau, ví dụ B v| C thì 3 đội A, B, C từng cặp đã đấu với nhau.

Tác giả: Nguyễn Công Lợi TÀI LIỆU TOÁN HỌC


54
Website:tailieumontoan.com
Như vậy bất cứ lúc n|o cũng có 3 đội trong đó từng cặp đã đấu với nhau hoặc chưa đấu với nhau

trận nào.

Bài 34. Gọi (O) l| đường tròn ngoại tiếp lục gi{c đều có cạnh bằng 2, khi đó (O) có bán kính

R  2. Gọi ABCD là hình vuông ngoại tiếp (O). Cạnh của hình vuông này bằng 4. Chia hình

vuông thành 16 hình vuông nhỏ, có cạnh bằng 1.

Rõ ràng 16 hình vuông này chứa 81 điểm đã cho. Vì 81  16.5  1 nên theo nguyên lý

Dirichlet sẽ tồn tại hình vuông cạnh bằng 1 chứa ít nhất 6 điểm trong số c{c điểm đã cho.

Tác giả: Nguyễn Công Lợi TÀI LIỆU TOÁN HỌC


55
Website:tailieumontoan.com
BÀI TẬP TỰ LUYỆN

Bài 1. Có 15 đội bóng tham dự giải vô địch quốc gia theo thể thức đấu vòng tròn một lượt. Chứng

minh rằng tại bất kì thời điểm nào của giải ta luôn tìm được 2 đội có cùng số trận đấu bằng nhau

tại thời điểm đó(có thể là 0 trận).

Bài 2. Một bà mẹ chiều con nên ng|y n|o cũng cho con ăn ít nhất một chiếc kẹo. Để hạn chế, mỗi

tuần b| cho con không ăn qu{ 12 chiếc kẹo. Chứng minh rằng trong một số ngày liên tiếp n|o đó

bà mẹ đã cho con tổng số 20 chiếc kẹo.

Bài 3. Chứng minh rằng trong 2001 người bất kỳ, luôn có ít nhất hai người có số người quen bằng

nhau(số người quen chỉ tính trong nhóm)

Bài 4. Trong một thời gian nọ của một lớp học Toán có một nhóm gồm 5 học sinh mà cứ mỗi người

trong nhóm n|y thì rơi v|o trong trạng thái ngủ gục trong lớp đúng 2 lần. Với mỗi cặp học sinh,

đều có cả hai cùng ngủ gục một lần. Chứng minh rằng tại một thời điểm n|o đó có ba học sinh

trong nhóm đó đồng thời ngủ gục .

Bài 5. Có 5 người đấu cờ với nhau. Hãy x{c định kết quả của tất cả các trận đấu nếu biết rằng mỗi

người chơi một lần với 4 người kia và số điểm của mỗi người nhận được đều khác nhau. Ngoài ra:

a) Người xếp thứ nhất không hoà trận nào.

b) Người xếp thứ nhì không thua trận nào.

c) Người xếp thứ tư không thắng trận nào.

Bài 6. Các học sinh được phát bài kiểm tra với mỗi môn một bài và trong n( n  3 ) môn học. Biết

rằng với một môn học bất kỳ có đúng 3 học sinh đạt điểm tối ưu, còn với hai môn tuỳ ý thì có

đúng 1 học sinh đạt điểm tối ưu cho mỗi môn trong cả hai môn đó. Hãy x{c định số n bé nhất sao

cho từ c{c điều kiện trên có thể suy ra rằng có đúng 1 học sinh đạt điểm tối ưu cho mỗi môn trong

cả n môn học.

 
Bài 7. Cho m máy tính và n máy in m  n mỗi sợi dây cáp chỉ nối được một máy tính và một

máy in. Tại một thời điểm bất kỳ mỗi máy tính chỉ có thể điều khiển được một m{y in v| người lại

mỗi máy in chỉ in được cho một máy tính. Hỏi phải dùng ít nhất là bao nhiêu sợi d}y c{p để n máy

tính bất kỳ có thể đồng thời in được?

Tác giả: Nguyễn Công Lợi TÀI LIỆU TOÁN HỌC


56
Website:tailieumontoan.com
Bài 7. Kì thi tuyển sinh v|o trường THPT chuyên Long An năm nay có 529 học sinh đến từ 16 địa

phương kh{c nhau tham dự. Giả sử điểm bài thi môn Toán của mỗi học sinh đều là số nguyên lớn

hơn 4 v| bé hơn hoặc bằng 10. Chứng minh rằng luôn tìm được 6 học sinh có điểm môn Toán

giống nhau v| cùng đến từ một địa phương.

Bài 8. Xét 20 số nguyên dương đầu tiên 1, 2, 3, , 20. Hãy tìm số nguyên dương k nhỏ nhất có

tính chất: Với mỗi cách lấy ra k số phân biệt từ 20 số trên, đều lấy được hai số phân biệt a và b sao

cho a  b là một số nguyên tố.

 
Bài 9. Cho tập hợp X  1; 2; 3;...; 2024 . Chứng minh rằng trong 45 số khác nhau bất kỳ

được lấy ra từ tập X luôn tồn tại hai số x, y sao cho x  y  1 .

 
Bài 10. Chứng minh rằng tồn tại số nguyên dương n thỏa mãn 13579 n  1 chia hết cho 313579 .

Bài 11. Trong một cái bát hình vuông cạnh 18 cm có 128 hạt vừng. Chứng minh rằng tồn tại hai hạt

vừng có khoảng cách tới nhau nhỏ hơn 2 cm.

Bài 12. Bên trong tam gi{c đều ABC cạnh 1 đặt 5 điểm. Chứng minh rằng tồn tại 2 điểm có khoảng

cách nhỏ hơn 0,5.

Bài 13. Cho hình tròn có bán kính n, ở đ}y n l| số nguyên dương. Trong hình tròn có 4n đoạn

thẳng đều có độ dài bẳng 1. Cho trước một đường thẳng d. Chứng minh rằng tồn tại đường thẳng

d’ hoặc song song với d, hoặc là vuông góc với d sao cho d’ cắt ít nhất hai đoạn thẳng đã cho.

Bài 14. Cho một bảng có kích thước 2n  2n ô vuông. Người ta đ{nh dấu vào 3n ô bất kì của

bảng. Chứng minh rằng có thể chọn ra n hàng và n cột của bảng sao cho c{c ô được đ{nh dấu đều

nằm trên n hàng và n cột này.

Bài 15. Chứng minh rằng trong mọi đa gi{c lồi với số cạnh chẵn, tồn tại đường chéo không song

song với một cạnh nào của đa gi{c.

Bài 16. Một hình lập phương có cạnh bằng 15 chứa 11000 điểm. Chứng minh rằng có một hình cầu

bán kính 1 chứa ít nhất 6 điểm trong số 11000 điểm đã cho.

Bài 17. Giả sử 1 bàn cờ hình chữ nhật có 3x7 ô vuông được sơn đen hoặc trắng. Chứng minh rằng

với c{ch sơn m|u bất kì, trong bàn cờ luôn tồn tại hình chữ nhật gồm các ô ở 4 góc là các ô cùng

màu.

Tác giả: Nguyễn Công Lợi TÀI LIỆU TOÁN HỌC


57
Website:tailieumontoan.com
Bài 18. Trong một tờ giấy hình vuông bằng giấy có cạnh bằng 12 cm có 31 lỗ kim châm. Chứng

minh rằng ta vẫn có thể cắt từ tờ giấy này ra một hình tròn có bán kính 1 cm mà không chứa một

lỗ kim châm nào.

Bài 19. Cho hình tròn (C) có diện tích bằng 8, đặt 17 điểm phân biệt bất kì. Chứng minh rằng bao

giờ cũng tìm được ít nhất ba điểm tạo thành một tam giác có diện tích bé hơn 1.

Bài 20. Trong hình vuông cạnh bằng 15 đặt 20 hình vuông nhỏ cạnh bằng 1 và từng đôi một không

cắt nhau. Chứng minh rằng trong hình vuông lớn có thể đặt một hình tròn bán kính 1 sao cho nó

không cắt hình vuông nào.

Bài 21. Trong mặt phẳng cho tập S gồm 8065 điểm đôi một phân biệt mà diện tích của mỗi tam giác có 3

đỉnh thuộc tập S đều không lớn hơn 1 (quy ước nếu 3 điểm thẳng hàng thì diện tích của tam giác tạo bởi

3 điểm này bằng 0). Chứng minh rằng tồn tại một tam giác T có diện tích không lớn hơn 1 chứa ít nhất

2017 điểm thuộc tập S (mỗi điểm trong số 2017 điểm đó nằm trong hoặc nằm trên cạnh của tam giác T).

Bài 22. Cho tam gi{c đều MNP có cạnh bằng 2 cm. Lấy n điểm thuộc các cạnh hoặc ở phía trong

tam gi{c đều MNP sao cho khoảng cách giửa hai điểm tuỳ ý lớn hơn 1 cm (với n là số nguyên

dương). Tìm n lớn nhất thoả mãn điều kiện đã cho.

Bài 23. Trên mặt phẳng cho 25 điểm phân biệt và trong ba điểm bất kì bao giờ cũng tìm được hai

điểm có khoảng cách giữa chúng nhỏ hơn 1. Chứng minh rằng tồn tại một hình tròn có bán kính

bằng 1 chứa không ít hơn 13 điểm trong c{c điểm trên.

Bài 24. Cho điểm P nằm trong đa gi{c lồi 2n cạnh. Vẽ c{c đường thẳng đi qua P và mỗi đỉnh của

đa gi{c. Chứng minh rằng bao giờ cũng tìm được một cạnh của đa gi{c sao cho không một đường

thẳng n|o trong c{c đường thẳng trên có điểm chung với cạnh đó.

Bài 25. Cho 19 điểm phân biệt nằm trong một tam gi{c đều có cạnh bằng 3, trong đó không có 3

điểm nào thẳng hàng. Chứng minh rằng luôn tìm được một tam gi{c có 3 đỉnh l| 3 trong 19 điểm đã

3
cho mà có diện tích không lớn hơn .
4

Bài 26. Trong hình vuông cạnh bằng 1 cho 5 điểm bất kỳ. Chứng minh rằng, trong các

2
điểm đã cho có thể tìm được 2 điểm sao cho khoảng cách giữa chúng không lớn hơn
2

Tác giả: Nguyễn Công Lợi TÀI LIỆU TOÁN HỌC


58
Website:tailieumontoan.com

Bài 27. Cho tam giác nhọn ABC có BAC  600 và BC  2 3cm . Bên trong tam giác này

cho 2017 điểm bất kì. Chứng minh rằng trong 2017 điểm ấy luôn tìm được 169 điểm mà khoảng

cách giữa hai điểm trong chúng không lớn hơn 1cm.

Bài 28. Trên mặt phẳng cho năm điểm phân biệt sao cho không có ba điểm nào thẳng hàng và

không có bốn điểm nào thuộc cùng một đường tròn. Chứng minh rằng tồn tại một đường tròn đi

qua ba điểm trong năm điểm đã cho v| hai điểm còn lại có đúng một điểm nằm bên trong đường

tròn

Bài 29. Trong hình chữ nhật có chiều dài và rộng lần lượt bằng 4 v| 3 cho 49 điểm, trong đó không

có ba điểm nào thẳng hàng. Chứng minh rằng tồn tại một tam gi{c có c{c đỉnh thuộc 49 điểm trên

1
mà diện tích nhỏ hơn 
2

Bài 30. Trong tam gi{c đều có cạnh bằng 8 đặt 193 điểm ph}n biệt. Chứng minh tồn tại 2

3
điểm trong 193 điểm đã cho có khoảng c{ch không vượt qu{ .
3
Bài 31. Trên cùng một mặt phẳng cho 4033 điểm, biết rằng 3 điểm bất kì trong 4033 điểm trên luôn

chọn được hai điểm có khoảng cách nhỏ hơn 1. Chứng minh rằng trong c{c điểm nói trên có ít

nhất 2016 điểm nằm trong đường tròn bán kính 1.

Bài 32. Trong mặt phẳng cho 2015 điểm. Mỗi điểm là tâm một đường tròn đi qua một điểm cố định

O. Chứng minh rằng từ những hình tròn tạo ra có thể chọn được 5 hình tròn mà chúng phủ tất cả

2015 điểm.

Bài 33. Có 6 đội bóng thi đấu với nhau(mỗi đội phải đấu 1 trận với 5 đội khác). Chứng minh rằng

vào bất cứ lúc n|o cũng có 3 đội trong đó từng cặp đã đấu với nhau hoặc chưa đấu với nhau trận

nào.

Bài 34. Bên trong hình lục gi{c đều có cạnh bằng 2 cho 81 điểm phân biệt. Chứng minh rằng tồn tại

một hình vuông có cạnh bằng 1 (kể cả biên) chứa ít nhất 6 điểm trong số c{c điểm đã cho.

Tác giả: Nguyễn Công Lợi TÀI LIỆU TOÁN HỌC


59
Website:tailieumontoan.com
HƯỚNG DẪN GIẢI

Bài 1. Số lần gặp nhau mà mỗi đội có, có thể nhận 15 giá trị kh{c nhau: 0; 1; 2; <<<..; 14.

Trong trường hợp này không thể áp dụng nguyên tắc Dirichlet được vì số đội cũng l| 15.

Hai trường hợp 0 trận và 14 trận không thể xảy ra đồng thời vì nếu có một đội n|o chưa

đấu trận n|o thì đồng thời không thể có một đội n|o đó đã đấu hết 14 trận, ngược lại nếu

có một đội đã đ{ 14 trận thì không thể có 1 đội chưa đ{ một trận nào. Vì vậy số lần gặp

nhau mà mỗi đội đã thực hiện trong thực tế có thể nhận thêm 14 giá trị từ 0 đến 13 hoặc từ

1 đến 14. Khi đó theo nguyên tắc Dirichlet ta luôn có thể tìm được hai đội có cùng một số

trận đấu .
Bài 2. Xét 21 ngày liên tiếp kể từ một ngày thứ hai n|o đó. Gọi S(n) là tổng số kẹo mà bà mẹ đã cho


con tính đến ngày thứ n 1  n  21 . 

Ta có S  m   S  n  , m  n 1  m, n  21 và 1  S  n   3.12  36 .

Vì có 21 ngày và chú ý rằng 0  S  m   S  n   36 nên tồn tại m  1 sao cho

S  m   S  n  mod 20   S  m   S  n  20  S  m   S  n   20

Như vậy từ ngày n  1 đến ngày thứ m, bà mẹ đã cho con tổng cộng đúng 20 chiếc kẹo.

Bài 3. Gọi số người quen của A i là a i , khi đó ta có 0  a i  2001 với 1  i  2001 . Xét c{c trường

hợp:

 Tồn tại một người trong 2001 không quen ai, suy ra không có ai quen cả 2000 người còn lại trong
nhóm.

Khi đó đó ta có 0  a i  1999 với 1  i  2001 từ đó suy ra tồn tại hai số a k  a m với

1  k,m  2001 hay tồn tại hai người có số người quen bằng nhau

 Mỗi người đều quen ít nhất một người suy ra, khi đó ta có 0  ai  2000 với 1  i  2001 từ đó

suy ra tồn tại hai số a k  a m với 1  k,m  2001 hay tồn tại hai người có số người quen bằng

nhau

Vậy b|i to{n được chứng minh.

Tác giả: Nguyễn Công Lợi TÀI LIỆU TOÁN HỌC


60
Website:tailieumontoan.com
Bài 4. Giả sử ngược lại rằng không hề có chuyện 3 học sinh đồng thời ngủ gục. Ta sẽ chứng minh

điều này mâu thuẫn.

Thật vậy, trong khoảng thời gian có hai người đồng thời ngủ gục, 3 người còn lại tỉnh táo.

Theo đề bài, mỗi học sinh trong nhóm đều ngủ gục đúng hai lần nên một trong hai người(đang

ngủ gục) sẽ có lúc lại ngủ gục với một trong 3 người còn lại.

Như vậy nhiều nhất sẽ có tất cả là 9 khoảng thời gian diễn ra ngủ gục từng cặp. Nhưng

nhóm này có 5 học sinh nên số cặp là học sinh có thể ra là 10, mà chỉ có nhiều lắm là 9 khoảng thời

gian. Do vậy sẽ có ít nhất một cặp không đồng thời ngủ gục. Ta có điều mâu thuẫn.

Bài 5. Theo điều kiện của bài ra ta thấy ngay người xếp thứ nhất thắng người xếp thứ ba, thứ tư,

thứ năm v| được tất cả 3 điễm. Còn người thứ nhì thắng người xếp thứ nhất. Người thứ nhì hoà

trong các trận đấu với người xếp thứ ba, thứ tư, thứ năm v| nhận 2,5 điểm.

Những người còn lại chỉ nhận số điểm lớn nhất lần lượt là 2; 1; 5; 1. Ta chứng minh họ

không thể nhận ít hơn.

Thật vậy, vì có 5 người nên họ chơi tất cả 10 trận và nhận tất cả 10 điểm. Nhưng người xếp

thứ nhất và thứ nhì đã nhận 5,5 điểm nên ba người còn lại nhận 4,5 điểm. Mặt khác

2  1,5  1  4,5 nên họ không thể nhận ít hơn.

Như vậy, do người thứ tư không thắng trận nào nên anh ta hoàvới người xếp thứ ba và thứ

năm. Còn lại người thứ ba thắng người thứ năm.

Bài 6. Ta biểu thị mỗi học sinh bằng một điểm trong mặt phẳng sao cho không có ba điểm nào

thẳng hàng. Nếu hai học sinh đạt điểm tối ưu ở một môn n|o đó, ta nối hai điểm tương ứng lại với

nhau. Khi đó, theo đề bài, mỗi môn học sẽ cho tương ứng duy nhất một tam giác vàbất cứ hai tam

gi{c n|o cũng có đúng một đỉnh chung.

Chú ý rằng nếu như bốn tam giác có chung một đỉnh thì tất cả c{c tam gi{c đều có chung

đỉnh đó, bởi vì nếu không thì tam giác thứ năm sẽ có chung đỉnh với mỗi một trong bốn tam giác

đó. Như vậy tam giác thứ năm n|y sẽ có bốm đỉnh, điều này mâu thuẩn.

Bây giờ nếu n  8 thì một tam giác sẽ có chung một đỉnh với mỗi một trong 7 tam giác còn lại.

Theo nguyên lí Dirichlet thì một trong c{c đỉnh của nó sẽ có chung đỉnh với ít nhất ba tam giác

khác, tức là tồn tại 4 tam giác có chung một đỉnh.

Tác giả: Nguyễn Công Lợi TÀI LIỆU TOÁN HỌC


61
Website:tailieumontoan.com
Cuối cùng ví dụ sau đ}y chứng tỏ rằng trường hợp n  7 không thỏa mãn đề bài. Trong

bảng dưới đ}y ta dùng dấu chéo () để chỉ học sinh đạt điểm tối ưu ở môn học tương ứng

Học

sinh 1 2 3 4 5 6 7

Môn học

I x x x

II x x x

III x x x

IV x x x

V x x x

VI x x x

VII x x x

Như vậy giá trị nhỏ nhất của n là 8.

Bài 7. Ta xét một cách nối thoả mãn đề b|i như sau: Với n m{y tính đầu tiên mỗi máy nối với một

máy in, còn với m  n máy tính còn lại, mỗi máy nối với tất cả n máy in.

Khi đó số dây cáp cần dùng trong cách nối này là S  n  n  m – n   n  m  n  1

Ta sẽ chứng minh rằng nếu số dây cáp S  n  m  n  1 thì không thoả mãn điều kiện đầu bài.

Thật vậy, nếu S  n  m  n  1 thì có ít nhất một m{y in x n|o đó được nối với không quá m  n

máy tính. Từ đó suy ra rằng có m máy tín mà trong số đó có m{y n|o nối với m{y in x, điều này có

nghĩa l| m{y tính đó không thể n|o đồng thời in được.

Tóm lại số sợi dây cáp ít nhất cần phải dùng là S  n  m  n  1

Bài 7. Ta có 529 học sinh có điểm bài thi từ 5 điểm đến 10 điểm. Theo nguyên lý Dirichlet ta có 89

học sinh có điểm b|i thi như nhau (từ 5 điểm đến 10 điểm).

Tác giả: Nguyễn Công Lợi TÀI LIỆU TOÁN HỌC


62
Website:tailieumontoan.com
Ta có 89 học sinh có điểm b|i thi như nhau v| đến từ 16 địa phương. Theo nguyên lý

Dirichlet tìm được 6 em có cùng điểm thi môn to{n v| đến từ cùng một địa phương.

Bài 8. Xét tập hợp 2; 4; 6; 8;10;12;14;16;18; 20  , ta thấy tổng của hai phần tử bất kì của tập hợp

n|y đều không phải là số nguyên tố. Do đó k  11 , ta sẽ chứng minh k  11 là số nhỏ nhất thỏa

mãn yêu cầu bài toán. Thật vậy, ta chia tập hợp A  1; 2; 3;...; 20 thành 10 cặp số sau:

1,2  ,  3,16  ,  4,19  ,  5,6  , 7,10  , 8,9  , 11,20  , 12,17  , 13,18  , 14,15
Tổng của hai số trong mỗi cặp số trên là số nguyên tố. Khi đó mỗi tập con của A có 11 phần

tử thì tồn tại ít nhất hai phần tử thuộc cùng vào một trong 10 cặp số trên. Suy ra trong A luôn có

hai phần tử phân biệt có tổng là một số nguyên tố.

Bài 9.

Chia 2012 số 1; 2; 3;...; 2024 th|nh 44 đoạn gồm  1; 3  ,  4; 8  ,...,  1936; 2024  .
     
 
 k  1
2
C{c đoạn trên có dạng tổng quát là  k 2 ;  1 .
 

Như vậy 45 số thuộc tập hợp X nằm trong 44 đoạn trên. Theo nguyên lí Dirichlet thì tồn tại hai số

trong 45 số trên nằm trong cùng một đoạn. Không mất tính tổng quát ta giả sử hai số đó l| x, y v|

 
 k  1
2
chúng nằm trong đoạn  k 2 ;  1 .
 

 k  1  k  1
2 2
Khi đó ta có x  y   1  k2   k2  k  1  k  1 .

Từ đó ta có điều phải chứng minh.

Bài 10. Đặt a  313579 , khi đó ta được  a,13579   1 .

Xét a  1 số có dạng 13579;135792 ;135793 ;...;13579a 1 .

Chia a  1 số trên cho a  313579 ta được a  1 số dư . M| trong phép chia cho a thì có a số dư. Như

vậy Trong a  1 số dư như trên tồn tại hai số dư bằng nhau hay tong tại hai số trong dãy số trên có

cùng số dư khi chia cho a.

Giả sử hai số đó l| 13579m và 13579 với m  p .


p

Tác giả: Nguyễn Công Lợi TÀI LIỆU TOÁN HỌC


63
Website:tailieumontoan.com

  
Khi đó ta được 13579m  13579p a hay 13579p 13579 m p  1 a . 
  
Do  a,13579   1 nên a,13579 p  1 , suy ra 13579m p  1 a
 
Điều n|y có nghĩa l| tông tại số có dạng 13579 n  1 chia hết cho 313579 .

Bài 11. Lấy mỗi hạt vừng làm tâm dựng hình tròn bán kính 1 cm. Các hình tròn này nằm hoàn

toàn trong hình vuông có cạnh 20cm thu được từ hình vuông đã cho bằng cách tịnh tiến bốn cạnh

của nó một khoảng 1cm ra phía ngoài. Tổng diện tích của các hình tròn bán kính 1cm này là 128 

> 402,112 > 400. Do đó tổng diện tích các hình tròn này lớn hơn diện tích hình vuông cạnh 20 cm.

Bài 12. C{c đường trung bình của tam giác đều cạnh 1 sẽ chia nó ra l|m 4 tam gi{c đều cạnh 0,5.

Do đó trong một tam giác nhỏ đó có ít nhất 2 điểm đã cho, v| c{c điểm đó không thể rơi v|o c{c

đỉnh của tam giác ABC.Vậy khoảng cách giữa hai điểm đó nhỏ hơn 0,5.

Bài 13. Giả sử AB l| đoạn thẳng có độ dài bằng 1, a v| a’ l| hai đường thẳng bất kì vuông góc với

nhau. Gọi A’B’ v| A‛B‛ l| c{c hình chiếu của AB lên a v| a’. Khi đó ta có: A’B’  A‛B‛  AB hay

A’B’  A‛B‛  1.

Áp dụng vào bài toán ta gọi d‛ l| đường thẳng bất kì vuông góc với d. Chiếu vuông góc tất cả 4n

đoạn thẳng lên d v| d‛. từ (1) suy ra tổng độ dài hình chiếu của tất cả 4n đoạn thẳng không bé hơn

4n.

Vì vậy, theo nguyên lí Dirichlet trong hai đường thẳng d v| d‛ có ít nhất một đường thẳng mà

tổng độ dài của hình chiếu c{c đoạn thằng lên nó không bé hơn 2n. Không mất tính tổng quát ta có

thể giả sử đó l| d.

Mặt khác, mỗi đoạn thẳng đầu nằm trọn trong hình tròn b{n kính n (đường kính 2n), nên hợp các

hình chiếu của chúng trên d có độ d|i không vượt quá 2n.

Vì vậy, theo nguyên lí dirichlet trên d tồn tại ít nhất một điểm M thuộc vào hình chiếu của ít nhất

hai đoạn thẳng trong số 4n đoạn thẳng đã cho. Gọi d’ l| đường thẳng vuông góc với d tại M.

Đường thẳng d’ chính l| đường thẳng cần tìm.

Bài 14. Chọn ra n hàng có chứa số ô được đ{nh dấu nhiều trên c{c h|ng đó nhất. Ta chứng minh

rằng c{c ô được đ{nh dấu còn nhỏ hơn hoặc bằng n.

Tác giả: Nguyễn Công Lợi TÀI LIỆU TOÁN HỌC


64
Website:tailieumontoan.com
Giả sử ngược lại không phải như vậy, tức là

số ô được đ{nh dấu lớn hơn hoặc bằng n  1

. Số các hàng còn lại chưa chọn là n. Vậy theo

nguyên lí Dirichlet sẽ có ít nhất một hàng (

tỏng số n hàng còn lại) chứa ít nhất hai ô đã

đ{nh dấu. Chú ý rằng theo cách chọn thì n

h|ng đã chọn có chứa số ô được đ{nh dấu

nhiều trên c{c h|ng đó nhất. Có một hàng

còn lại chưa chọn có ít nhất hai ô đ{nh dấu,

nên suy ra mọi hàng trong số n h|ng đã chọn

đều có ít nhất hai ô được chọn, tức là trên n

h|ng đã chọn có không ít hơn 2n ô đã được

đ{nh dấu.

Như vậy, số ô được đ{nh dấu lớn hơn hoặc bằng 2n   n  1  3n . Vô lí vì chỉ có 3n ô được đ{nh

dấu. Vậy nhận xét được chứng mình.

Như vậy, sau khi đã chọn ra n hàng(với cách chọn như trên), theo nhận xét còn lại có không quá n

ô được đ{nh dấu. Vì thế cùng lắm là có n cột chứa chúng. Vì lẽ đó sẽ không thấy còn ô đ{nh dấu

nào nằm ngoài các hàng hay cột được chọn.

n  n  3
Bài 15. Ta giả thiết rằng nếu một đa gi{c có n cạnh thì có đường chéo.
2

Xét một đa gi{c lồi bất kì với số cạnh là chẵn (đa gi{c lồi 2k cạnh với k  2 ).

2k  2k  3 
Khi đó số đường chéo của nó là s  .
2


Ta có s  k  2k – 3   2k  k – 2   k nên suy ra s  2k k – 2 . 
Giả sử ngược lại đa gi{c n|y có tính chất mỗi đường chéo của nó đều song song với một cạnh nào

 
đó của đa gi{c. Đa gi{c n|y có 2k cạnh, vì thế từ s  2k k – 2 suy ra tồn tại ít nhất k  1 đường

chéo d1 ,d2 ,d3 ,...,dk1 m| c{c đường chéo này cùng song song với một cạnh a n|o đó của tam giác

Tác giả: Nguyễn Công Lợi TÀI LIỆU TOÁN HỌC


65
Website:tailieumontoan.com
đã cho. Thật vậy, nếu ngược lại mỗi cạnh tối đa l| song song k  2 đường chéo thì tối đa ta chỉ có

k  k  2  đường chéo và s  2k  k – 2  . Điều này mâu thuẫn với (1).

Như thế ta có k đường thẳng song song với nhau a,d1 ,d2 ,d3 ,...,dk1 .

Mặt kh{c đa gi{c đã cho l| đa gi{c lồi nên c{c đường chéo d1 ,d2 ,d3 ,...,dk1 cùng nằm trên một

nửa mặt phẳng bờ x{c định cạnh a. Không mất tính tổng quát có thế cho d1 l| đường chéo xa nhất

đối với a. Ta có tất cả k đoạn thẳng phân biệt, nên mỗi đỉnh của đa gi{c đều l| đầu mút của một

đoạn n|o đó trong k đoạn trên. Từ đó suy ra to|n bộ đa gi{c nằm hẳn về một nửa mặt phẳng xác

định bởi d1 . Do d1 l| đường chéo, nên điều này mâu thuẫn với tính lồi của đa gi{c. Vậy giả thiết

phản chứng là sai.

Từ đó ta có điều phải chứng minh.

Bài 16. Chia mỗi cạnh của hình lập phương th|nh 13 phần bằng nhau. Như thể hình lập phương

đã cho được chia thành 133  2197 hình lập phương nhỏ. Do 11000  5.2197 – 10985 , nên tồn tại

ít nhất 1 hình lập phương nhỏ, mà hình lập phương n|y chứa ít nhất 6 điểm. Như đã biết, nếu gọi

1
cạnh hình l}p phương bằng a, thì hình cầu ngoại tiếp có bán kính R với R  a 3.
2

15
Vì thế hình cầu ngoại tiếp hình lập phương nhỏ(cạnh của nó là ) được x{c định là
13

2
1 15 1  15  1 675 1
R 3 3    4 1
2 13 2  13  2 169 2

Hình cầu b{n kính R n|y dĩ nhiên chứa ít nhất 6 điểm trong số 11000 điểm đã cho.

Bài 17. Mẫu sơn m|u có thể xảy ra với bàn cờ này có dạng từ 1 đến 8. Giả sử một trong số các cột

thuộc dạng 1. Bài toán sẽ được chứng minh nếu tất cả các cột còn lại thuộc dạng 1, 2, 3 hoặc 4. Giả

sử tất cả các cột còn lại thuộc dạng 5, 6, 7, 8. Khi đó theo nguyên lí Dirichlet 2 trong số 6 cột có 2 cột

cùng 1 dạng v| như vậy b|i to{n cũng được chứng minh.

Chứng minh ho|n to|n tương tự nếu 1 cột có dạng 8. Giả sử không có cột nào trong các cột

1, 8 thì theo nguyên lí Dirichlet cũng có 2 cột cùng dạng và b|i to{n cũng đựơc chứng minh.

Bài 18. Lấy mỗi lỗ kim là tâm dựng một hình tròn bán kính 1cm. Tổng diện tích của 31 hình tròn

này sẽ là 31 nhỏ hơn diện tích của hình vuông cạnh 10 cm . Do đó phải có một điếm M trong
Tác giả: Nguyễn Công Lợi TÀI LIỆU TOÁN HỌC
66
Website:tailieumontoan.com
hình vuông cạnh 10 cm(l| hình vuông thu được từ hình vuông cạnh 12 cm đã cho bằng cách thu

hẹp các chiều 1cm) và không nằm trong 31 hình tròn b{n kính được dựng như đã trình b|y ở trên.

Lấy điểm M làm tâm ta cắt một hình tròn bán kính 1cm, thì hình tròn này nằm hoàn toàn trong

hình vuông đã cho có cạnh dài 12 cm và không chứa một lỗ kim châm nào cả.

Bài toán tổng quát có thể được phát biểu như sau: Trong một tờ giấy hình vuông có cạnh

bằng b có a lỗ kim ch}m. (trong đó b  ar 2 ). Chứng minh rằng ta vẫn có thể cắt từ tờ giấy này

ra một hình tròn có bán kính r cm mà không chứa một lỗ kim châm nào.

Bài 19. Chia hình tròn thành (C) thành 8 hình quạt bằng nhau, mỗi hình quat có diện tích bằng 1.

Theo nguyên lý Dirichlet, tồn tại ít nhất một hình quạt (a) chứa 3 điểm trong số 17 điểm đã cho.

Tam gi{c có 3 đỉnh l| 3 điểm đó nằm trọn trong hình quạt (a) nên có diện tích nhỏ hơn diện tích

hình quạt, tức l| bé hơn 1.

Bài 20. Xét hình gồm tất cả c{c điểm cách hình vuông nhỏ cạnh 1 một khoảng không lớn hơn 1. Rõ

ràng hình tròn bán kính 1 có tâm nằm ngo|i hình đó nên không thể cắt hình vuông nhỏ. Diện tích

hình đó bằng 5   . Tâm hình tròn cần tìm cũng cần phải cách các cạnh của hình vuông lớn hơn

 
một khoảng lớn hơn 1, tức là ở bên trong hình vuông cạnh 13. Vì 20 5    132 . Hình tròn có

tâm tại điểm không bị phủ sẽ có tính chất thỏa mãn đề bài.

Bài 21. Gọi d l| khoảng c{ch hai điểm Ai , A j xa nhất trong tất cả c{c điểm thuộc tập hợp

S. Giả sử A k có khoảng c{ch đến đường thẳng Ai A j lớn nhất. Khi đó tam gi{c Ai A jAk có

S Ai AjAk  1 v| lớn nhất. Từ c{c điểm Ai , A j và A k vẽ c{c đường thẳng song song với c{c

cạnh của tam gi{c Ai A jAk thì ta thu được bốn tam gi{c con bằng nhau v| một tam gi{c

lớn. Tam gi{c lớn có diện tích không vượt qu{ 4 đơn vị. Tam gi{c n|y chứa 8065 điểm đã

cho. Vì 8065 : 4  2016 dư 1. Nên theo nguyên lí Dirichlet thì tồn tại một tam gi{c con chứa

2017 điểm thuộc tập hợp S thỏa mãn đề b|i.

Bài 22. Tam gi{c đều có cạnh bằng 2 cm thì diện tích bằng 3 cm2 , tam gi{c đều có cạnh bằng 1

3 3
cm thì diện tích bằng cm2 . Nếu tam gi{c đều có cạnh lớn hơn 1cm thì diện tích lớn hơn
4 4
cm2

Tác giả: Nguyễn Công Lợi TÀI LIỆU TOÁN HỌC


67
Website:tailieumontoan.com
Gọi t là số tam gi{c đều có cạnh lớn hơn hoặc bằng 1cm chứa được trong tam gi{c đều có cạnh 2

cm:

1  t  4 (với t là số nguyên dương). Suy ra t max  3 .

Theo nguyên lý Drichlet sẽ có 1 trong t tam gi{c đều có cạnh lớn hơn 1cm đó chứa tối đa 2

điểm thoả mãn khoảng cách giữa hai điểm bất kỳ luôn lớn hơn 1cm.

Vậy số điểm thoả yêu cầu bài toán là 2  n  4 nên giá trị lớn nhất của n 4

Bài 23. Xét hai điểm A v| B trong 25 điểm đã cho thỏa mãn điều kiện AB có độ dài lớn nhất. Ta xét

c{c trường hợp sau:

 Trường hợp 1: Với AB  1 . Khi đó mọi điểm C bất kì trong 25 điểm đã cho ta đều có
AC  AB  1 . Suy ra toàn bộ 25 điểm trên cùng nằm trong đường tròn tâm A có bán kính bằng 1.

 Trường hợp 2: Với AB  1 . Khi đó xét điểm C trong số c{c điểm còn lại.

Theo giả thiết với ba điểm A, B, C ta luôn có AC  1 hoặc BC  1 .

Như vậy với c{c điểm còn lại có ít nhất 12 đoạn thẳng xuất phát từ A có độ dài nhỏ nhơn 1 hoặc

có ít nhất 12 điểm xuất phát từ B có độ dài nhỏ hơn 1. Vẽ đường tròn tâm A bán kính bằng 1 hoặc

đường tròn tâm B bán kính bằng 1.

Suy ra đường tròn tâm A hoặc đường tròn tâm B chứa ít nhất 13 điểm tròn số c{c điểm đã cho.

Từ đó b|i to{n được chứng minh.

Bài 24. Xét đa gi{c lồi 2n cạnh A1A2 A3 ...A2n . Ta xét c{c trường hợp sau:

 Trường hợp 1: Điểm P nằm trên một đường chéo của đa gi{c A1A2 A3 ...A2n .

Khòn mất tình tổng quát ta giả sử P nằm trên đường chéo Ai A j của đa gi{c. Khi đó hai đường

thẳng PAi và PA j trùng nhau và không cắt phần trong của bất cứ cạnh nào của đa gi{c. Khi đó

2n  2 chỉ cắt tối đa 2n  2 cạnh của đa gi{c. Vậy có ít nhất hai cạnh của đa gi{c thỏa mãn yêu cầu
của bài toán.

 Trường hợp 2: Điểm P không nằm trên bất kì đường chéo nào của đa gi{c A1A2 A3 ...A2n . Vẽ

đường chéo A1An 1 , khi đó mỗi phía của đường chéo chứa n cạnh của đa gi{c. Không mất tính

Tác giả: Nguyễn Công Lợi TÀI LIỆU TOÁN HỌC


68
Website:tailieumontoan.com
tổng quát ta giả sử điểm P nằm trong đa gi{c A1A2 A3 ...An 1 . Khi đó n  1 đường thẳng

PAn1 ; PAn2 ; PAn3 ;...; PA2n không thể cắt các cạnh An1An2 ; An2 An2 ;...; A2n A1 . Còn lại

n  1 đường thẳng cắt phần trong tối đa của n  1 cạnh trong n cạnh n|y. Như vậy tồn tại một
trong n cạnh An1An2 ; An2 An2 ;...; A2n A1 thỏa mãn yêu cầu bài toán.

Bài 25. Giả sử 19 điểm nằm trong tam gi{c đều ABC cạnh bằng 3. Chia tam giác ABC thành 9 tam

gi{c đều, có cạnh bằng 1 (gọi là tam giác nhỏ) như hình vẽ.

3
Mỗi tam giác nhỏ có diện tích là S  . Để ý là 19  2.9  1
4

Vì có 19 điểm nằm trong 9 tam giác nhỏ nên theo nguyên lý Dirichlets thì có ít nhất 3 điểm cùng

thuộc một hình tam giác nhỏ. Giả sử 3 điểm đó l| I1 ,I 2 ,I 3 .

3
Khi đó tam gi{c I1I 2 I 3 nằm trong một tam giác nhỏ nên S I I I  .
1 2 3
4

Bài 26. Chia hình vuông đã th|nh 4 hình vuông con. Dễ d|ng tính được, cạnh của một hình vuông

1 2
con là v| đường chéo là . Gieo 5 điểm đã cho v|o hình vuông ban đầu, 5 điểm đó sẽ nằm
2 2
trong 4 hình vuông con. Theo nguyên tắc Dirichlets thì tồn tại 2 điểm nằm trong cùng một hình

vuông. V| do đó khoảng cách giữa 2 điểm đó sẽ không lớn hơn đường chéo của hình vuông chứa

2
nó, tức là không lớn hơn .
2

Bài 27. Gọi O l| t}m đường tròn ngoại tiếp ΔABC. Kẻ OH, OK, OG lần lượt vuông góc với

c{c cạnh AB, AC, BC. Dễ d|ng chứng minh được c{c tứ gi{c AHOK, BHOG, KOGC nội

tiếp c{c đường tròn đường kính OA  OB  OC  2 cm . Ta có 2017 điểm nên theo nguyên

lí Diriclet thì sẽ tồn tại một tứ gi{c có chứa ít nhất 673 điểm, giả sử đó l| tứ gi{c OKCG.

Xét tứ gi{c OKCG. Gọi I l| trung điểm OC nên I l| t}m đường tròn ngoại tiếp tứ gi{c

OKCG

Khi đó ta được IO  IK  IC  IG  1 cm . Kẻ IM, IN, IP, IQ lần lượt vuông góc với OK, KC,

CG, GO. Suy ra bốn tứ gi{c OMIQ, MKNI, INCP, PGQI nội tiếp c{c đường tròn đường

kính bằng 1 cm

Tác giả: Nguyễn Công Lợi TÀI LIỆU TOÁN HỌC


69
Website:tailieumontoan.com
m| có 673 điểm. Như vậy theo nguyên lí Diriclet thì sẽ tồn tại một tứ gi{c có chứa ít nhất

169 điểm, giả sử đó l| tứ gi{c MKNI. Khi đó 169 điểm n|y sẽ thuộc đường tròn ngoại tiếp

tứ gi{c MKNI có đường kính IK  1cm . Do đó khoảng c{ch 169 điểm n|y không lớn hơn

1cm

Bài 28. Giả sử 5 điểm đó l| A, B, C, D, E. Vì trong năm điểm n|y không có ba điểm n|o

thẳng h|ng nên ta tồn tại 2 điểm trong năm điểm đã cho sao cho đường thẳng đi

qua 2 điểm n|y chia mặt phẳng th|nh hai nửa mặt phẳng v| 3 điểm còn lại nằm cùng về

một nửa mặt phẳng bờ l| đường thẳng trên. Giả sử đó l| 2 điểm A và B. Xét 3 góc sau

ACB; ADB;AEB . Khi đó do trong 5 điểm không có bốn điểm n|o cùng thuộc một đường

tròn nên ta ho|n to|n có thể sắp thứ tự chúng như sau: ACB  ADB  AEB . Khi đó dễ

thấy điểm E nằm bên trong đường tròn ngoại tiếp tam giác ADB còn điểm C nằm bên

ngo|i đường tròn ngoại tiếp tam gi{c ADB.(chứng minh đơn giản dựa theo c{ch tính số đo

của góc có đỉnh nằm trong, nằm ngo|i đường tròn)


1
Bài 29. Chia hình chữ nhật 4  3 thành 24 hình chữ nhật  1 , mỗi hình chữ nhật có diện tích là
2
1 1
 Vì có 49 điểm nằm trong 24 hình chữ nhật nên tồn tại một hình chữ nhật  1 chứa ít nhất 3
2 2
điểm trong 49 điểm đã cho. Tam gi{c có ba đỉnh l| 3 điểm nằm trong hình chữ nhật này có diện tích

1
nhỏ hơn 
2

Bài 30. Chia mỗi cạnh của tam gi{c th|nh 8 đoạn thẳng

bằng nhau. Nối c{c điểm chia đó bằng c{c đoạn thẳng song A

song với các cạnh của tam gi{c. Ta được c{c tam gi{c đều
O
có cạnh bằng 1 K I

G
Số tam gi{c đều là 1  3  15  82  64
C B
H

Đặt ngẫu nhiên 193 điểm vào 64 tam giác này ( 193 : 64  3

dư 1)

Theo nguyên lí dirichlet thì sẽ có ít nhất 1 tam giác

đều có ít nhất 4 điểm.

Tác giả: Nguyễn Công Lợi TÀI LIỆU TOÁN HỌC


70
Website:tailieumontoan.com
Xét tam gi{c đều này, gọi G là trọng tâm của tam giác, từ G vẽ c{c đoạn thẳng vuông góc

đến các cạnh, tạo thành 3 tứ giác bằng nhau

Đặt ngẫu nhiên 4 điểm vào tam giác này theo nguyên lí Dirichlet sẽ có một

tứ giác chứa ít nhất 2 điểm. Mà tứ giác này nội tiếp trong đường tròn đường kính

GA nên khoảng cách của chúng d  AG Mà ta có

AB 3 2 2 AB 3 3 3
AH   GA  .AH  .  . Do đó d 
2 3 3 2 3 3
Bài 31. Lấy 1 điểm A bất kì trong 403 điểm đó vẽ đường tròn tâm A bán kính 1.

+ Nếu 4032 điểm còn lại thuộc hình tròn thì b|i to{n được chứng minh.

+ Nếu tất cả 4032 điểm đó nằm ngo|i hinh tròn thì ta lấy 1 điểm B bất kì trong số đó vẽ

đường tròn t}m B b{n kính 1.

+ Nếu 4031 điểm còn lại thuộc hình tròn thì b|i to{n được chứng minh.

+ Nếu trong đó có một điểm C không thuộc cả 2 hình tròn thì AC  1; AB  1; BC  1 điều

n|y tr{i với giả thiết đầu b|i.

Suy ra 4033 điểm đó phải thuộc hình tròn t}m A hoặc t}m B. Như vậy theo nguyên lý

Dirichlet thì tồn tại 1 hình tròn có b{n kính 1 chứa ít nhất 2016 điểm

Bài 32. Gọi 2015 điểm trên mặt phẳng lần lượt là A1 ; A2 ; A3 : ...A 2015 .

Vẽ c{c đường tròn t}m O b{n kính lần lượt l| OA1 ; OA2 ; OA3 ;...OA2015 .

Ta gọi điểm A1 gần điểm O hơn A j nếu OAi  OA j

Giả sử A1 l| điểm gần O nhất thì ta có nhiều nhất 2015 đường tròn t}m OO.

Vẽ hai đường thẳng vuông góc với nhau tại O thì hai đường thẳng đó sẽ chia mặt phẳng

làm 4 phần.

Từ 4 phần trên ở mỗi phần ta chọn điểm xa O nhất ở từng phần rồi từ 4 điểm ấy ta vẽ

đường tròn theo yêu cầu đề b|i thì 44 đường tròn ấy sẽ phủ kín to|n bộ 2015 điểm trên

hoặc sẽ phủ hầu hết c{c điểm v| vẫn còn một v|i điểm chưa được phủ. Tuy nhiên c{c

điểm chưa được phủ sẽ nằm trong vùng cắt của ba đường tròn. Khi đó từ một trong c{c

điểm đó ta có thể vẽ một đường tròn bao phủ 2015 điểm còn lại.

Tác giả: Nguyễn Công Lợi TÀI LIỆU TOÁN HỌC


71
Website:tailieumontoan.com
Bài 33. Giả sử 6 đội bóng đó l| A, B, C, D, E, F. Xét đội A, theo nguyên lý Dirichlê ta suy ra A phải

đấu hoặc không đấu với ít nhất 3 đội khác. Không mất tính tổng quát ta giả sử A đã đấu với B, C,

D.

+ Nếu B,C, D từng cặp chưa đấu với nhau thì b|i to{n được chứng minh.

+ Nếu B, C, D có hai đội đã đấu với nhau, ví dụ B v| C thì 3 đội A, B, C từng cặp đã đấu với nhau.

Như vậy bất cứ lúc n|o cũng có 3 đội trong đó từng cặp đã đấu với nhau hoặc chưa đấu với nhau

trận nào.

Bài 34. Gọi (O) l| đường tròn ngoại tiếp lục gi{c đều có cạnh bằng 2, khi đó (O) có bán kính

R  2. Gọi ABCD là hình vuông ngoại tiếp (O). Cạnh của hình vuông này bằng 4. Chia hình

vuông thành 16 hình vuông nhỏ, có cạnh bằng 1.

Rõ ràng 16 hình vuông này chứa 81 điểm đã cho. Vì 81  16.5  1 nên theo nguyên lý

Dirichlet sẽ tồn tại hình vuông cạnh bằng 1 chứa ít nhất 6 điểm trong số c{c điểm đã cho.

Tác giả: Nguyễn Công Lợi TÀI LIỆU TOÁN HỌC


72
Website:tailieumontoan.com
Chủ đề 2

CÁC BÀI TOÁN ỨNG DỤNG NGUYÊN LÝ CỰC HẠN

I. Nguyên lý cực hạn

Nguyên lí cực hạn được phát biểu đơn giản như sau:

Nguyên lí 1: Trong một tập hữu hạn và khác rỗng các số thực luôn luôn có thể chọn được

số bé nhất và số lớn nhất.

Nguyên lí 2: Trong một tập khác rỗng các số tự nhiên luôn luôn có thể chọn được số bé

nhất.

Sử dụng nguyên lí cực hạn là một phương ph{p được vận dụng cho nhiều lớp bài toán

kh{c, đặc biệt nó có ích khi giải các bài toán tổ hợp. Trong quá trình tìm kiếm lời giải nhiều bài

toán, sẽ rất có lợi nếu chúng ta xem xét các phần tử biên, phần tử giới hạn n|o đó, tức là phần tử

mà tại đó mỗi đại lượng hình học cá thể nhận giá trị lớn nhất hoặc giá trị nhỏ nhất, chẳng hạn như

cạnh lớn nhất, cạnh nhỏ nhất của một tam giác, góc lớn nhất hoặc góc nhỏ nhất của một đa gi{c <.

Những tính chất của các phần từ biên, phần tử giới hạn nhiều khi giúp chúng ta tìm kiếm được lời

giải thu gọn của bài toán.

Nguyên lí cực hạn thường được sử dụng kết hợp với c{c phương ph{p kh{c, đặc biệt là

phương ph{p phản chứng, được vận dụng trong trong trường hợp tập các giá trị cần khảo sát chỉ

tập hợp hữu hạn( nguyên lí 1) hoặc có thể có vô hạn nhưng tồn tại một phần tử lớn nhất hoặc nhỏ

nhất (nguyên lí 2).

Khi vận dụng nguyên lí này, ta phải tiến h|nh c{c bước sau:

 Bước 1. Chứng minh rằng trong tất cả các giá trị cần khảo sát luôn tồn tại giá trị lớn nhất hoặc
giá trị nhỏ nhất.

 Bước 2. Xét b|i to{n trong trường hợp riêng khi nó nhận giá trị này (nhỏ nhất hoặc lớn nhất)
 Bước 3. Chỉ ra một mâu thuẫn, chỉ ra một giá trị còn nhỏ hơn (hay lớn hơn) gi{ trị ta đang khảo
sát .

Theo nguyên lí của phương ph{p phản chứng, ta sẽ suy ra điều phải chứng minh.

II. Một số ví dụ minh họa

Tác giả: Nguyễn Công Lợi TÀI LIỆU TOÁN HỌC


73
Website:tailieumontoan.com
Ví dụ 1. Chứng minh rằng bốn hình tròn đường kính là bốn cạnh của một tự giác lồi thì phủ kín

miền tứ giác ABCD.

Lời giải

Lấy M là một điểm tùy ý của tứ giác lồi ABCD. Có hai


B
khả năng xảy ra:

 Nếu M nằm trên biên của đa gi{c (tức M nằm trên C

một cạnh của tứ gi{c ABCD). Khi đó M nằm trong hình


M
tròn có đường kính là cạnh ấy. Trong trường hợp này A

kết luận của bài toán hiển nhiên đúng. D

 Nếu M nằm bên trong tứ giác lồi ABCD .

Khi đó ta có AMB  BMC  CMD  DMA  3600

Theo nguyên lí cực hạn thì trong các góc

AMB, BMC,CMD, DMA luôn tồn tại một góc có số


đo lớn nhất.

 
Giả sử MaxBMC  AMB, BMC,CMD, DMA . Khi đó BMC  900

Từ đó suy ra M nằm trong (hoặc cùng lắm là nằm trên) đường tròn đường kính BC. Vậy dĩ nhiên

M bị phủ bởi đường tròn n|y. Như thế do M l| điểm tùy ý của tứ giác ABCD, ta suy ra bốn hình

tròn nói trên phủ kín tứ giác lồi đã cho. Vậy ta có điều phải chứng minh.

Ví dụ 2. Cho 2011 đường thẳng phân biệt, trong đó ba đường thẳng bất kì trong số chúng thì đồng

quy. Chứng minh rằng cả 2011 đường thẳng đã cho đồng quy tại một điểm.

Lời giải

l
B Q C D

Tác giả: Nguyễn Công Lợi TÀI LIỆU TOÁN HỌC


74
Website:tailieumontoan.com
Ta sẽ đi giải quyết bài toán bằng phương ph{p phản chứng: Giả sử ngược lại c{c đường

thẳng đã cho không đi qua một điểm. Ta xét c{c giao điểm tạo nên bởi 2011 đường thẳng đã cho.

Xét tất cả các khoảng cách khác 0 hạ từ c{c giao điểm này tới các đường thẳng đã cho. Giả sử A là

một giao điểm trong số đó v| gọi AQ là khoảng cách nhỏ nhất trong số đó vẽ từ A đến đường

thẳng trong số 2011 đường thẳng. Qua A theo giải thiết, phải có ít nhất l| 3 đường thẳng, và 3

đường thẳng này cả lần lượt tại B, C và D. Vẽ AQ vuông góc với , thì hai trong ba điểm B, C,

D phải nằm cùng một phía của điểm Q, chẳng hạn là C và D. Không mất tính tổng quát, giả sử

QC  QD .

Vẽ CP vuông góc với AD và QK vuông góc với AD. Suy ra CP  QK  AQ . Điều này là vô lý vì

trái với giả thiết giả sử AQ là khoảng cách bé nhất. Điều vô lí trên chứng tỏ rằng 2011 đường thẳng

đã cho đồng quy tại một điểm.

Ví dụ 3. Một nước có 80 sân bay mà khoảng cách giữa hai s}n bay n|o cũng kh{c nhau. Mỗi máy

bay cất cánh từ một s}n bay v| bay đến sân bay nào gần nhất. Chứng minh rằng trên bất kỳ sân

bay n|o cũng không thể có qu{ 5 m{y bay đến.

Lời giải

Từ giả thiết suy ra nếu các máy bay từ c{c s}n bay M v| N đến sân bay O thì khoảng cách MN là

lớn nhất trong các cạnh của tam gi{c MON, do đó ta được MON  600 .

Giả sử rằng các máy bay bay từ các sân bay M1 ; M2 ; M3 ; M4 ;...; Mn đến sân bay O thì một trong

3600
các góc Mi ON j không lớn hơn với i, j, n 1;2;3;4;5;...;80 vì tổng c{c góc đã cho bằng
n
3600 .

3600
Như vậy ta có  600 do đó n  6 . Suy ra điều phải chứng minh.
n

Ví dụ 4. Trong tam giác ABC có ba góc nhọn. Lấy một điểm P bất kì trong tam giác. Chứng minh

khoảng cách lớn nhất trong các khoảng cách từ điểm P đến c{c đỉnh A, B, C của tam giác không

nhỏ hơn 2 lần khoảng cách bé nhất trong các khoảng cách từ điểm P đến các cạnh của tam gi{c đó.

Lời giải

Tác giả: Nguyễn Công Lợi TÀI LIỆU TOÁN HỌC


75
Website:tailieumontoan.com
Dựng PA1 ,PB1 ,PC1 tương ứng vuông góc với các A

cạnh BC, CA, AB. Vì tam giác ABC có ba góc nhọn nên
A1
c{c điểm A1 , B1 ,C1 tương ứng nằm trong đoạn BC, CA B1

và AB. P

Nối PA, PB, PC ta có


B C1 C
APC1  C1PB  BPA1  A1PC  CPB1  B1PA  360 0

Suy ra góc lớn nhất trong 6 góc này không thế nhỏ hơn

600 .

Không mất tính tổng quát, ta giả sử APC1 là lớn nhất, khi đó APC1  600 .Xét APC1 vuông tại

PC1 1
C1 , khi đó ta có  cosAPC1  cos600 
AP 2

Từ đó ta được AP  PC1 . Nếu thay PA bằng khoảng cách lớn nhất trong các khoảng cách từ P đến

c{c đỉnh và thay PC1 bằng khoảng cách ngắn nhất từ P tới các cạnh thì bất đắng thức c|ng được

thỏa mãn. Vậy b|i to{n được chứng minh.

Ví dụ 5. Cho tứ giác lồi ABCD có hai đường chéo AC và BD cắt nhau tại E. Chứng minh rằng nếu

các bán kính của 4 đường tròn nội tiếp các tam giác EAB, ECD, EDA mà bằng nhau thì tứ giác

ABCD là hình thoi.

Lời giải

Hoàn toàn không mất tính tổng quát ta có thể giả B C

sử rằng CE  AE; BE  DE . Gọi B1 và C1 tương

ứng l| c{c điểm đối xứng của B và C qua tâm E, ta C1 B1

có tam giác C1EB1 nằm trong miền tam giác AED.


A D
Giả sử đoạn thẳng AD không trùng với đoạn

thẳng C1 B1 .

Khi đó đường tròn nội tiếp tam giác AED nằm bên trong đường tròn nội tiếp tam gi{c AED, đồng

dạng với đường tròn này với t}m đồng dạng E, hệ số đồng dạng lớn hơn 1.

Như vậy rAED  rC EB  rCBE ( rAED l| b{n kính đường tròn nội tiếp tam giác AED) .
1 1

Điều này vô lí vì trái với giả thiết là rAED  rCEB .


Tác giả: Nguyễn Công Lợi TÀI LIỆU TOÁN HỌC
76
Website:tailieumontoan.com
Như vậy chứng tỏ là A trùng với C1 và D trùng với B1

Khi đó OA  OC và OB  OD nên từ giác ABCD là hình bình hành.

Trong hình bình hành ABCD có p1r  SAEB  S BEC  p2r , trong đó p1 và p2 tương ứng là nửa chu

vi của các tam giác AEB và BEC.

AB  BE  EA BC  CE  BE
Suy ra ta được p1  p2 hay  nên AB  BC
2 2

Hình bình hành ABCD có AB  BC nên ABDC là hinh thoi.

Ví dụ 6. Trên mặt phẳng cho 2  2000 điểm, trong đó không có bất kỳ 3 điểm nào thẳng hàng.

Người ta tô 2011 điểm bẳng m|u đỏ v| tô 2011 điểm còn lại bằng màu xanh. Chứng minh rằng bao

giờ cũng tồn tại một cách nối tất cả c{c điểm m|u đỏ với tất cả c{c điểm màu xanh bởi 2011 đoạn

thẳng không có điểm nào chung.

Lời giải

Ta nhận thấy rằng luôn tồn tại cách nối 2011 cặp điểm với
Y
A
nhau bằng 2011 đoạn thẳng và vì có 2011 cặp điểm nên số

cách nối là hữu hạn và nếu dùng tổ hợp thì ta có thể tính

được con số chính xác các cách nối. Và hiển nhiên là trong
X
hữu hạn cách nối đó ta luôn tìm ra được một cách nối có
B
tổng độ d|i c{c đoạn thẳng là ngắn nhất. Ta chứng minh

cách nối đó l| c{ch m| chúng ta cần tìm.

Thật vậy, giả sử ngược lại ta có hai đoạn thẳng AX và BY mà cắt nhau tại điểm O (giả sử A và B tô

m|u đỏ, còn X v| Y tô m|u xanh). Khi đó, nếu ta thay đoạn thẳng AX và BY bằng hai đoạn AY và

BX, c{c đoạn còn lại giữ nguyên thì ta có cách nối này có tính chất:

AY  BX   AO  OY    BO  OX    AO  OX    BO  OY 

Từ đó ta được AY  BX  AX  BY . Như vậy, việc thay hai đoạn thẳng AX và BY bằng hai đoạn

thẳng AY và BX , ta nhận được một cách nối mới có tổng độ d|i đoạn thẳng là nhỏ hơn. Điều này

vô lý vì trái với giả thiết l| đã chọn cách nối có tổng c{c độ dài là bé nhất.

Điều đó chứng tỏ cách nối có tổng độ d|i c{c đoạn thẳng là ngắn nhất l| không có điểm chung.

Tác giả: Nguyễn Công Lợi TÀI LIỆU TOÁN HỌC


77
Website:tailieumontoan.com
Ví dụ 7. Cho tứ giác ABCD thỏa mãn điều kiện b{n kính c{c đường tròn nội tiếp bốn tam giác

ABC, BCD, CDA và DAB bằng nhau. Chứng minh tứ giác ABCD là hình chữ nhật.

Lời giải

Giả sử rABC  rBCD  rCDA  rDAB . Vẽ các hình B B'

bình h|nh ABB’C v| ADD’C.

Suy ra tứ gi{c BB’D’C l| hình bình h|nh . E


A
C
Do đó ABC  B'CB và

ADC  D'CD suy ra rABC  rB'CD và


D D'
rADC  rD'CD

Mặt khác ABD  CB' D' nên rABD  rCB'D'

Theo giả thiết rABC  rBCD  rCDA  rDAB nên rB'CB  rCB'D'  rD'CD  rCBD

Gọi E l| giao điểm của BD’ v| DB’. Ta chứng minh C trùng với E.

Giả sử C khác E suy ra C thuộc vào một trong bốn tam gi{c EBD, EBB’, EB’D’, ED’D. Giả sử C

thuộc vào miền t}m gi{c BDE, khi đó rBCD  rBED  rB'ED  rCB'D' , điều này vô lí.

Như vậy chứng tỏ E trùng với C. Suy ra B, C, D’ thẳng h|ng v| D, C, B’ thẳng hàng.

Ta có D’C song song với AD suy ra BC song song với AD.

Vì CB’ song song với AB nên DC song song với AB. Suy ra ABCD là hình bình hành.

1
Ta lại có S ABD  S ADC  S (vì ABCD là hình bình hành).
2 ABCD

AB  BD  DA AD  DC  CA
Do đó ta được rABD .  rADC .
2 2

Hay l| ta được AB  BD  DA  AD  DC  CA nên BD  CA

Vậy ABCD là hình chữ nhật.

Ví dụ 8. Bên trong đường tròn tâm O bán kính R  1 có cho 7 điểm phân biệt. Chứng minh rằng

tồn tại ít nhất hai điểm trong số chứng mà khoảng cách giữa hai điểm này nhỏ hơn 1.

Lời giải

Tác giả: Nguyễn Công Lợi TÀI LIỆU TOÁN HỌC


78
Website:tailieumontoan.com
Nhận thấy rằng có ít nhất 7 điểm trong số 8 điểm đã cho l|

khác tâm O. Ta gọi c{c điểm đó l| A2


A1
A1 ,A2 ,A3 ,A4 ,A5 ,A6 ,A7 ,A8 . O

Ta có góc nhỏ nhất trong số c{c góc đỉnh O v| hai điểm còn lại

3600
Ai OAk là không lớn hơn  600 với i  k,1  i,k  8
7

Không mất tính tổng quát ta giả sử A1OA2 là góc bé nhất.

OA A  600 OA 2  A1A 2


Xét A1OA2 ta có A1OA2  600 nên ta được  1 2

 A1A 2 O  600 OA1  A1A 2

Mà ta có OA1  1 hoặc OA2  1 suy ra A1A2  1 . Ta có điều phải chứng minh.

Ví dụ 9. Cho tứ giác ABCD ngoại tiếp đường tròn tâm O. Chứng minh rằng: Nếu c{c đường chéo

AC và BD giao nhau tại O thì tứ giác ABCD là hình thoi.

Lời giải

Không mất tính tổng quát ta có thể giả sử D

OA  OC và OB  OD . Gọi B’ v| C’ lần lượt là


c{c điểm đối xứng của B và C qua O nên
A C
OB  OB' và OC  OC' O

Bởi vì BC là tiếp tuyến của đường tròn (O) nên


B
B’C’ cũng tiếp xúc với đường tròn (O).

Từ đó ta được A trùng với C’ v| D trùng với B’,

suy ra AO  OC và OB  OD . Từ đ}y ta được tứ

giác ABCD là hình bình hành.

Mặt khác ABCD ngoại tiếp đường tròn (O) nên AB  CD  AD  BC , do đó AB  AD .

Do đó hình bình h|nh ABCD l| hình thoi.

Ví dụ 10. Trên mặt phẳng cho một số hữu hạn điểm không cùng nằm trên một đường thẳng.

Chứng minh rằng tồn tại ba điểm sao cho đường tròn đi qua ba điểm đó không chứa điểm nào ở

bên trong.

Tác giả: Nguyễn Công Lợi TÀI LIỆU TOÁN HỌC


79
Website:tailieumontoan.com
Lời giải

A1 C

Ak

A2

Ak+1
Ak Ak+1
Ap Ap-1

Vì số c{c điểm đã cho l| hữu hạn và chúng không cùng nằm trên một đường thẳng, nên khi lấy

bao lồi hệ điểm ta sẽ được một đa gi{c. Giả sử đó l| đa gi{c lồi A1A2 ...Ap . Như thế c{c điểm còn

lại đã cho phải nằm trong bao lồi. Gọi Ak , Ak1 l| hai đỉnh liên tiếp của của đa gi{c lồi (nghĩa l|

xét một cạnh tùy ý Ak Ak 1 ). Khi ấy mọi điểm đã cho đều nằm ở một nửa mặt phẳng x{c định bởi

Ak Ak 1 . Từ giả thiết suy ra tập hợp c{c điểm đã cho không thuộc Ak Ak1 là khác rỗng. Vì thế

theo nguyên lý cực hạn, tồn tại C sao cho Ak CAk1  MaxAk A1Ak 1 , ở đ}y gi{ trị lớn nhất lấy

theo mọi i  1, n mà

i  k,i  k  1 ( giả sử A1 ,A2 ,...,An là hệ hữu hạn điểm cho trước). Khi đó đường tròn ngoại tiếp

ta giác CAk Ak 1 l| đường tròn cần tìm

Ví dụ 11. Bên trong một hình vuông cạnh 1 cho n điểm sao cho không có ba điểm thẳng hàng.

Chứng minh rằng tồn tại môt tam gi{c có đỉnh tại c{c điểm đã cho v| diện tích S của nó thỏa mãn

1
bất đẳng thức S 
n2

Lời giải

A2 A3 Xét bao lồi của n điểm nằm bên trong hình vuông. Vì

không có ba điểm nào thẳng hàng, nên bao lồi l| đa

giác lồi có k đỉnh  k  n  , ngo|i ra c{c điểm đã cho


A1

hoặc l| đỉnh của đa gi{c lồi, hoặc nằm hẳn bên trong

đa gi{c lồi. Chỉ có hai khả năng xảy ra


An

 Nếu k  n , khi đó số đường chéo xuất phát từ A1

Tác giả: Nguyễn Công Lợi TÀI LIỆU TOÁN HỌC


80
Website:tailieumontoan.com
của đa gi{c bao lồi tạo thành cùng các cạnh của đa gi{c

n  2 tam giác. Gọi S là diện tích tam giác nhỏ nhất


trong n  2 tam giác ấy

Vì tổng các diện tích của n  2 tam giác nhỏ hơn 1(chú ý 1 l| diện tích hình vuông chứa chọn

1
n  2 tam giác này). Từ đó suy ra S  .
n2

 Nếu k  n , khi đó bên trong đa gi{c bao lồi A1A2 ...Ak có n  k điểm Ak1 ,Ak2 ,....,An . Nối

A k 1 với c{c đỉnh A1 ; A2 ;...; Ak . Khi đó có k tam gi{c Ak1A1A2 ; Ak1A2 A3 ;....; Ak1Ak A1

A2 A3 Vì không có ba điểm nào thẳng h|ng, nên c{c điểm

Ak 2 ,...,An phải nằm hẳn trong k tam giác nói trên.

A1
Ak+1
Giả sử A k  2 nằm hẳn trong tam gi{c n|o đó. Nối

A k  2 với ba đỉnh của tam giác này thì từ một tam

Ak giác sẽ có ba tam giác mới . Sau mỗi lần làm số tam

gi{c tăng lên 2. Như vậy ta đi đến

k  2  n  k – 1  2n – k – 2   n – 2    n – k  tam

giác mà bên trong mỗi tam gi{c n|y không có điểm

nào thuộc n điểm đã cho. Gọi S là diện tích bé nhất

trong các tam giác trên, thế thì

1 1
S 
 n – 2  n – k n  2
1
Bất đẳng thức S  được chứng minh.
n2

Nhận xét: Ta có một số b|i to{n tương tự.

Bài toán 1. Cho n điểm nằm trong tam giác ABC có diện tích là 1cm 2 . Chứng minh rằng từ n điểm

1
đó cùng với 3 điểm A, B, C luôn tồn tại một tam giác có diện tích không lớn hơn cm 2 .
3  2(n  1)

Tác giả: Nguyễn Công Lợi TÀI LIỆU TOÁN HỌC


81
Website:tailieumontoan.com

Bài toán 2. Cho n điểm nằm trong đa gi{c lồi m đỉnh có diện tích là 1cm 2 . Chứng minh rằng từ n

điểm đó cùng với m đỉnh của đa gi{c, luôn tồn tại một tam giác có diện tích không lớn hơn

1
cm 2 .
m  2(n  1)

Ví dụ 12. Cho tam giác ABC, lấy điểm C1 thuộc cạnh AB, A1 thuộc cạnh BC và B1 thuộc cạnh

1
CA. Biết rằng độ d|i đoạn thẳng AA1 , BB1 ,CC1 không lớn hơn 1. Chứng minh rằng S ABC  (
3

S ABC là diện tích tam giác ABC)

Lời giải

Ta xét c{c trường hợp sau đ}y A

+ Trường hợp 1: Tam giác ABC không tù, khi


C1
đó trong ba góc của tam giác ABC có ít nhất

một góc lớn lớn hơn hoặc bằng 600 . B1

Không mất tính tổng quát, giả sử

A  B  C  A  600 B A1 C

Kẻ c{c đường cao BD v| CE, khi đó ta được

1
S ABC  BD.AC
2
1 1 1
Mà BD  BB1  1 nên ta được S ABC  BD.AC  .BB1 .AC  .AC
2 2 2

1 1
Chứng minh tương tự ta được S ABC  .AB . Do đó ta có S 2ABC  .AB.AC .
2 4

1 1 3.AB.AC
Lại có S ABC  .AB.AC.sin A  .AB.AC.sin 600 
2 2 4

3.AB.AC 1
Do đó ta được S ABC   3.S 2ABC  S ABC 
4 3

+ Trường hợp 2: Tam giác ABC tù, không mất tính tổng quát ta giả sử A  900

- Khi đó nếu 900  A  1200 , chứng minh tương tự trương hợp tam giác ABC không từ.

Tác giả: Nguyễn Công Lợi TÀI LIỆU TOÁN HỌC


82
Website:tailieumontoan.com

- Khi A  1200 , trong tam giác ABB1 có A  900  AB1B nên ta được AB  BB1  1 .

Trong tam giác ACC1 có A  900  AC1C nên ta được AC  CC1  1 .

1 1 1 3 1
Ta có S ABC  .AB.CK  .AB.AC.sin KAC  .1.1.sin 600  
2 2 2 4 3

1
Vậy ta luôn có S ABC  .
3

Ví dụ 13. Trong mặt phẳng cho 2009 điểm sao cho 3 điểm bất kỳ trong chúng l| 3 đỉnh của một

tam giác có diện tích không lớn hơn 1. Chứng minh rằng tất cả những điểm đã cho nằm trong một

tam giác có diện tích không lớn hơn 4.

Lời giải

Trong số các tam giác tạo thành, xét tam giác ABC
A'
có diện tích lớn nhất (diện tích S). Khi đó S  1 .

Qua mỗi đỉnh của tam giác, kẻ c{c đường thẳng


C B
P
song song với cạnh đối diện, c{c đường thẳng này

giới hạn tạo thành một tam giác A' B'C' (hình vẽ).

Khi đó SA'B'C'  4SABC  4 . Ta sẽ chứng minh tất cả B' A C' P'

c{c điểm đã cho nằm trong A' B'C' .

Giả sử trái lại, có một điểm P nằm ngoài A' B'C' chẳng hạn như trên hình vẽ . Khi đó

d  P; AB   d  C; AB  , suy ra SPAB  SCAB , mâu thuẫn với giả thiết tam giác ABC có diện tích lớn

nhất. Vậy tất cả c{c điểm đã cho đều nằm bên trong A' B'C' có diện tích không lớn hơn 4.

Vì dụ 14. Cho 2015 điểm phân biệt trên mặt phẳng sao cho trong ba điểm bất kì luôn có hai điểm

cách nhau một khoảng không vượt quá 1. Chứng minh rằng tồn tại một đường tròn có bán kính

bằng 1 chứa ít nhất 1008 điểm.

Lời giải

Nối hai điểm bất kì trong c{c điểm trên ta được hữu hạn c{c đoạn thẳng, khi đó theo nguyên lí cực

hạn thì tồn tại đoạn thẳng có độ dài lớn nhất. Không mất tính tổng quát ta giả sử đoạn thẳng AB

có độ dài lớn nhất.

Tác giả: Nguyễn Công Lợi TÀI LIỆU TOÁN HỌC


83
Website:tailieumontoan.com
Lấy điểm M trong số 2013 điểm còn lại. Khi đó trong ba đoạn thẳng AB, AM, BM luôn tồn tại một

đoạn thẳng có độ d|i không vượt qu{ 1. Khi đó ta có c{c khả năng xẩy ra như sau:

+ Nếu AB  1 thì AM  1; BM  1 .

+ Nếu AB  1 thì AM  1 hoặc BM  1 .

Như vậy trong hai đoạn AM và BM có một đoạn thẳng có độ d|i không vượt qu{ 1. Như vậy điểm

M nằm trong đường tròn tâm A hoặc đường tròn tâm B có bán kính bằng 1. Như vậy một trong

hai đường tròn n|y chưa ít nhất 1008 điểm trong số 2015 điểm đã cho.

Ví dụ 15. Trong đường tròn tâm O có bán kính bằng 10 lấy 450 điểm phân biệt. Chứng minh rằng

luôn tìm được hai điểm có khoảng c{ch không vượt quá 1.

Lời giải

Nối hai điểm bất kì ta được một đoạn thẳng, do đó từ 450 điểm ta được hữu hạn c{c đoạn thẳng.

Như vậy theo nguyên lí cực hạn ta luôn tìm được đoạn thẳng ngắn nhất.

Không mất tính tổng quát ta giả sử đoạn thẳng ngắn nhất đó l| AB. Đặt AB  m .

m
Khi đó lấy c{c điểm đã cho l|m t}m đường tròn và lấy bán kính là thì c{c đường tròn nhận
2
450 m 2
được đôi một không cắt nhau. Khi đó tổng diện tích của 450 hình tròn đó l| .
2

Do 450 điểm nằm trong đường tròn nằm trong đường tròn b{n kính l| 10 nên 450 đường tròn trên
2
 m  m
nằm trong đường tròn bán kính  10 
2 
, đường tròn này có diện tích là  10  2   .
  

2
450 m 2  m
Từ đó ta có bất đẳng thức   10    , khi đó rút ra được m  1 . Từ đó ta có điều
2  2

phải chứng minh.

Ví dụ 16. Trên mặt phẳng cho một số điểm được tô bằng màu xanh và một số điểm được tô bằng

m|u đỏ sao cho hai điểm khác màu có khoảng c{ch không vượt quá 1. Chứng minh rằng tồn tại

1
một đường tròn có bán kính chứa tất cả c{c điểm m|u đỏ hoặc chứa tất cả c{c điểm màu
2
xanh.

Tác giả: Nguyễn Công Lợi TÀI LIỆU TOÁN HỌC


84
Website:tailieumontoan.com
Lời giải

Ta kí hiệu Đ l| đường tròn có bán kính nhỏ nhất chứa tất cả c{c điểm m|u đỏ v| X l| đường tròn

có bán kính bé nhất chứa tất cả c{c điểm màu xanh. Không mất tính tổng quát ta giả sử bán kính

của Đ không vượt quá bán kính của X.

Nếu tất cả c{c điểm m|u xanh đều nằm trong đường tròn Đ thì đường tròn nhỏ hơn đường tròn X

chứa tất cả c{c điểm m|u xanh. Điều này trái với giả thiết X l| đường tròn có bán kính bé nhất

chứa tất cả c{c điểm màu xanh.

Như vậy có một điểm A màu xanh nằm trên biên của đường tròn Đ hoặc nằm ngo|i đường tròn Đ.

1
Nếu bán kính của đường tròn Đ lớn hơn thì đường tròn tâm A bán kính bằng 1 chứa ít hơn
2
một nửa biên của đường tròn Đ. Ta có mội điểm m|u đỏ nằm trong phần chung của của đường

tròn Đ v| đường tròn tâm A bán kính bằng 1(vì khoảng cách từ A đến một điểm m|u đỏ không

vượt qu{ 1. Hình tròn có đường kính là dây chung của hai đường trong Đ v| (A, 1) chứa tất cả các

điểm m|u đỏ và nhỏ hơn đường tròn Đ, điều này mâu thuẫn với Đ l| đường tròn có bán kính nhỏ

nhất chứa tất cả c{c điểm m|u đỏ).

Vậy ta có điều phải chứng minh.

Ví dụ 17. Trên mỗi ô của bàn cờ 8x8 người ta viết một số sao cho mỗi số là trung bình cộng của các

số ở các ô liền kề(hai ô liền kề là hai ô có chung một cạnh). Chứng minh rằng tất cả các số viết trên

bàn cờ đều bằng nhau.

Lời giải

Ta thấy bàn cờ 8x8 có tất cả 64 ô trong đó có một số ô có bốn ô liền kề và một số có 3 ô liền kề.

Số các số điền trên các ô bàn cờ là hữu hạn. Khi đó trong c{c số diền trên ô bàn cờ tôn tại số nhỏ

nhất. Giả sử số nhỏ nhất đó l| m. Ta xét c{c trường hợp sau:

 Trường hợp 1: Nếu ô ghi số m có bốn ô liền kề.

Gọi số điền trong bốn ô liền kề với ô điền số m lần lượt là a, b, c, d.

a bcd
Khi đó theo b|i ra ta có m  hay 4m  a  b  c  d . Mặt khác theo cách chọn m là số
4
nhỏ nhất nên ta có a  m; b  m; c  m;d  m . Khi đó ta lại được a  b  c  d  4m .

Tác giả: Nguyễn Công Lợi TÀI LIỆU TOÁN HỌC


85
Website:tailieumontoan.com
Kết hợp hai kết quả trên ta suy ra được m  a  b  c  d . Như vậy nếu một ô được điền số m thì

các ô liền kề cùng được ghi số m. Do đó tất cả c{c ô đều được ghi số m.

 Trường hợp 2: Nếu ô ghi số m có ba ô liền kề. Lập luận ho|n to|n tương tự như trên ta suy ra
được tất cả c{c ô đều được ghi số m.

Vậy b|i to{n được chứng minh.

Ví dụ 18. Cho p là số nguyên tố lớn hơn 3 v| n l| số tự nhiên khác 0. Chứng minh rằng pn không

thể là tổng của hai lập phương của hai số nguyên dương kh{c nhau.

Lời giải

Giả sử n là số nguyên dương nhỏ nhất sao cho pn là tổng của hai lập phương của hai số nguyên

dương kh{c nhau. Tức là ta có pn  a 3  b3 với a, b  Z . Khi đó ta được


pn   a  b  a 2  ab  b2 . 

a  b  p
k


Do p là số nguyên tố nên từ p   a  b  a  ab  b
n 2 2
 ta được  2 n k
.
a  ab  b  p
2

Dễ thấy a  b  2 nên k  0 . Lại thấy a 2  ab  b2  ab  1 , do đó ta được n  k  0 hay n  k .

   a 
2
Lại có 3ab  a  b 2
 ab  b2  p2k  pn k , do đó ta được 3ab chia hết cho p.

Do p là số nguyên tố nên a chia hết cho p hoặc b chia hết cho p. Mà ta lại có a  b  pk nên ta được

a và b cùng chia hết cho p.

Đặt a  a1p; b  b1p với a1 ; b1  N* , khi đó từ pn  a 3  b3 ta được pn 3  a13  b13 , điều này trái

với giải sử n là số tự nhiên bé nhất thỏa mãn yêu cầu bài toán.

Vậy không thỏa mãn số tự nhiên n thỏa mãn yêu cầu bài toán.

Ví dụ 19. Cho 10 điểm phân biệt không có ba điểm nào thẳng hàng và nằm trong một tam giác có

cạnh bằng 2cm. Chứng minh rằng luôn tìm được ba điểm trong 10 điểm n|y sao cho ba điểm đó l|

3
ba đỉnh của một tam giác có diện tích không vượt quá cm 2 và có ít nhất một góc nhỏ hoặc
3

bằng 450 .

Tác giả: Nguyễn Công Lợi TÀI LIỆU TOÁN HỌC


86
Website:tailieumontoan.com
Lời giải

Nối t}m đường tròn ngoại tiếp tam gi{c đều với mỗi đỉnh của tam gi{c đều, khi đó ta được ba tam

3
giác mà mỗi tam giác có diện tích là cm 2 . Đặt 10 điểm v|o trong tam gi{c đều khi đó theo
3

3
nguyên lí Dirichlet thì tồn tại ít nhất bốn điểm cùng thuộc một tam giác có diện tích cm 2 , như
3

3
vậy luôn tồn tại một tam giác có diện tích không vượt quá cm 2 .
3

Để chứng minh tam gi{c đó có ít nhất một góc nhỏ hoặc bằng 450 ta gọi bốn điểm đó l| A, B, C, D

v| ta xét c{c trường hợp sau:

 Trường hợp 1: Nếu bốn điểm A, B, C, D là bốn điểm của một tứ giác lồi.

Ta có ABC  BCD  CDA  DAB  3600

Khi đó ta được ABD  DBC  BCA  ACD  CDB  BDA  DAC  CAB  3600 .

3600
Theo nguyên lí cực hạn thì trong tam góc trên tồn tại một góc bé nhất không vượt quá  450 .
8

Giả sử đó l| góc BAC  450 . Như vậy tam giác ABC thỏa mãn yêu cầu bài toán.

 Trường hợp 2: Nếu bốn điểm A, B, C, D không phải là bốn đỉnh của một tứ giác lồi.

Khi đó ta có BAC  CAD  DAB  3600 .

Khi đó trong ba góc trên tồn tại ít nhất một góc không nhỏ hơn 1200 . Giả sử góc đó l| BAC .

Khi đó trong tan gi{c ABC có ABC  ACB  600 . Như vậy trong hai góc ABC và ACB có một

góc không vượt quá 300 . Giả sử góc đó l| ABC  300  450 . Như vậy tam giác ABC thỏa mãn yêu

cầu bài toán.

Ví dụ 20. Cho 19 điểm trong đó không có ba điểm nào thẳng hàng, nằm trong một hình lục giác

đều có cạnh bằng 1. Chứng minh rằng luôn tồn tại một tam gi{c m| đỉnh l| ba trong 19 điểm trên

3
có ít nhất một góc không lớn hơn 450 và nằm trong đường tròn bán kính nhỏ hơn .
5

Lời giải
Tác giả: Nguyễn Công Lợi TÀI LIỆU TOÁN HỌC
87
Website:tailieumontoan.com
Vẽ c{c đường chéo của lục gi{c đều. C{c đường chéo này A

chia lục gi{c đều thành 6 tam giác bằng nhau mỗi cạnh tam

gi{c có độ dài bằng 1. Theo nguyên lí Dirichlet thì trong 19


B
điểm luôn tồn tại bốn điểm nằm tròn một tam gi{c đều.
D
Giả sử bốn điểm cùng nằm trong một tam gi{c đều là A, B,

C, D. Ta xét các vị trí của bốn điểm A, B, C, D theo các

trường hợp sau: C

 Trường hợp 1: Bốn điểm A, B, C, D tạo thành một tứ giác

lồi. Khi đó ta có A  B  C  D  3600 .

Như vậy trong bốn góc trên tồn tại một góc nhỏ hơn hoặc bằng 900 , giả sử đó l| góc A. Khi đó ta

có DAC  CAB  900 nên một trong hai góc DAC; CAB có một góc không lớn hơn 450 .

Như vậy một trong hai tam giác ADC và ABD có một góc không lớn hơn 450 .

 Trường hợp 2: Trong bốn điểm A, B, C, D có một điểm nằn trong tam gi{c có ba đỉnh l| ba điểm
còn lại. Giả sử điểm D nằm trong tam giác ABC.

+ Nếu BDC  900 thì ta được DBC  DCB  900 nên một A

trong hai góc DBC; DCB không lớn hơn 450 . Suy ra tam

giác BCD thỏa mãn yêu cầu bài toán.

+ Nếu BDC  900 thì ta được BAC  900 , do đó D

CAD  BAD  900

Từ đó ta được một trong hai góc CAD; BAD không lớn hơn B C

450 hay một trong hai tam giác ADC và ADB thỏa mãn yêu
cầu bài toán.

3
Mạt kh{c ta gi{c đều có cạnh bằng một nên b{n kính đường tròn ngoại tiếp tam gi{c đều là .
3

3 3
Mà  nên ta có điều phải chứng minh.
3 5

Tác giả: Nguyễn Công Lợi TÀI LIỆU TOÁN HỌC


88
Website:tailieumontoan.com
Ví dụ 21. Bên trong hình vuông cạnh 1 cho n điểm. Chứng minh rằng tồn tại một tam gi{c có đỉnh

tại c{c điểm đã cho hoặc đỉnh của hình vuông sao cho diện tích S của nó thỏa mãn bất đẳng thức

1
S
2(n  1)

Lời giải

B C B C B C
An

A1 A1
A1
A2
Ak

A3 A2

A2

A D A D A D

Gọi A, B, C, D là bốn đỉnh hình vuông và A1 ; A2 ;...; An l| n điểm nằm trong hình vuông. Nối A1

với 4 đỉnh A, B, C, D. Khi đó ta được 4 hình tam giác.

+ Nếu A 2 nằm trong một trong 4 tam gi{c đó (giả sử A 2 nằm trong tam giác ADA1 ) Ta nối A 2

với A, D và A1 . Sau khi nối xong thì số tam gi{c tăng thêm 2.

+ Nếu A 2 nằm trên cạnh chung nối A 2 với A v| C. Khi đó số tam gi{c cũng tăng thêm 2.

Như vậy trong mọi trường hợp, số tam giác sẽ tăng thêm 2. Với c{c điểm A3 ; A4 ;...; A n ta làm

tương tự.

Cuối cùng số tam gi{c được tạo thành là 4  2  n  1  2n  2 tam gi{c. C{c tam gi{c trên đều có

đỉnh l| đỉnh của hình vuông hoặc n điểm đã cho. Khi đó, tổng diện tích của 2n  2 tam giác này

bằng diện tích hình vuông(bằng 1).

Theo nguyên lý cực hạn thì tồn tại tam giác có diện tích nhỏ nhất trong 2n  2 tam giác ấy. Gọi

1
diện tích này là S thì S  . Ta có điều cần chứng minh.
2(n  1)

Ví dụ 22. Trong mặt phẳng cho s{u điểm A1 ,A2 ,...,A6 trong đó không có ba điểm nào thẳng

hàng. Với ba điểm bất kỳ trong số s{u điểm n|y luôn tìm được hai điểm mà khoảng cách giữa

Tác giả: Nguyễn Công Lợi TÀI LIỆU TOÁN HỌC


89
Website:tailieumontoan.com
chúng nhỏ hơn 671. Chứng minh trong số s{u điểm A1 ,A2 ,...,A6 đã cho, luôn tìm được ba điểm

l| ba đỉnh một tam giác có chu vi nhỏ hơn 2013.

Lời giải

Dễ thấy 6 điểm không thẳng hàng cho ta 20 tam giác. Mỗi tam giác có ít nhất 1 cạnh nhỏ hơn 671

(gọi là cạnh ngắn), nhưng như thế mỗi cạnh ngắn có thể đã tính 4 lần, vậy có ít nhất 5 cạnh ngắn.

Giả sử không có 3 cạnh ngắn nào cùng xuất phát từ 1 điểm khi đó chắc chắn sẽ tồn tại 1

đường gấp khúc nối liền 6 điểm bằng 5 đoạn ngắn. không mất tính tổng quát gọi đường gấp khúc

đó l| A1A2 A3 A4 A5 A6 khi đó tam giác A1A3 A6 không có cạnh ngắn (mâu thuẫn với đề bài)

Vậy phải tồn tại 3 đoạn ngắn cùng xuất phát từ 1 điểm. Không mất tính tổng quát giả sử 3

đoạn đó l| A1A2 ; A1A3 ; A1A4 . Xét tam giác A2 A3 A4 , theo bài ra phải có 1 cạnh ngắn, giả sử cạnh

A2 A3 là cạnh ngắn thế thì tam giác A1A2 A3 có 3 cạnh ngắn nên chu vi nhỏ hơn 671.3  2013 . Từ

đó ta có điều phải chứng minh.

Ví dụ 23. Trong mặt phẳng cho n đường thẳng m| đôi một không song song với nhau, sao cho qua

giao điểm của mỗi cặp đường thẳng thì có một đường thẳng thứ ba. Chứng minh rằng tất cả n

đường thẳng đã cho dồng quy.

Lời giải

Giả sử tất cả c{c đường thẳng đã cho không

đồng quy tại một điểm. Xét giao điểm A của

hai đường thẳng tùy ý trong n đường thẳng A


d5
đã cho. Kí hiệu hai đường thẳng này là d1 E

và d 2 . Với mỗi đường thẳng d 3 không đi


B C d3
D
qua điểm A ta xét khoẳng cách từ điểm A
d2 d1
đến đường thẳng d 3 . Trong số các cặp điểm d4

A v| đường thẳng d 3 như vậy ta chọn cặp

điểm A v| đường thẳng d 3 mà khoảng cách

từ A đến d 3 là nhỏ nhất.

Tác giả: Nguyễn Công Lợi TÀI LIỆU TOÁN HỌC


90
Website:tailieumontoan.com
Giả sử đường thẳng d 3 cắt hai đường thẳng d1 và d 2 lần lượt tại B và C. Theo giả thiết của bài

toán thì tồn tại đường thẳng d 4 đi qua A v| cắt đường thẳng d 3 lại D. Không mất tính tổng quát

ta có thể giả sử D thuộc đoạn BC. Qua D tồn tại đường thẳng d 5 cắt phần trong của đoạn thẳng

AB hoặc AC. Giả sử đường thẳng d 5 cắt AC tại E nằm trong đoạn AC. Khi đó khoảng cách từ E

đến đường thẳng d 3 nhỏ hơn khoảng cách từ A đến đường thẳng d 3 , điều này mâu thuẫn với

khoảng cách từ A đến d 3 là nhỏ nhất.

Vậy tất cả c{c đường thẳng đã cho đồng quy tại một điểm.

Ví dụ 24. Trên mặt phẳng cho n điểm phân biệt. Chứng minh rằng tồn tại một đường gấp khúc với

c{c đỉnh l| n điểm đã cho m| chúng không tự cắt nhau.

Lời giải

Xét hệ tất cả c{c đường gấp khúc khép kín gồm có n


Ai Aj+1
khúc với n đỉnh chính l| n điểm đã cho. Vì số đường

gấp khúc là hữu hạn nên tồn tại một đường có tổng độ
Aj
dài bé nhất. Gọi đường gấp khúc có tổng độ dài bé nhất
Ai+1
là A1A2 ...An A1 . Khi đó nó chính là một đường gấp

khúc cần tìm và không có hai cạnh nào của đường cắt

nhau.

Thật vậy, ta giả sử hai cạnh của đường gấp khúc cắt nhau tại O là Ai Ai 1 và A j A j1 .

Từ đó ta có Ai Ai 1  A jA j1  Ai A j  Ai 1A j1 . Theo bất đẳng thức tam gi{c thì đường gấp khúc

nhày khép kín và A1A2 ...Ai A jA j1 ...Ai 1A j1A j2 ...An A1 có tổng đội dài ngắn hơn đường gấp

khúc đã chọn. Điều này mâu thuẫn. Do đó b|i to{n được chứng minh.

Ví dụ 25. Cho 5 điểm A, B, C, D, E trong đó không có ba điểm nào thẳng hàng. Chứng minh rằng

luôn tồn tại ba trong năm điểm đã cho l| ba đỉnh của một tam giác có một góc:

a) Nhỏ hơn hoặc bằng 360 b) Lớn hơn hoặc bằng 1080

Lời giải

Tác giả: Nguyễn Công Lợi TÀI LIỆU TOÁN HỌC


91
Website:tailieumontoan.com
a) Từ năm điểm A, B, C, D, E luôn tồn tại ba điểm tạo
A
thành hai tia cùng xuất phát từ một điểm và chứa hai

điểm còn lại.

Giả sử ba điểm B, C, D tạo thành góc CBD chứa hai B


D
điểm A v| E. Khi đó ta thấy

+ Nếu DBC  1080 , khi đó


C
DBA  ABE  EBC  1080 .

Như vậy một trong ba góc DBA; ABE; EBC không

1080
vượt quá  360 .
3

Giả sử ABD  360 , khi đó tam gi{c ABD thỏa mãn yêu cầu bài toán.

+ Nếu DBC  108 0 khi đó trong tan gi{c DBC có BDC  BCD  720 .

Như vậy ít nhất một trong hai góc BDC; BCD không vượt quá 360 .

Khi đó tam gi{c BCD thỏa mãn yêu cầu bài toán.

b) Với năm điểm A, B, C, D, E ta xét hai trường hợp sau:


B
 Trường hợp 1: Trong năm điểm đã cho tồn tại một
tam giác chứa ít nhất một trong hai điểm còn lại. Giả sử

tam giác ADE chứa điểm B.


A
Khi đó ta có ABE  EBD  DBA  3600 . Như vậy ít

nhất một trong ba góc ABE; EBD; DBA lớn hơn 1200 C D

nên tồn tại ít nhất một góc nhở hơn 1080 . Giả sử góc đó

là ABE , khi đó tam gi{c ABE thỏa mãn yêu cầu bài

toán.

 Trường hợp 2: Năm điểm A, B, C, D, E, F tạo th|nh ngũ gi{c lồi ABCDE.

Khi đó tổng các góc của ngũ gi{c lồi là ABC  BCD  CDE  DEA  DAB  5400 .

5400
Như vậy ít nhất một trong năm góc trên lớn hơn  1080 .
5

Tác giả: Nguyễn Công Lợi TÀI LIỆU TOÁN HỌC


92
Website:tailieumontoan.com

Giả sử ABC  1080 , khi đó tam gi{c ABC thỏa mãn yêu cầu bài toán.

Ví dụ 26. Có 2003 vận động viên thi đấu bóng bàn(kết quả chỉ có thắng hoặc thua, không có hòa)

theo thể thức thi đấu vòng tròn(mỗi vận động viên thi đấu với các vận động viên còn lại). Chứng

minh rằng có thể xếp tất cả 2003 vận động viên theo một hàng dọc sao cho người đứng trước

thắng người đứng kề sau.

Lời giải

Xét tất cả các cách xếp một số vận động viên theo hàng dọc sao cho người đứng trước thắng người

đứng sau. Vì số cách xếp là hữu hạn nên luôn tồn tại một cách xếp T có nhiều vận động viên nhất.

Ta sẽ chứng minh cách xếp T có đủ cả 2003 vận động viên.

Giả sử ngược lại, còn có vận động viên A không được xếp trong cách xếp T. Giả sử trong cách xếp

T có n v}n động viên là A1 ; A2 ; A3 ;...; An  2  n  2002  sao cho vận động viên A1 thắng vận

động viên A 2 , vận động viên A 2 thắng vận động viên A 3 ,..., vận động viên A n 1 thắng vận động

viên A n . Do thể thức thi đấu vòng tròn nên vận động viên A sẽ thi đấu với vận động viên A1 .

Nếu vận động viên A thắng thì A sẽ xếp trước A1 ,như vậy tồn tại cách xếp T1 là

A; A1 ; A2 ; A3 ;...; An có nhiều vận động viên cách xếp T. Điều này trái với giả sử ban đầu. Do đó

vận động viên A thua vận đông viên A1 . Như vậy A xếp sau A1 .

Lập luận ho|n to|n tương thự thì ta được A thu các vận động viên A2 ; A3 ;...; An . Như vậy tồn tại

cách xếp T2 là A1 ; A2 ; A3 ;...; An ; A có nhiều vận động viên hơn c{c xếp T, điều này trái với cách

chọn T.

Như vậy cách xếp T có đủ cả 2003 vận động viên.

Ví dụ 27. Trong số những ngừơi đến dự một cuộc họp, người ta nhận thấy rằng nếu hai người

quen nhau thì cả hai người này cùng không quen một người thứ ba. Còn nếu hai người không

quen nhau thì họ cùng quen đúng đúng hai người khác. Chứng minh rằng trong cuộc họp này tất

cả mội người có số người quen bằng nhau.

Lời giải

Tác giả: Nguyễn Công Lợi TÀI LIỆU TOÁN HỌC


93
Website:tailieumontoan.com
Vì số người là hữu hạn nên ta có thể chọn ra một người
A1
B1
quen nhiều người nhất. Giả sử A l| người quen nhiều

người nhất. Ta cần chứng minh mọi người quen A cũng

có số người quen như A. Giả sử c{c người quen của A


B2 A2
là A1 ; A2 ;...; An được minh họa như hình vẽ. Theo giả A

thiết các A i đôi một không quen nhau. Do A1 và A 2 A3

cùng quen A nên A1 và A 2 cùng quen B1 khác A.

Do A1 và A 3 cùng quen A nên A1 và A 3 cùng quen B 2 khác A.

Do A và B1 không quen nhau nên A và B1 có đúng hai người quen chung là A1 và A 2 . Từ đó ta

suy ra được B1 và A 3 không quen nhau và B1 khác B 2 .

Ho|n to|n tương tự ta được A1 và A 4 cùng quen B 3 ( B 3 khác B1 và B 2 );...; A1 và A n cùng

quen Bn 1 ( Bn khác B1 ; B2 ;...; Bn 1 ). Như vậy A1 có n người quen là A; B1 ; B2 ;...; Bn 1 .

Cứ lập luận như vậy thì ta có điều phải chứng minh.

Ví dụ 28. Bên trong một đường tròn có bán kính là 5 lấy bảy điểm phân biệt. Chứng minh rằng

trong bảy điểm đó luôn tồn tại hai điểm mà khoảng cách giữa chúng nhỏ hơn 5. Kết luận bài toán

còn đúng không nếu thay bảy điểm bằng s{u điểm.

Lời giải

 Chia hình tròn thành 6 hình quạt bằng nhau, khi đó theo nguyên lí Dirichlets thì tồn tại ít nhất
hai điểm cùng nằm tròn một hình quạt. Không mất tính tổng quát ta giả sử hai điểm đó l| A, B

nằm trong hình quạt COD. Dễ thấy nếu hai điểm A, B trùng với O thì b|i to{n được chứng minh.

Xét hai điểm A, B không trùng với O. Khi đó trên OC lấy A’ sao cho OA  OA' . Trên OD lấy

điểm B’ sao cho OB  OB' . Từ đó ta có A' B'  AB . Tam giác OA' B' có góc O  600 nên

A'  B'  1200 , như vậy trong hai góc A' và B' có một góc lớn hơn 600 . Không mất tính tổng

quát ta giả sử A'  600 .

Từ đó ta được A'  O nên suy ra A' B'  OB'  OD  5 . Như vậy AB  5 .

 Nếu thay bảy điểm bằng 6 điểm thì ta xét hai trường hợp sau:

Tác giả: Nguyễn Công Lợi TÀI LIỆU TOÁN HỌC


94
Website:tailieumontoan.com
+ Nếu trong 6 điểm đã cho tồn tại một điểm là tâm của đường tròn, khi đó b|i to{n được chứng

minh.

+ Nếu trong s{u điểm không có điểm nào trùng với tâm của đường tròn. Khi đó có hai khả năng

xẩy ra là

- Trong s{u điểm có hai điểm cùng nằm trên một bán kính của đường tròn, khi đó b|i to{n

được chứng minh.

- Trong s{u điểm đã cho không có hai điểm nào cùng nằm trên một b{n kính. Khi đó ta vẽ

s{u b{n kính đi qua s{u điểm đã cho. Cứ hai bán kính gần nhau tao ra một góc ở t}m. Như vậy ta

có sáu góc ở tâm. Theo nguyên lí cực hạn thì trong s{u góc đó tồn tại một góc có số đo bé nhất. Mà

tổng số đo của s{u góc đó l| 3600 nên góc bé nhất không vượt quá 600 . Không mất tnhs tổng

quát ta giả sử COD  600 .

Đến đ}y lập luận tương tự như trên ta có điều phải chứng minh.

Ví dụ 29. Cho 5 số tự nhiên phân biệt sao cho tổng ba số tự nhiên bất kì trong chúng không lớn

hơn tổng hai số còn lại,

a) Chứng minh rằng tất cả 5 số đó đều không nhỏ hơn 5

b) Tìm tất cả các bộ gồm 5 số thỏa mãn đề bài mà tổng của chúng nhỏ hơn 40.

Lời giải

a) Gọi 5 số đó l| a, b,c,d,e . Vì c{c số đã cho ph}n biệt nên không mất tính tổng qu{t ta giả

sử a  b  c  d  e .

Theo giả thiết ta có a  b  c  d  e  a  b  c  d  e  1  a  d  e  1  b  c

Lại có d  c  b  d  c  1; d  b  2  d  b  2 và e  d  c  e  c  2  e  c  2

Do đó ta được a   d  b    e  c   1  5 , suy ra b  5; c  5;d  5; e  5

Vậy tất cả 5 số đã cho đều không nhỏ hơn 5.

b) Xét a  6 , khi đó b  7; c  8;d  9; e  10 suy ra a  b  c  d  e  6  7  8  9  10  40

tr{i với giả thiết. Do đó a  6 nên ta được a  5 .

Khi đó ta có b  c  5  d  e  1  b  c  d  e  4

Mà d  2  b; e  2  c  d  e  4  b  c .

Tác giả: Nguyễn Công Lợi TÀI LIỆU TOÁN HỌC


95
Website:tailieumontoan.com
Kết hợp với bất đẳng thức trên ta được b  d  2; c  e  2

31 31
Suy ra a  b  c  d  e  5  2b  2c  4  40  b  c   2b  1  b7
3 3
Nên ta được b  6; b  7

+ Nếu b  6, ta được d  8  c  7; e  9 . Khi đó ta được bộ số  5; 6; 7; 8; 9 

+ Nếu b  7, ta được d  9; c  8; e  10 . Khi đó ta được bộ số  5; 7; 8; 9;10  .

Ví dụ 30. Cho tập hợp A gồm 21 phần tử là các số nguyên khác nhau thỏa mãn tổng của 11 phần

tử bất kỳ lớn hơn tổng của 10 phần tử còn lại. Biết các số 101 và 102 thuộc A. Tìm tất cả các phần tử

của A.

Lời giải

 
Giả sử A  a1 ; a 2 ; a 3; ...; a 21 với a1 ; a 2 ; a 3; ...; a 21  Z và a1  a 2  a 3  ...  a 21 .

Theo giả thiết ta có a1  a 2  a 3  ...  a11  a12  a13  ...  a 21

 a1  a12  a 2  a13  a 3  ...  a 21  a11 (1)

Mặt khác với x; y  Z và x  y thì y  x  1

 a12  a 2  10, a13  a 3  10,...,a 21  a11  10 (2)

Nên từ (1) suy ra a1  10  10  ...  10  100 nên a1  101 (vì 101  A).

 101  a12  a 2  a13  a 3  ...  a 21  a11  100 nên a12  a 2  a13  a 3  ...  a 21  a11  100 . Kết

hợp với (2) a12  a 2  a13  a 3  ...  a 21  a11  10 (3)

 10  a12  a 2   a12  a11    a11  a10   ...   a 3  a 2   10

 a12  a11  a11  a10  ...  a 3  a 2  1 (4)

Ta có a1  101 mà 102  A nên a 2  102

Kết hợp với (3) và (4) suy ra A  101;102;103;...;121.

Ví dụ 31. Trong dãy số gồm 6 số nguyên dương sắp theo thứ tự tăng dần thỏa mãn số đứng sau là

bội của số đứng trước nó và tổng của sáu số đó l| 79. Tìm dãy số mà số thứ sáu có giá trị lớn nhất.

Tác giả: Nguyễn Công Lợi TÀI LIỆU TOÁN HỌC


96
Website:tailieumontoan.com
Lời giải

Giả sử sáu số nguyên dương a1 ; a 2 ; a 3 ; a 4 ; a 5 ; a6 thỏa mãn thỏa mãn yêu cầu bài toán

Ta có a1  a 2  a 3  a 4  a 5  a 6 và a1  a 2  a 3  a 4  a 5  a 6  79

Ta có nhận xét, nếu a 4  12 thì a 5  2a 4  24 và a6  2a 5  48 .

Khi đó a1  a 2  a 3  a 4  a 5  a 6  12  24  48  79 không thỏa mãn yêu cầu bài toán.

Do đó ta được a 4  12 .

Để ý là trừ số đầu tiên thì các số còn lại trong dãy số trên là bội của số đứng trước nó, do đó ta có

một cách chọn bốn số đầu tiên là a1  1;a 2  2;a 3  4;a 4  8 .

Ta có a 5  ma 4  8m và a6  na 5  8mn với m, n là các số nguyên dương lớn hơn 1.

Mặt khác ta lại có a1  a 2  a 3  a 4  a 5  a 6  79 . Từ đó ta được

1  2  4  8  8m  8mn  79  m 1  n   8

Giải phương trình nghiệm nguyên trên kết hợp với điều kiện số thứ sáu của dãy lớn nhất ta được

m  2; n  3 nên ta được a 6  48 .

Vậy dãy số cần tìm là 1; 2; 4; 8; 16; 48.

Ví dụ 32. Cho 21 số nguyên đôi một khác nhau thỏa mãn điều kiện tổng của 11 số nguyên tùy ý

trong chúng lớn hơn tổng của 10 số nguyên còn lại. Biết rằng trong 21 số đó có một số là 101 và số

lớn nhất là 2014. Tìm 19 số còn lại.

Lời giải

Gọi 21 số đó l| a1 ;a 2 ;a 3 ;...;a 21 . Không mất tính tổng quát ta giả sử a1  a 2  a 3  ...  a 21 . Khi đó

ta được a 21  2014 .

Theo bài ra ta có a1  a 2  a 3  ...  a11  a12  a13  ...  a 21

Nên ta được a1   a12  a 2    a13  a 3   ...   a 21  a11  .

Tác giả: Nguyễn Công Lợi TÀI LIỆU TOÁN HỌC


97
Website:tailieumontoan.com
Do a1 ;a 2 ;a 3 ;...;a 21 là 21 số guyên đôi một khác nhau và a1  a 2  a 3  ...  a 21 nên ta suy ra được

a12  a 2  10; a13  a 3  10; ...; a21  a11  10 .

Do đó ta được a1  100 nên suy ra a1  101 . Theo bài ra trong 21 số trên có một số là 101 nên từ

các kết quả trên ta suy ra được a1  101 .

Và ta có 101   a12  a 2    a13  a 3   ...   a 21  a11 

Do đó 100   a12  a 2    a13  a 3   ...   a 21  a11  nên a12  a 2  a13  a 3  ...  a 21  a11  10

Từ đó ta được a11  a 21  10  2014  10  2004 .

Từ a11  2004 đến a 21  2014 có 11 số nguyên kh{c nhau nên ta được

a12  2005;a13  2006;...;a 20  2013

Từ đó ta được a 2  a12  10  2005  10  1995;a3  a13  10  2006  10  1996;...;a10  2004 .

Vậy 19 số cần tìm là 19 số nguyên liên tiếp từ 1995 đến 2013.

Ví dụ 33. Chọn 100 số tự nhiên khác nhau bất kì sao cho mỗi số đều không vượt qua 2015 và mỗi

số đều chia 17 dư 10. Chứng minh rằng trong 100 số trên luôn chọn được ba số có tổng không lớn

hơn 999.

Lời giải

Giả sử n là số tự nhiên chia 17 dư 10, khi đó n  0 và n có dạng n  17k  10 với k  N .

Gọi 100 số tự nhiên được chọn là 17k1  10;17k2  10;17k3  10;...;17k100  10 .

Không mất tính tổng quát ta giả sử k1  k2  k3  ...  k100 .

Nếu k100  118 thi khi đó 17k100  10  17.118  10  2016 . Do đó k100  117 .

Ta sẽ chứng minh k 3  20 . Thật vậy, giả sử k 3  21 .

Khi đó từ k1  k2  k3  ...  k100 suy ra k4  k3  1; k5  k4  1; k6  k5  1;...; k100  k99  1

Nên từ k 3  21 suy ra k4  21  1  22; k5  22  1  23; k6  23  1  24;...; k100  117  1  118 ,

điều này trái với k100  118 . Do đó k 3  20 . Vì k 3  20 nên suy ra k2  19; k1  18 .


Tác giả: Nguyễn Công Lợi TÀI LIỆU TOÁN HỌC
98
Website:tailieumontoan.com
Với kết quả trên ta chon ba số nhỏ nhất trong 100 số trên là 17k1  10;17k2  10;17k3  10 . Khi đó

ta được 17k1  10  17k2  10  17k3  10   17.18  10   17.19  10   17.20  10  999

Vậy ta luôn chọn được ba số có tổng không lớn hơn 999. b|i to{n được chứng minh.

BÀI TẬP TỰ LUYỆN

Bài 1. Chứng minh rằng nếu tất cả các cạnh của một tam gi{c đều nhỏ hơn 1 thì diện tích tam giác

3
nhỏ hơn .
4

Bài 2. Trên mặt phẳng đã cho 2011 điểm, khoảng cách giữa chúng đôi một khác nhau. Nối mỗi

điểm trong số 2011 điểm này với điểm gần nhất. Chứng minh rằng với mỗi cách nối đó không thể

nhận được một đường gấp khúc khép kín.

Bài 3. Trên mặt phẳng cho 2011 điểm thỏa mãn ba điểm bất kì trong số chúng đều thẳng hàng.

Chứng minh rằng 2011 điểm đã cho l| thẳng hàng.

Bài 4. Trên mặt phẳng có 2011 điểm bất kỳ, ít nhất ba điểm không thẳng hàng. Chứng minh rằng

luôn vẽ được một đường tròn qua ba trong số 2011 điểm đã cho m| 2008 điểm còn lại không nằm

ngo|i đường tròn.

Bài 5. Trên mặt phẳng cho m  n điểm phân biệt(m, n là các số nguyên dương). Chứng minh rằng

tồn tại một đường tròn không đi qua điểm n|o trong c{c điểm trên và chứa đúng n điểm trong

m  n điểm đã cho.

Bài 6. Trong một bảng ô vuông 10x10 người ta viết 100 số nguyên từ 1 đến 100. Mỗi hàng ta chọn

ra số lớn thứ ba trong 10 số của hàng. Chứng minh rằng tổng của 10 số được chọn không nhỏ hơn

tổng của 10 số trong bất cứ một hàng nào của bảng số.

Bài 7. a) Trên một mạng lưới ô vuông vô hạn trên một mặt phẳng, đặt v|o mỗi ô vuông

một số tự nhiên sao cho số trong mỗi ô bằng trung bình cộng của bốn số trong c{c ô vuông

có cạnh kề với ô đó. Chứng minh tất cả c{c số bằng nhau.

b) Nếu số trong mỗi ô bằng trung bình cộng của bốn số trong c{c ô vuông bốn góc.

Ta có thể đặt được tối đa bao nhiêu gi{ trị của c{c số?

Tác giả: Nguyễn Công Lợi TÀI LIỆU TOÁN HỌC


99
Website:tailieumontoan.com
Bài 8. Bảy người c}u được 100 con cá. Biết rằng không có hai người n|o c}u được số c{ như nhau.

Chứng minh rằng có ba người c}u được tổng cộng không ít hơn 50 con c{.

 
Bài 9. Đặt các số nguyên 1; 2; 3;...; n 2 vào một bàn cờ n.n n  2 một cách ngẫu nhiên, mỗi số

đúng một lần, mỗi ô một số. Chứng minh rằng tồn tại hai ô vuông kề nhau(chung cạnh hoặc

chung đỉnh) mà có giá trị khác nhau ít nhất n  1 .

Bài 10. Cho trước một bảng m.n c{c số thực. Một phép biến đổi bảng l| một lần ta đổi dấu

của tất cả c{c số trong một h|ng hay một cột n|o đó của bảng. Chứng minh rằng ta luôn có

thể thực hiện được một dãy hữu hạn c{c phép biến đổi bảng để kết quả thu được l| một

bảng với tổng c{c số trong một dòng, một cột bất kì đều không }m.
Bài 11. Trong một buổi tiệc với một số lượng người tham gia nhất định, xét quan hệ ‚bạn bè‛ theo

nghĩa: ‚Nếu A là bạn của B thì B cũng l| bạn của A‛. Chứng minh rằng người trong buổi tiệc luôn

có thể chia l|m hai nhóm để đưa v|o trong hai phòng kh{c nhau sao cho: Với mỗi người trong một

phòng bất kì, ít nhất một nửa số bạn của người đó ở phòng còn lại.

Bài 12. Trong các ô của bảng vuông kích thước n.n ô vuông, người ta viết các số sao cho tổng của

các số có mặt trong các ô của một ‚chữ thập‛ (tức là hình gồm một hàng và một cột) bất kỳ không

nhỏ hơn a. Tính giá trị nhỏ nhất của tổng các ô trong bảng.

Bài 13. Lấy 2011 điểm thuộc miền trong của tứ gi{c để cùng với 4 đỉnh ta được 2015 điểm, trong

đó không có ba điểm nào thẳng hàng. Biết diện tích của tứ gi{c ban đầu là 1cm2. Chứng minh rằng

1
tồn tại một tam gi{c có 3 đỉnh lấy từ 2015 điểm đã cho có diện tích không vượt quá cm2.
4024

Bài 14. Trong mặt phẳng cho tập hợp P gồm hữu hạn điểm bất kỳ không cùng nằm trên

một đường thẳng. Xét tất cả c{c đường thẳng đi qua hai điểm bất kỳ của P. Chứng minh

luôn có ít nhất một đường thẳng chỉ đi qua hai điểm của P.
Bài 15. Trên mặt phẳng kẻ 2016 đường thẳng sao cho không có ba đường n|o đồng quy. Tam giác

tao bởi ba đường thẳng trong c{c đường thẳng trên được gọi là tam giác xanh nếu nó không bị

đường thẳng cào trong số c{c đường thẳng còn lại cắt. Gọi k là số tam giác xanh có thể có được từ

c{c đường thẳng trên.

a) Chứng minh rằng số tam gi{c xanh không ít hơn 672.

b) Chứng minh rằng số tam gi{c xanh không ít hơn 1344.

Tác giả: Nguyễn Công Lợi TÀI LIỆU TOÁN HỌC


100
Website:tailieumontoan.com
Bài 16. Chứng minh rằng nếu a, b l| c{c số nguyên dương nguyên tố cùng nhau thì tồn tại c{c số

nguyên x, y sao cho ax  by  1 .

Bài 17. Trên một vòng tròn người ta xếp ít nhất bốn số thực không âm có tổng bằng 1. Chứng minh

1
rằng tổng của tất cả các tích hai số kề nhau không lớn hơn .
4

Bài 18. Cho n là một số nguyên dương v| c{c số nguyên dương a1 ;a 2 ;a 3 ;...;a n có tổng bằng

2n  1 . Chứng minh rằng tồn tại một số số trong các số nguyên dương trên có tổng bằng n.

Bài 19. Cho 5 số nguyên phân biệt sao cho tổng ba số bất kì trong chúng luôn lớn hơn tổng hai số

còn lại. Tìm giá trị nhỏ nhất của tích năm số nguyên đó.

HƯỚNG DẪN GIẢI

Bài 1.

Gọi A là góc nhỏ nhất của tam giác ABC thì B

suy ra BAC  600 .

1 1
Ta có S ABC  BH.AC  AB.AC.sin A
2 2
Do đó ta được C H A

1 1 3 3
S ABC  AB.AC.sin 600  .1.1. 
2 2 2 4
Suy ra điều phải chứng minh

Bài 2.

Giả sử ngược lại với cách nối đó, chúng ta nhận được B
một đường thẳng gấp khúc khép kín.

Gọi AB là mắt lớn nhất của đường gấp khúc khép kín
A
này. Tức l| đoạn thẳng lớn nhất trong c{c đoạn thẳng

tạo nên đường gấp khúc trên. Giả sử AC và BD là hai


D
C
mắt kề với mắt AB. Khi đó ta thấy

 Nếu AC  AB thì B không phải l| điểm gần nhất của


A.

Tác giả: Nguyễn Công Lợi TÀI LIỆU TOÁN HỌC

You might also like